Anda di halaman 1dari 178

CASSY BARRAS

JURISDICTION CASES:
PEOPLE v. MARIANO
71 SCRA 600

Military Tribunal, the Court of


First Instance of Bulacan had lost
jurisdiction over the case against
him.
Judge issued an Order granting
the motion to quash on the ground
of lack of jurisdiction.
ISSUE:

FACTS:
Accused Mariano was appointed as
Liaison Officer by the then
incumbent
Municipal
Mayor,
Constantino Nolasco, acting for
and in behalf of the municipality of
San Jose del Monte, Bulacan and
authorized to receive and be
receipted for US excess property
of USAID/NEC for the use and
benefit
of
said
municipality.
Mariano, instead of delivering it to
the
Office
of
the
Mayor,
misappropriated, misapply and
converteed the said items for his
personal benefit.
Office of the Provincial Fiscal of
Bulacan filed an Information
accusing
private
respondent
herein Hermogenes Mariano of
estafa.
Mariano filed an MTQ
claiming that the items which were
the
subject
matter
of
the
Information against him were the
same items for which Mayor
Constantino A. Nolasco of San Jose
del Monte, province of Bulacan,
was indicted before a Military
Commission under a charge of
malversation of public property,
and for which Mayor Nolasco had
been found guilty and sentenced to
imprisonment in as much as the
case against Mayor Nolasco had
already been decided by the

Does the civil court and military


tribunals
exercise
concurrent
jurisdiction over estafa case of
goods not exceeding 6,000 pesos
HELD:
The settled
rule
that
the
jurisdiction
of
a
court
is
determined by the statute in force
at the time of the commencement
of the act on. In the case at bar, it
is rightly contended by the
Solicitor General that at the time
Criminal Case was filed, with CFI
of Bulacan, that was December 18,
1974, the law in force vesting
jurisdiction upon said court was
the Judiciary Act of 1948, the
particular provision of which was
not affected one way or the other
by any Presidential issuances
under Martial Law. General Order
No. 49 dated October 4, 1974,
which repeals General Order No.
12 and the latters amendments
and
related
General
Orders
inconsistent with the former,
redefines
the
jurisdiction
of
military tribunals over certain
offenses,
and
estafa,
and
malversation are not among those
enumerated therein.
In other
words the Military Commission is

not vested with jurisdiction over


the crime of estafa.
Fundamental is the fact that the
case does not involve a situation
involving two tribunals vested with
concurrent jurisdiction over a
particular crime so as to apply the
rule that the court or tribunal
which first takes cognizance of the
case acquires jurisdiction thereof
exclusive of the other.
The
Military Commission as stated
earlier is without power or
authority to hear and determine
the particular offense charged
against
respondent
Mariano,
hence, there is no concurrent
jurisdiction
between
it
and
respondent court to speak of.
Estafa
as
described
in
the
information falls within the sole
exclusive
jurisdiction
of
civil
courts.

CASSY C. BARRAS
PEOPLE vs.
SCRA 838

CHUPECO,

10

FACTS:
In the City of Manila, the accused
Jose L. Chupeco executed a
Chattel mortgage as to some of his
properties located at sitio Saguing,
Dinalupihan, Bataan in favor of
Agricultural and Industrial Bank,
whose capital, assets, accounts,

contracts and choses in action


were subsequently transferred to
the complainant Rehabilitation
Finance Corporation, to secure a
loan of P20,000.00.
The mortgagee alleged that the
Chupeco willfully, unlawfully and
feloniously, with intent to defraud
the said corporation, pledged and
incumbered, cause to be pledged
and incumbered the pledged
personal properties to Mateo B.
Pinile without fully satisfying or
paying
the
mortgage,
and,
thereafter knowingly transferred
and removed the properties to the
municipality of Subic, Zambales.
The accused moved to quash the
information on the ground that
more than one offense is charged
and that the court had no
jurisdiction.
The Court denied the motion.

After the case was partly tried,


the defense counsel and the fiscal
entered into an agreement to have
the information amended to the
effect that the charge be only for
removal of properties mortgaged,
eliminating the portion referring to
pledging already pledged property.
The information was not amended.
The accused then filed a motion to
dismiss invoking the agreement
having the information amended.
The court denied it and ordered
that the case be tried on the
charge
"of
having
pledged
property
which
had
been
previously pledged or mortgaged".
After trial, the court found the

accused guilty
charged.

of

the

offense

It is argued that since the place


where the chattels were, as well as
the site to which they were moved,
are both outside of Manila, the
courts of the latter acquired no
jurisdiction to try the case,
because the offense was not
committed within the Manila
territory.
ISSUE:
Whether or not RTC Manila has
jurisdiction over Chupecos case
HELD:
No. The original terms of the
charge alleged (and it is not
disputed) the crime of repledging
already
encumbered
property
without the creditor's consent, and
one of the essential ingredients of
the offense (the execution of the
first
mortgage)
having
been
alleged, to have taken place in
Manila, the court of first instance
of that city acquired jurisdiction
over the offense under the Rules of
Court (People vs. Mission, 48 O.G.,
1331; Rule 110, section 9).
It is well-established that once
vested, the jurisdiction is not tolled
by subsequent amendment or
which in this case amounted to no
more than an avowal by the
prosecution that it could not
establish the other elements of the
offense.
Furthermore, the court actually
rejected the defense motion to

dismiss, and directed that the case


be tried on the original charge of
repledging
property
already
encumbered or mortgaged. The
accused obeyed that directive, and
by so doing it renounced the claim
that the information had been so
amended as to discard that
particular averment.

MANILA RAILROAD CO.


ATTY.GENERAL, 20 Phil 523

vs

FACTS:
Action brought by the plaintiff
Manila Railroad Company in the
Court of First Instance of Laguna
for the purpose of condemning
certain parcels of land which will
be used in the extension of its
railway line from Calamba to Santa
Cruz.
Plaintiff began an action in CFI
Tarlac for the condemnation of
certain real estate, stated by the
plaintiff in his complaint to be
located in Tarlac. It is alleged in
the complaint that the plaintiff is
authorized by law to construct a
railroad line "from Paniqui to
Tayug in the Province of Tarlac,"
and it is for the purpose of
condemning
lands
for
the
construction of line that this action
is brought.
The complaint states that before
beginning the action the plaintiff
had caused to be made a thorough
search in the office of the registry
of property and of the tax where

the lands sought to be condemned


were located and to whom they
belonged. As a result of such
investigations the plaintiff alleged
that the lands in question were
located in the Province of Tarlac.
Plaintiff gave notice to the
defendants that a motion would be
made to the court to dismiss the
action upon the ground that the
court had no jurisdiction of the
subject matter, it having just been
ascertained by the plaintiff that
the land sought to be condemned
was situated in the Province of
Nueva Ecija, instead of the
Province of Tarlac, as alleged in
the complaint. This motion was
heard and, after due consideration,
the trial court dismissed the action
upon the ground presented by the
plaintiff.

ISSUE:
1. Whether or not the CFI Tarlac
can take cognizance of an action
by a railroad company for the
condemnation
of
real
estate
located in another province.
HELD:
1. Yes, CFI Tarlac has power and
authority to take cognizance of
condemnation
of
real
estate
located in another province.
Sections 55 and 56[1] of Act No.
136 of the Philippine Commission
confer
perfect
and
complete
jurisdiction upon the CFI of these
Islands with respect to real estate

in the Philippine Islands. Such


jurisdiction is not made to depend
upon
locality.
There
is
no
suggestion of limitation. The
jurisdiction is universal. It is
nowhere suggested, much less
provided, that a CFI of one
province, regularly sitting in said
province, may not under certain
conditions take cognizance of an
action arising in another province
or of an action relating to real
estate located outside of the
boundaries of the province to
which it may at the time be
assigned.
Venue is not connected with
jurisdiction
over
the
subject
matter. If
the
parties
consent thereto there
is no
legal
reason why
the CFI
o f M a n i l a m a y n o t cognizance
of and determine a controversy
affecting the tile to or an interest
in real estate situated in another
province. With the consent of the
defendants, express or implied, the
venue may be laid and the action
tried in any province selected by
the plaintiff. Any one of the
defendants who have lands lying in
another province may also choose
the venue. In such case, the action
as to all the defendants not
objecting would continue as to the
objecting defendants.
The plaintiff having brought the
action must submit itself to the
jurisdiction of the court. It took
advantage of the situation itself
created and took possession of the
land while the case is being
litigated. It is estopped from

alleging that the court has no


jurisdiction over it.

FUKUZUME vs. PEOPLE, 474


SCRA 580
FACTS:
A petition for certiorari finding the
accused-appelant guilty beyond
reasonable doubt of a crime of
estafa.
In an Information filed with
the RTC of Makati, Fukuzume was
charged with estafa committed as
follows:
That
sometime
in
the
Municipality of Makati, Metro
Manila, Philippines, a place within
the jurisdiction of this Honorable
Court, accused, with intent to
prejudice and defraud Javier Yu y
Ng, unlawfully and feloniously
make false representation and
fraudulent manifestation that he is
the duly authorized representative
of Furukawa Electric Co. Ltd., in
the
Philippines,
and
was
authorized to sell excess aluminum
conductor materials not being
used by NAPOCOR and Furukawa,
the accused knowing full well that
those representations were false
and were only made to induce and
convince said Javier Yu y Ng to buy
said materials, who believing said

representations to be true, gave


and delivered the total amount
of P424,000.00 but the accused
once in possession of the money,
applied and used for his own
personal use and benefit and
despite repeated demands failed
and refused and still fails and
refuses to account for, to the
damage and prejudice of Javier Yu
y Ng.
Upon
being
arraigned,
Fukuzume pleaded not guilty. Trial
ensued.
Trial
court
found
Fukuzume guilty as charged. C.A.
promulgated its decision affirming
the findings.
ISSUE:
Whether or not the RTC of Makati
have jurisdiction over the case of
Fukuzume.

HELD:
No. The Court found nothing
in the direct or cross-examination
of Yu to establish that he gave any
money to Fukuzume or transacted
business with him with respect to
the subject aluminum scrap wires
inside or within the premises of
the Intercontinental Hotel in
Makati, or anywhere in Makati for
that matter.

Venue in criminal cases is an


essential element of jurisdiction.
It is a fundamental rule that for
jurisdiction to be acquired by
courts in criminal cases the
offense
should
have
been
committed or any one of its
essential ingredients took place
within the territorial jurisdiction of
the court.

Territorial
jurisdiction
in
criminal cases is the territory
where the court has jurisdiction to
take cognizance or to try the
offense
allegedly
committed
therein by the accused. Thus, it
cannot take jurisdiction over a
person charged with an offense
allegedly committed outside of
that limited territory. Furthermore,
the jurisdiction of a court over the
criminal case is determined by the
allegations in the complaint or
information. And once it is so
shown, the court may validly take
cognizance of the case. However,
if the evidence adduced during
the trial show that the offense
was
committed
somewhere
else, the court should dismiss
the
action
for
want
of
jurisdiction.

The criminal information against


Fukuzume was filed with and tried
by the RTC of Makati. Thus, having
found that the RTC of Makati did
not have jurisdiction to try the
case against Fukuzume.

PEOPLE vs. MAGALLANES, 249


SCRA 212
FACTS:
Spouses Dumancas, P/Col. Nicolas
Torres, Police Inspector Abeto and
other accused were charged of
murder.
The cases were consolidated and
the accused pleaded not guilty and
filed motions for bail.. After the
prosecution rested its case, the
trial court received evidence for
the accused, but the reception of
evidence was suspended because
of the motions for inhibition of
Judge Gravilles filed by several
accused.
Garvilles
voluntarily
inhibited himself and the case was
re-raffled.
However,
the
prosecution
moved
for
the

transmittal of the records to the


Sandiganbayan
because
the
offenses charged were committed
in relation to the office of the
accused PNP officers. The trial
court
ruled
that
the
Sandiganbayan does not have
jurisdiction
because
the
informations do not state that the
offenses charged were committed
in relation to the office of the
accused PNP officers and denied
the Motion for the Transfer of
Records to Sandiganbayan The
cases were then re-raffled to
Branch 49 of the Regional Trial
Court of Bacolod. The prosecution
filed a petition for certiorari,
prohibition and mandamus with
prayer for a temporary restraining
order, challenging the refusal of
the judge to transfer the cases to
the Sandiganbayan.
ISSUE:
Whether or not the Sandiganbayan
has the jurisdiction over this case.
HELD:
An offense may be considered as
committed in relation to the office
if it cannot exist without the office,
or if the office is a constituent
element of the crime as defined in
the statute, such as, for instance,
the crimes defined and punished in
Chapter Two to Six, Title Seven, of
the Revised Penal Code. The
offense
must
be
intimately
connected with the office of the
offender, and we further intimated
that the fact that the offense was
committed in relation to the office

must
be
information.

alleged

in

the

The allegation of taking advantage


of
his
position"
or
"taking
advantage of their respective
positions" incorporated in the
informations is not sufficient to
bring the offenses within the
definition of "offenses committed
in relation to public office." In
Montilla v. Hilario, such an
allegation was considered merely
as an allegation of an aggravating
circumstance, and not as one that
qualifies the crime as having been
committed in relation to public
office. Despite the allegation that
the
accused
public
officers
committed
the
crime
of
falsification of official document by
"taking advantage of their official
positions, the Sandiganbayan had
no jurisdiction over the case
because" [t]he information [did]
not allege that there was an
intimate connection between the
discharge of official duties and the
commission
of
the
offense.
Accordingly,
for
lack
of
an
allegation in the informations that
the offenses were committed in
relation to the office of the
accused PNP officers or were
intimately connected with the
discharge of the functions of the
accused, the subject cases come
within the jurisdiction of the
Regional Trial Court and not of the
Sandiganbayan as insisted by the
petitioner.

accounting of the transactions and


remit the same to the principal
office of CBIC in Manila. However,
an audit of Buayas account
showed that there was a shortage
in the amount of P358,850.7. As a
result, she was charged with
estafa before the Regional Trial
Court of Manila has no jurisdiction
because she is based in Cebu City,
but the same was denied by
respondent Judge Polo.
ISSUE:
Whether or not the Regional Trial
Court of Manila has jurisdiction to
try the criminal case against
petitioner Buaya.
HELD:

BUAYA vs. POLO, 169 SCRA 471


FACTS:
Petitioner Solemnidad Buaya was
an insurance agent of Country
Bankers Insurance Corporation
(CBIC) and was authorized to
collect premiums for and in behalf
of CBIC then make a report and

The allegation in the complaint or


information
determine
the
jurisdiction of the court in criminal
cases. 14(a) of Rule 110 provides
that the action in all criminal
prosecutions shall be instituted
and tried in the court of the
municipality or province where the
offense was committed or where
any of its essential elements took
place. The subject information
charges
Buaya
with
estafa
committed during the period of
1980 to June 15, 1982 inclusive in
the City of Manila, Philippines. The
claim of Buaya that RTC Manila
has no jurisdiction because she is
based in Cebu City is without
merit.
Clearly, RTC Manila has no
jurisdiction since the respondents
principal place of business in
Manila and Buayas failure to

remit
the
premiums
caused
damage
and
prejudice
to
respondent in Manila. Besides,
estafa is a continuing offense
which may be prosecuted at any
place where any of the essential
elements of the crime took place.
REPUBLIC
OF
THE
PHILIPPINES
vs.
HON. DELFIN VIR. SUNGA, as
Presiding Judge, CFI Branch I,
Camarines
Sur,
ARISTON
ANADILLA, RAFAEL ANADILLA
and JOSE ANADILLA
FACTS:
The complainant Jose Dadis filed a
case for the attempted homicide
against
Ariston
and
Rafael
Anadilla. While
the
case
is
pending, Jose Dadis filed an
affidavit of desistance and was no
longer
interested
in
the
prosecution
of the
case. Jose
said that he had forgiven the
accused and that his material
witness could no longer be
contracted the court then lifted the
order of arrest, cancelled the bail
bond and ordered the release
of the accused. The Provincial
Fiscal
moved
for
the
reconsideration of the order of
dismissal, which was denied,
hence this appeal
ISSUE:
Whether the court a quo may
dismiss a criminal case on the
basis of an affidavit of desistance
executed by the offended party, but

without a motion to dismiss filed


by the prosecuting fiscal?
HELD:
"The
rule
therefore
in
this
jurisdiction
is
that
once
a
complaint or information is filed in
Court and disposition of the case
as its dismissal or the conviction or
acquittal of the accused rests in
the sound discretion of the Court.
Although the fiscal retains the
direction and control of the
prosecution of criminal cases even
while the case is already in Court
he cannot impose his opinion on
the trial court. The Court is the
best and sole judge on what to do
with the case before it. The
determination of the case is within
its exclusive jurisdiction and
competence. A motion to dismiss
the case filed by the fiscal should
be addressed to the Court who has
the option to grant or deny the
same. It does not matter if this is
done
before
or
after
the
arraignment of the accused or that
the motion was filed after a
reinvestigation
or
upon
instructions of the Secretary of
Justice who reviewed the records
of the investigation.
To avoid similar situations, the
Court takes the view that, while
the Crespo doctrine has settled
that the trial court is the sole
judge on whether a criminal case
should be dismissed (after the
complaint or information has been
filed in court), still, any move on
the part of the complainant or
offended party to dismiss the
criminal case, even if without

objection of the accused, should


first be referred to the prosecuting
fiscal for his own view on the
matter. He is, after all, in control
of the prosecution of the case and
he may have his own reasons why
the case should not be dismissed.
It is only after hearing the
prosecuting fiscals view that the
Court should exercise its exclusive
authority to continue or dismiss
the case.
MIRANDA vs. TULIAO, G.R. No.
158763, March 31, 2006.
FACTS:
Two
burnt
cadavers
were
discovered in Isabela which were
later identified as the bodies of
Vicente Bauzon and Elizer Tuliao,
son of the private respondent
Virgilio Tulio who is now under the
witness protection program.
Two informations for murder were
filed against the 5 police officer
including SPO2 Maderal in RTC
of Santiago City. The venue was
later transferred to Manila. RTC
Manila convicted all the accused
and sentenced them 2 counts of
reclusion perpetua except SPO2
Maderal who was yet to be
arraigned at that time, being at
large. Upon automatic review, the
SC acquitted the four accused on
the ground of reasonable doubt.
Then Maderal was arrested. He
executed a sworn confession and
identified the herein petitioner
Miranda and 4 others responsible
for the death of the victims.

Respondent Tuliao then filed


a criminal complaint for murder
against the petitioners.
Warrant of arrest against the
petitioners and SPO2 Maderal. In
the hearing of the urgent motion,
Judge Tumalian noted the absence
of petitioners and issued a Joint
order denying the said urgent
motion on the ground that since
the
court
did
not
acquire jurisdiction
over their persons,
the motion
cannot be properly
heard by the court. The
petitioners appealed the resolution
of the Public prosecutor to the
DOJ.
The new Presiding took over
the case and issued a Joint Order
reversing the Joint Order of Judge
Tumalian. He also ordered the
cancellation of the warrant of
arrest. Respondent Tuliao filed a
petition for certiorari, mandamus
and prohibition with a prayer for
TRO seeking to enjoin Judge
Anghad from further proceeding of
the case and seeking to nullify the
Joint Orders of the said Judge. The
SC issued a resolution granting the
prayer. Notwithstanding the said
resolution, Judge Anghad issued a
Joint
Order
dismissing
the
information against the petition.
Respondent Tuliao filed a motion
to cite Judge Anghad in contempt.
The SC referred the said motion to
the CA. The CA rendered the
assailed
decision
granting the
petition
and
ordering
the
reinstatement of the criminal cases
in the RTC of Santiago City as well
as the issuance of warrant of
arrest.

ISSUE:
Whether or not an accused
can seek judicial relief if he does
not submit to the jurisdiction of
the court
HELD:
As a general rule, one who seeks
an affirmative relief is deemed to
have submitted to the jurisdiction
of
the
court.
Seeking
an
affirmative relief in court, whether
in civil or criminal proceedings,
constitutes voluntary appearance
There is, however, an exception to
the rule that filing pleadings
seeking
affirmative
relief
constitutes voluntary appearance,
and the consequent submission of
one's person to the jurisdiction of
the court. This is in the case of
pleadings
whose
prayer
is
precisely for the avoidance of the
jurisdiction of the court, which
only leads to a special appearance.
These pleadings are: (1) in civil
cases, motions to dismiss on the
ground of lack of jurisdiction over
the person of the defendant,
whether or not other grounds for
dismissal are included; (2) in
criminal cases, motions to quash a
complaint on the ground of lack of
jurisdiction over the person of the
accused; and (3) motions to quash
a warrant of arrest. The first two
are consequences of the fact that
failure
to
file
them
would
constitute a waiver of the defense
of lack of jurisdiction over the
person. The third is a consequence
of the fact that it is the very
legality of the court process
forcing the submission of the
person of the accused that is the

very issue in a motion to quash a


warrant of arrest
JESSER CAPARAS
PEOPLE OF THE PHILIPPINES
v. ALFREDO L. BENIPAYO
G.R. No. 154473 April 24, 2009
Topic: Jurisdiction
Facts:
Alfredo Benipayo, then Chairman
of the COMELEC, delivered a
speech in the Forum on Electoral
Problems: Roots and Responses in
the Philippines held in UP Diliman.
The same was published in Manila
Bulletin. In the same speech he
allegedly delivered libelous speech
against
Photokina
Marketing
Corporation regarding anomalous
contract contracted by the latter.
Hence, People of the Philippines
with
Photokina
Marketing
Corporation
filed
a
criminal
complaint
for
libel
against
Benipayo
in
RTC.
Benipayo
questioned the jurisdiction of RTC
to try the libel case alleging that
the speech was delivered in
relation to his office and hence, it
should be the Sandiganbayan that
should have jurisdiction over the
case.
Issue:
Whether or not the Sandiganbayan
has jurisdiction over the case.
Held:
No.
The jurisdiction of the court to
hear and decide a case is
conferred by the law in force at
the time of the institution of
the action, unless a latter statute
provides
for
a
retroactive
application thereof.

Article 360 of the Revised Penal


Code
(RPC),as
amended
by
Republic Act No. 4363, is explicit
on which court has jurisdiction to
try cases of written defamations,
thus:
The criminal and civil action for
damages in cases of written
defamations as provided for in this
chapter, shall
be
filed
simultaneously or separately with
the court of first instance [now, the
Regional Trial Court] of the
province or city where the libelous
article
is
printed
and
first
published or where any of the
offended parties actually resides at
the time of the commission of the
offense.
A subsequent enactment of a law
defining the jurisdiction of other
courts cannot simply override, in
the absence of an express repeal
or
modification,
the
specific
provision in the RPC vesting in the
RTC, as aforesaid, jurisdiction over
defamations in writing or by
similar means. The grant to the
Sandiganbayan of jurisdiction over
offenses committed in relation to
(public) office, similar to the
expansion of the jurisdiction of the
MTCs, did not divest the RTC of its
exclusive and original jurisdiction
to try written defamation cases
regardless of whether the offense
is committed in relation to office
MAYOR FRANCISCO LECAROZ
v. SANDIGANBAYAN
Topic: JURISDICTION
FACTS:
Petitioner was charged with the
crime of grave coercion in an

information
filed
before
the
Sandiganbayan. The complaint
alleged that the accused, a public
officer, being then the mayor of
Sta. Cruz, Marinduque, taking
advantage of his public position
and which offense was committed
in relation to his office, did then
and there, willfully, unlawfully and
feloniously take over the operation
and control of the gasoline station
owned by Pedro Par, sell the
gasoline therein to the public
issuing the invoices of said
gasoline station and some pieces
of yellow pad paper for the
purpose, and padlock dispensing
pump thereof without authority of
law, depriving Pedro Par of the
possession and exercise of a lawful
trade or occupation. Petitioner
filed a motion to quash the
information principally on the
ground that the respondent court
lacks jurisdiction to entertain the
case and that it should have been
filed with the ordinary courts in
Marinduque where the alleged
crime was committed. However
the Motion to Quash was then
denied.
ISSUE:
Whether or not Sandiganbayan has
jurisdiction over the case.
HELD:
Yes. The crime for which petitioner
is charged, grave coercion, is
penalized by arresto mayor and a
fine not exceeding P500.00 under
the first paragraph of Article 286
of the Revised Penal Code, as
amended.
Respondent
court,

pursuant to the provisions of


Section 4 of Presidential Decree
No. 1606, as amended, has
concurrent jurisdiction with the
regular courts. Well established
is the rule that once a court
acquires jurisdiction in a case
where
said
jurisdiction
is
concurrent with another court,
it must continue exercising the
same to the exclusion of all
other courts. In Laquian vs.
Baltazar, 31 SCRA 551, We ruled
that in case of concurrent
jurisdiction, it is axiomatic that the
court first acquiring jurisdiction
excludes the other courts. Thus,
respondents denial of petitioners
request for the transfer of the case
to the Court of First Instance of
Marinduque was well-grounded
and certainly not a grave abuse of
discretion.
It is true that on March 23, 1983, P
residential
Decree
No.
1861
amended Presidential Decree No.
1606, and it provides, among
others, that where the penalty for
offenses or felonies committed by
public officers and employees in
relation totheir office does not
exceed
prision correccional
or
imprisonment for six (6) years, or
fine of P6,000.00, they are no
longer within the concurrent
jurisdiction of respondent court
and the regular courts but are now
vested in the latter. However,
Section 2 of said Presidential
Decree
No.
1861
states:
SECTION
2
All cases pending in the Sandigan
bayan or in the appropriate courts
as of the date of the effectivity of
this Decree shall remain with and
be disposed of by the courts where

they are pending. The information


against petitioner was filed in
1980 therefore, respondent court
retains jurisdiction over the case
subject of instant petition.

PANFILO M. LACSON v. THE


EXECUTIVE SECRETARY
G.R. No. 128096 January 20,
1999
Topic: Jurisdiction

Whether or not Sandiganbayan has


jurisdiction
over the case.

FACTS:

To fall under the exclusive original


jurisdiction of the Sandiganbayan,
the following requisites must
concur: (1) the offense committed
is a violation of (a) R.A. 3019, as
amended (the Anti-Graft and
Corrupt Practices Act), (b) R.A.
1379 (the law on ill-gotten wealth),
(c) Chapter II, Section 2, Title VII,
Book II of the Revised Penal Code
(the law on bribery), (d) Executive
Order Nos. 1, 2, 14, and 14-A,
issued in 1986 (sequestration
cases), or (e) other offenses or
felonies
whether
simple
or
complexed with other crimes; (2)
the
offender
committing
the
offenses in items (a), (b), (c) and
(e) is a public official or employee
holding any of the positions
enumerated in paragraph a of
Section 4; and (3) the offense
committed is in relation to the
office.
In People v. Montejo, the court
held that an offense is said to have
been committed in relation to the
office
if
it
is
intimately
connected with the office of the
offender and perpetrated whie he
was in the performance of his
official functions. This intimacy
must be alleged in he information,
which determines the jurisdiction
of the court. The controlling factor
is the specific factual allegationsin
the information that would sow the
close intimacy of the discharge of
the accused official duties and the
commission of the offense charged.

Eleven persons believed to be


members of the KuratongBaleleng
gang, reportedly an organized
crime syndicate which had been
involve in a spate of bank
robberies in Metro Manila, were
slain along Commonwealth Avenue
in Quezon City by elements of the
Anti-Bank
Robbery
and
Intelligence Task Group (ABRITG).
The ABRITG was composed of
police officers with Presidential
Anti-Crime Commission Task Force
Habagat (PACC-TFH) headed by
petitioner Chief Superintendent
Panfilo M. Lacson. An information
for murder was filed against
petitioner after an allegation that
was transpired was a summary
execution (or a rub out) and not a
shoot-out
between
the
KuratongBaleleng gang members
and the ABRITG. Ombudsman filed
on
March
1,
1996
eleven
amended informationsbefore the
Sandiganbayan, wherein petitioner
was charged only as an accessory.
Accused filed separate motions
questioning the jurisdiction of the
Sandiganbayan,
asserting
that
under the amended information,
the
cases
fall
within
the
jurisdiction of the Regional Trial
Court and not of Sandiganbayan.
ISSUE:

HELD:

It does not even matter the phrase


committed in relation to his office
appears in the information or not.
In the case at bar, what the
amended information contains is a
mere allegation that the offense
was committed by the accused
public officer in relation to his
office and that is not sufficient.
Such phrase is merely a conclusion
of law. Since it was not proven that
the
crime
of
murder
was
committed in the discharge of their
duties, the Sandiganbayan does
not have jurisdiction over the case

RULE 110 PROSECUTION OF


OFFENSES
FUKUZUME v. PEOPLE
G.R. No. 143647, November 11,
2005
TOPIC: Venue/Jurisdiction
FACTS:
Private complainant Javier Ng Yu
is a businessman engaged in
buying and selling aluminum scrap
wires. Sometime in 1991, Yu,
accompanied by a friend, Mr.
Jovate, went to the house of the
accused-appellant
Yusuke
Fukuzume in Paraaque. Jovate
introduced Fukuzume to Yu telling
the latter that Fukuzume is from
Furukawa Electric Corporation
and that he has at his disposal
aluminum
scrap
wires.
Fukuzumethen told Yu that the
scrap wires belong to Furukawa
but they are under the care of
NAPOCOR. Believing Fukuzumes
representation to be true, Yu
agreed to buy the aluminum scrap
wires from Fukuzume. Thereafter
on 1992, Fukuzume gave Yu a
letter, authorizing Fukuzume to
dispose
of
excess
aluminum
conductor
materials.
Fukuzumethen
agreed
to
accompany Yu when the latter is
going to take the aluminum scrap
wires
from
the
NAPOCOR
compound. When Yu arrived at the
NAPOCOR compound, Fukuzume
was nowhere to be found. Yu
proceeded to show the documents
of authorization to NAPOCOR
personnel but the people from
NAPOCOR did not honor the

authorization letter. Unable to get


the aluminum scrap wires from the
NAPOCOR compound, Yu talked to
Fukuzume and asked from the
latter the refund of the money he
paid him.Failing to refund the
money, Yu then filed a criminal
complaint in the Regional Trial
Court (RTC) of Makati against
Fukuzume for the crime of estafa.
ISSUE:
Whether or not RTC of Makati has
jurisdiction over the case.
HELD:
No evidence was presented that Yu
gave any money to Fukuzume or
transacted business with him with
respect to the subject aluminum
scrap wires inside or within the
premises of the Intercontinental
Hotel in Makati, or anywhere in
Makati for that matter.
It is a fundamental rule that for
jurisdiction to be acquired by
courts in criminal cases the
offense
should
have
been
committed or any one of its
essential ingredients took place
within the territorial jurisdiction of
the court. Territorial jurisdiction in
criminal cases is the territory
where the court has jurisdiction to
take cognizance or to try the
offense
allegedly
committed
therein by the accused. Thus, it
cannot take jurisdiction over a
person charged with an offense
allegedly committed outside of
that limited territory. Furthermore,
the jurisdiction of a court over the
criminal case is determined by the
allegations in the complaint or
information. And once it is so

shown, the court may validly take


cognizance of the case. However,
if the evidence adduced during
the trial show that the offense
was
committed
somewhere
else, the court should dismiss
the
action
for
want
of
jurisdiction.
Venue in criminal cases is an
essential
element
of
jurisdiction; If the evidence
adduced during the trial show
that the offense was committed
outside
its
territorial
jurisdiction, the court should
dismiss the action for want of
jurisdiction.

REPUBLIC
OF
THE
PHILIPPINES
v.
HON.
ASUNCION, MANIO
G.R. No. L-108208 March 11,
1994
TOPIC: Venue/Jurisdiction
FACTS:
Private
respondent
Alexander
Dionisio y Manio, a member of the
PNP assigned to the Central Police
District Command Station 2 in
Novaliches, Quezon City, was
dispatched by his Commanding
Officer to Dumalay Street in
Novaliches to respond to a
complaint that a person was
creating trouble there. Dionisio
arrived ath the plance and shot to
death a man. Pursuant to Section
7, Rule 112 of the Rules of Court,
the Office of the City Prosecutor
filed with the RTC of Quezon City
an Information charging Dionisio
with the crime of homicide. The
respondent Judge dismissed the
criminal complaint "for re-filing
with the Sandiganbayan" on the
ground that the Sandiganbayan,
and not the RTC, has jurisdiction
over the case for crimes committed
by public officer when penalty
prescribed by law for the offense is
higher than prision correccional.
Private prosecutor moved for a
reconsideration of the dismissal,
citing the opinion of the Secretary
of Justice that "crimes committed
by
PNP
members
are
not
cognizable by the Sandiganbayan"
because "they fall within the
exclusive
jurisdiction
of
the
regular
courts"
and
"the
Sandiganbayan is not a regular
court but a special court."

ISSUE:
Whether or not the Sandiganbayan
has jurisdiction over offenses
punishable by prision correctional
or higher commited by public
official.
HELD:
No. If the crime committed by the
public officer is not related to his
public function, then the RTC has
exclusive
jurisdiction
of
the
offense. There is no indication at
all that the trouble-maker was the
victim and that he was shot by the
private respondent in the course of
the latter's mission. However, it
may yet be true that the crime of
homicide charged therein was
committed
by
the
private
respondent in the course of his
public
mission,
which
fact,
however, was not alleged in the
information.
The Sandiganbayan shall exercise:
(a) Exclusive original jurisdiction
in all cases involving: (1) Violations
of Republic Act No. 3019, as
amended, otherwise known as the
Anti-Graft and Corrupt Practices
Act, Republic Act No. 1379, and
Chapter II, Section 2, Title VII of
the Revised Penal Code; (2) Other
offenses or felonies committed by
public officers and employees in
relation to their office, including
those employed in governmentowned or controlled corporations,
whether simple or complexed with
other crimes, where the penalty
prescribed by law is higher than
prision
correccional
or
imprisonment for six (6) years, or a

fine of P6,000.00: PROVIDED,


HOWEVER,
that
offenses
or
felonies
mentioned
in
this
paragraph where the penalty
prescribed by law does not exceed
prision
correccional
or
imprisonment for six (6) years or a
fine of P6,000.00 shall be tried by
the proper Regional Trial Court,
Metropolitan
Trial
Court,
Municipal
Trial
Court
and
Municipal Circuit Trial Court, (b)
Exclusive appellate jurisdiction: (1)
On
appeal,
from
the
final
judgments, resolutions or orders of
the Regional Trial Court in cases
originally decided by them in their
respective territorial jurisdiction.
(2) By petition for review, from the
final judgments, resolutions or
orders of the Regional Trial Courts
in the exercise of their appellate
jurisdiction over cases originally
decided by the Metropolitan Trial
Courts, Municipal Trial Courts and
Municipal Circuit Trial Courts, in
their respective jurisdiction. . . .
Undoubtedly
then,
the
Sandiganbayan is a regular court
and is thus included in the term
regular courts in Section 46 of R.A.

PEOPLE v. VANZUELA
G.R. NO. 178266, JULY 21, 2008
Topic: Venue/Jurisdiction
FACTS:
Veneranda is the wife of the late
Dionisio Paler, Sr. who is the
registered owner of a parcel of
irrigated
Riceland.
The
respondents allegedly failed to pay
the rentals since 1997. Initially,
Veneranda brought the matter
before the Department of Agrarian
Reform (DAR) Office in Surigaodel
Norte, but no amicable settlement
was reached by the parties. Thus,
Veneranda
filed
a
criminal
complaint for estafa against the
respondents.The RTC dismissed
the criminal case contending that
the instant case pertains to the
non-payment of rentals by the
accused
to
the
private
complainant, involving a lease of
an agricultural land by the former
from the latter. This being so, the
controversy in the case involves an
agrarian dispute which falls under
the primary and exclusive original
jurisdiction of the Department of
Agrarian
Reform
Adjudication
Board (DARAB).
ISSUE:
Whether or not RTC of Surigao
City has jurisdiction over the
charge for estafa even if it involves
agricultural tenants of the private
complainant.
HELD:
It is a well-entrenched doctrine
that the jurisdiction of a

tribunal
over
the
subject
matter of an action is conferred
by law. It is determined by the
material allegations of the
complaint or information and
the law at the time the action
was
commenced.
Lack
of
jurisdiction of the court over an
action or the subject matter of an
action, cannot be cured by the
silence, acquiescence, or even by
express consent of the parties.
Thus, the jurisdiction of the court
over the nature of the action and
the subject matter thereof cannot
be made to depend upon the
defenses set up in the court or
upon
a
motion
to
dismiss;
otherwise,
the
question
of
jurisdiction would depend almost
entirely on the defendant. Once
jurisdiction is vested, the same
is retained up to the end of the
litigation. The RTC likewise
acquired jurisdiction over the
persons of the respondents
because
they
voluntarily
submitted
to
the
RTCs
authority. Where the court has
jurisdiction over the subject
matter and over the person of
the accused, and the crime was
committed within its territorial
jurisdiction,
the
court
necessarily
exercises
jurisdiction over all issues that
the law requires the court to
resolve.
It is worth stressing that even the
jurisdiction over the prosecution of
criminal offenses in violation of RA
6657 per se is lodged with the
SACs and not with the DARAB.
While indeed, the parties admit
that there is an agricultural
tenancy relationship in this case,

and that under the circumstances,


Veneranda as landowner could
have simply filed a case before the
DARAB for collection of lease
rentals and/or dispossession of
respondents as tenants due to
their failure to pay said lease
rentals, there is no law which
prohibits
landowners
from
instituting a criminal case for
estafa, as defined and penalized
under Article 315 of the Revised
Penal Code, against their tenants.
Succinctly put, though the matter
before us apparently presents an
agrarian dispute, the RTC cannot
shirk from its duty to adjudicate on
the merits a criminal case initially
filed before it, based on the law
and evidence presented, in order
to determine whether an accused
is guilty beyond reasonable doubt
of the crime charged.

KWONG SING v. CITY OF


MANILA 41 PHIL., 103
TOPIC: Injuction on Criminal
Prosecution
Facts:
Kwong Sing, in his own behalf and
in behalf of all others having a
common or general interest in the
subject-matter of this action, filed
a complaint for a preliminary
injunction, prohibiting the city of
Manila from enforcing Ordinance
No.
532,
questioning
the
ordinances validity. The said
ordinance requires receipts in
duplicate in English and Spanish
duly signed showing the kind and
number of articles delivered by
laundries and dyeing and cleaning
establishments. Appellants claim
is that the ordinance savors of
class legislation; that it unjustly
discriminates between persons in
similar circumstances; and that it
constitutes
an
arbitrary
infringement of property rights.
There are, in the city of Manila,
more than 40 Chinese laundries.
The laundrymen and employees in
Chinese laundries do not, as a rule,
speak, read, and write English or
Spanish
Issue:
Whether or not the preliminary
injunction must be granted.
HELD:
It was perfectly proper for the trial
and appellate courts to determine
the validity of the municipal
ordinance on a complaint for an
injunction, since it was very

apparent that irreparable injury


was impending, that a municipality
of suits was threatened, and that
complainants had no other plain,
speedy, and adequate remedy. But
finding that the ordinance is
valid, the general rule to the
effect that an injunction will
not be granted to restrain a
criminal prosecution should be
followed.

ZALDIVIA v. REYES
G.R. No. 102342 July 3, 1992
Topic: Prescription
FACTS:
The petitioner is charged with
quarrying for commercial purposes
without a mayor's permit in
violation of an ordinance of the
Municipality of Rodriguez, in the
Province of Rizal. The offense was
allegedly committed on May 11,
1990. The referral-complaint of the
police was received by the Office
of the Provincial Prosecutor of
Rizal on May 30, 1990. The
corresponding information was
filed with the Municipal Trial
Court (MTC) of Rodriguez on
October 2, 1990. The petitioner
moved to quash the information on
the ground that the crime had
prescribed, but the motion was
denied. The petitioner argues that
the charge against her was
governed by Sections 1 and 2 of
the Rule on Summary Procedure
and
Act.
No.
3326.
The
prosecution, on the other hand,
contends that the prescriptive
period was suspended upon the
filing of the complaint against her
with the Office of the Provincial
Prosecutor. Agreeing with the
respondent judge, the Solicitor
General invokes Section 1, Rule
110 of the 1985 Rules on Criminal
Procedure, which provides that for
offenses not subject to the rule on
summary procedure in special
cases, the institution of criminal
action interrupts the period of
prescription
of
the
offense
charged.

ISSUE:
Whether or not the said offense
had already prescribed.
HELD:
Under Section 9 of the Rule on
Summary
Procedure,
"the
complaint or information shall be
filed directly in court without need
of a prior preliminary examination
or preliminary investigation." Both
parties agree that this provision
does not prevent the prosecutor
from conducting a preliminary
investigation if he wants to.
However, the case shall be deemed
commenced only when it is filed in
court,
whether
or
not
the
prosecution decides to conduct a
preliminary
investigation.
This
means that the running of the
prescriptive period shall be halted
on the date the case is actually
filed in court and not on any date
before that.
This
interpretation
is
in
consonance with the afore-quoted
Act No. 3326 which says that the
period of prescription shall be
suspended "when proceedings are
instituted against the guilty party."
The proceedings referred to in
Section 2 thereof are "judicial
proceedings," The Court feels that
if there be a conflict between the
Rule on Summary Procedure and
Section 1 of Rule 110 of the Rules
on Criminal Procedure, the former
should prevail as the special law.
And if there be a conflict between
Act. No. 3326 and Rule 110 of the
Rules on Criminal Procedure, the
latter must again yield because
this Court, in the exercise of its

rule-making power, is not allowed


to "diminish, increase or modify
substantive rights" under Article
VIII,
Section
5(5)
of
the
Constitution.
Prescription in
criminal cases is a substantive
right.
BENJAMIN K. GOROSPE, ET
AL.,
v.
MARIANO
B.
PEAFLORIDA, ET AL G.R. No.
11583, 19 JULY 1957
TOPIC: Injunction on Criminal
Prosecution

Peaflorida and Ladrido, through


counsel, filed a petition forprohibition with injunction with
the Court of First Instance of Iloilo
praying that the provincial fiscal
be enjoined from proceeding with
the investigation of the criminal
case until after the election
contest shall have been finally
determined, and the court issued a
preliminary
injunction
after
petitioner had filed a bond in the
amount of P1,000.

FACTS:

ISSUE:

Zulueta and Peaflorida were


candidates for the position of
Provincial Governor of Iloilo.
Penaflorida was declared elected
by the Board of Canvassers.
Zulueta filed with the Court of
First Instance of Iloilo a protest
contesting
the
election
of
Peaflorida on the grounds of
errors, irregularities, frauds and
corrupt
practices.
Similarly,
Ceferino de los Santos, Jr., a
defeated candidate for board
member, filed a criminal complaint
in said court against Peaflorida
and Ladrido charging the latter
with a violation of Section 49 of
the Revised Election Code relative
to
corrupt
practices.
This
complaint was dismissed on the
ground that, the violation charged
being a public offense, the same
can only be prosecuted by a
government prosecutor and not by
a private individual. And taking
cue of this suggestion, Zulueta
lodged a complaint with the fiscal
involving
the
same
charged
against Peaflorida and Ladrido.

Whether the injuction shall be


granted.
HELD:
As a general rule, an injunction
will not be granted to restrain a
criminal prosecution" (Kwong
Sing vs. City of Manila, 41 Phil.,
103). The reason is obvious. Public
interest requires that criminal acts
be immediately investigated and
prosecuted for the protection of
society. This is more so in
connection with a violation of the
Election Law. The only way to curb
fraud, terrorism and other corrupt
practices that are committed in the
elections is to demand their
immediate
investigation
and
prosecution. Only in this way can
we maintain a clean election and
secure the free expression of the
people's will at the polls. Statutes
or municipal ordinances, or to stay
the enforcement of orders of a
board or commission. This general
rule is based, in addition to other
considerations, on the principle
that equity is concerned only with

the protection of civil and property


rights,
and
is
intended
to
supplement, and not usurp, the
functions of the courts of the law,
and on the fact that the party has
an adequate at law by establishing
as a defense to the prosecution
that he did not commit the act
charged, or that the statute or
ordinance
on
which
the
prosecution is based is invalid,
and, in case of conviction, by
taking an appeal.
It is contended that this rule
admits of exceptions, one of them
being that "injunctions may be
issued to restrain vexatious and
oppressive criminal prosecutions"
(Yellowstone Kit vs. Wood, 43, S.
W. 1068, 19 Tex. C. App. 683).This
may be true, but we are not
prepared to hold that Zulueta
filed the criminal charge with
the intent to harass and
oppress
respondents,
there
being no clear findings to that
effect by the Court of Appeals.
The fact remains that the
injunction is predicated not on
that ground but on another
which we have found to be
legally untenable.

JESUS GUIAO v. ALBINO L.


FIGUEROA
G.R.
NO.
69863-65,
10
DECEMBER 1990
TOPIC: CRIMINAL ACTIONS,
WHEN
ENJOINED;
PRELIMINARY/FINAL
INJUNCTION
FACTS:
Petitioners were arrested by the
Northern Police District following
the forcible and violent dispersal
of a demonstration held in
sympathy with the jeepney strike
called
by
the
Alliance
of
Concerned Transport Organization
(ACTO). Thereafter, they were
charged with Illegal Assembly.
Brocka, et al. were subsequently
charged with Inciting to Sedition,
without prior notice to their
counsel.The original informations
filed recommended no bail. The
circumstances surrounding the
hasty filing of this second offense
are cited by Brocka, et al. Brocka,
et al. contend that respondents'
manifest
bad
faith
and/or
harassment are sufficient bases for
enjoining
their
criminal
prosecution, aside from the fact
that the second offense of inciting
to sedition is illegal, since it is
premised on one and the same act
of attending and participating in
the ACTO jeepney strike. They
maintain that while there may be a
complex crime from a single act
(Art. 48, RTC), the law does not
allow the splitting of a single act
into two offenses and filing two
informations therefor, further, that
they will be placed in double
jeopardy.

ISSUE:
Whether or not the prosecution of
the criminal cases for Inciting to
Sedition may lawfully be enjoined.
HELD:
Yes. Indeed, the general rule is
that criminal prosecution may not
be restrained or stayed by
injunction, preliminary or final.
There are however exceptions,
among which are:
a. To afford adequate protection to
the constitutional rights of the
accused;
b. When necessary for the orderly
administration of justice or to
avoid oppression or multiplicity of
actions;
c. When there is a pre-judicial
question which is sub judice;
d. When the acts of the officer are
without or in excess of authority;
e. Where the prosecution is under
an invalid law, ordinance or
regulation;
f. When double jeopardy is clearly
apparent;
g. Where the court has no
jurisdiction over the offense;
h. Where it is a case of persecution
rather than prosecution;
i.
Where
the
charges
are
manifestly false and motivated by
the lust for vengeance; and
j. When there is clearly no prima
facie case against the accused and
a motion to quash on that ground
has been denied.
k. Preliminary injunction has been
issued by the Supreme Court to
prevent the threatened unlawful
arrest of petitioners.

In the case at bar, Brocka, et al.


have cited the circumstances to
show that the criminal proceedings
had become a case of persecution,
having been undertaken by state
officials in bad faith.

JOSELITO v. NARCISO v. FLOR


MARIE STA. ROMANA-CRUZ
G.R. NO. 134504, 17 MARCH
2000
TOPIC: Prosecution Of Offense
(Offended Party)
FACTS:
Petitioner
was
charged
with
parricide which is punishable with
reclusion perpetua. Judge Pedro T.
Santiago of the Regional Trial
court (RTC) granted his Motion to
Post Bail. However, Court of
Appeals (CA) ruled otherwise. He
argued before the CA that he was
entitled to bail because the
evidence of his guilt was not
strong. He contended that the
prosecutor's conformity to his
Motion for Bail was tantamount to
a finding that the prosecution
evidence against him was not
strong. Private respondent then
know
questioned
the
order
granting bail at and he Court of
Appeals ruled, however, that there
was no basis for such finding,
since no hearing had been
conducted on the application for
bail -- summary or otherwise. Then
after
Petitioner
attacks
respondents legal standing to file
the Petition for Certiorari before
the appellate court, maintaining
that only the public prosecutor or
the solicitor general may challenge
the assailed Order granting the
bail.
ISSUE:
Whether or
respondent

not
has

the
the

private
legal

personality to intervene in the


present criminal case.
HELD:
Yes. In Paredes vs. Gopengco, 29
SCRA 688 (1969), this Court
ruled that the offended parties
in
criminal
cases
have
sufficient
interest
and
personality
as
person(s)
aggrieved to file the special
civil action of prohibition and
certiorari under Sections 1 and
2 of Rule 65 in line with the
underlying spirit of the liberal
construction of the Rules of
Court in order to promote their
object.
To rule otherwise would leave the
private respondent without any
recourse to rectify the public
injustice brought about by the trial
courts Order, leaving her with
only
the
standing
to
file
administrative
charges
for
ignorance of the law against the
judge and the prosecutor. A party
cannot be left without recourse to
address a substantive issue in law.
It should be remembered that
the crime charged against the
private respondent is parricide;
hence, the accused cannot be
regarded as an offended party.
That would be a contradiction in
terms and an absurdity in fact. Nor
can one expect the minor child to
think and to act for himself. Hence,
we rule that in view of the peculiar
circumstances of this case, the
sister of the deceased is a proper
party-litigant who is akin to the
offended party, she being a close
relative of the deceased. There is
no closer kin who may be expected

to take up the cudgels of justice


for the deceased.

BIENVENIDO A. EBARLE. HON.


JUDGE ASAALI S. ISNANI ET
AL.
G.R.
NO.
L-34162,
29
DECEMBER 1987
TOPIC:
PROSECUTION
OF
OFFENSE (Offended Party)
FACTS:
Petitioner Ebarle was then the
provincial governor of Zamboanga
and a candidate for re-election in
1971 local elections. The AntiGraft League of the Philippines
filed complaints with the city fiscal
against the petitioner for violations
of RA 3019 (Anti-Graft Law) and
Articles 171, 182,183, 213, and
318 of the Revised Penal Code.
The
challenge
the
petitioner
presents against the personality of
the Anti-Graft League of the
Philippines to bring suit is equally
without merit. That the Anti-Graft
League is not an "offended party"
within the meaning of Section 2,
Rule 110, of the Rules of Court
(now Section 3 of the 1985 Rules
on Criminal Procedure), cannot
abate the complaints in question.

(now Section 3 of the 1985 Rules


on Criminal Procedure). However,
A complaint for purposes of
preliminary investigation by
the fiscal need not be filed by
the "offended party." The rule
has been that, unless the offense
subject thereof is one that cannot
be prosecuted de oficio, the same
may be filed, for preliminary
investigation purposes, by any
competent person. The "complaint"
referred to in the Rule 110
contemplates one filed in court,
not with the fiscal. In that case,
the proceeding must be started by
the aggrieved party himself.
For as a general rule, a criminal
action is commenced by complaint
or information, both of which are
filed in court. In case of a
complaint, it must be filed by the
offended party; with respect to an
information, it is the fiscal who
files it. But a "complaint" filed
with the fiscal prior to a
judicial action may be filed by
any person.

ISSUE:
Whether or not the Anti-Graft
League has the standing to
commence
the
series
of
prosecutions
not
being
the
offended party.
HELD:
The Anti-Graft League of the
Philippines is not an "offended
party" within the meaning of Sec.
2, Rule 110, of the Rules of Court

ALBANO EMDI

RODRIGUEZ v. GADIANE ET AL
GR NO. 152903, JULY 17, 2006
FACTS:
The Court is called upon to resolve
the question of whether a private
offended party in a criminal

proceeding may file a special civil


action for certiorari under Rule 65,
assailing an interlocutory order,
without the conformity of the
public prosecutor.
Thomasita
Rodriguez
filed
complainant, a criminal case,
against Rolando Gadiane and
Ricardo Rafols, Jr., for violation of
Batas
PambansaBilang
22.
However, respondents filed a
motion to dismiss the petition on
the ground that the petition was
filed by the private complainant,
instead
of
the
government
prosecutor
representing
the
People of the Philippines in
criminal cases.
ISSUE:
Whether or not the aggrieved
person may file a special civil
action for certiorari.
HELD:
A special civil action for certiorari
may be filed by an aggrieved party
alleging grave abuse of discretion
amounting to excess or lack of
jurisdiction on the part of the trial
court.[8] In a long line of cases,
this Court construed the term
aggrieved parties to include the
State and the private offended
party or complainant.
It is well-settled that in criminal
cases where the offended party is
the State, the interest of the
private complainant or the private
offended party is limited to the
civil
liability.
Thus,
in
the
prosecution of the offense, the
complainant's role is limited to
that of a witness for the
prosecution. If a criminal case is

dismissed by the trial court or if


there is an acquittal, an appeal
therefrom on the criminal aspect
may be undertaken only by the
State
through
the
Solicitor
General. Only the Solicitor General
may represent the People of the
Philippines on appeal. The private
offended party or complainant may
not take such appeal.
However, the said offended party
or complainant may appeal the
civil aspect despite the acquittal of
the accused.
In a special civil action for
certiorari filed under Section 1,
Rule 65 of the Rules of Court
wherein it is alleged that the
trial court committed a grave
abuse of discretion amounting
to lack of jurisdiction or on
other jurisdictional grounds,
the rules state that the petition
may be filed by the person
aggrieved. In such case, the
aggrieved parties are the State
and the private offended party
or
complainant.
The
complainant has an interest in
the civil aspect of the case so
he may file such special civil
action questioning the decision
or action of the respondent
court on jurisdictional grounds.
In
so
doing,
complainant
should not bring the action in
the name of the People of the
Philippines. The action may be
prosecuted in name of said
complainant.
BAUTISTA v. FISCAL GR NO.
59830, JULY 31, 1984

FACTS:
Juan Bautista and Nenita Marquez
filed with the office of the City
Fiscal a complaint against Estrada,
Banigued, and F. Bautista for
estafa thru falsification of public
document. The assistant Fiscal
dismissed the case for lack of
prima facie evidence. Bautista did
not move for the reconsideration of
the fiscal's resolution; neither did
he appeal to the Ministry of
Justice. Instead, Mr. Bautista filed
a new complaint with the City
Court of Dagupan against the
same respondents, charging them
with the same offense. The City
Court found that an offense has
been
committed
and
the
respondents therein are probably
guilty thereof. Accordingly, a
warrant for their arrest was issued
and an order directing respondent
city fiscal to file the corresponding
information.
However,
the
respondent city fiscal, through
Assistant Fiscal Manaois, filed a
manifestation with the city court
that he will reinvestigate the case
in view of his prior resolution.
After reinvestigation, the city
Fiscal, filed a motion to dismiss the
case but was denied by the City
Court. Thereafter, the city court
again forwarded the records of the
case to respondent city fiscal for
the filing of the information. In
turn, respondent city fiscal filed a
manifestation informing the city
court of his inability to prosecute
the case because of his sincere and
honest belief that he has no prima
facie
case
to
warrant
the
prosecution of the accused. The
court
filed
a
petition
for

mandamus, ordering the city fiscal


to
file
the
corresponding
information for falsification of
public documents. The court of
appeals reversed the decision or
the trial court and dismissed the
petition for mandamus.
ISSUE:
Whether or not the appellate court
can direct the City Fiscal to file the
corresponding information and to
prosecute the case.
HELD:
The Supreme Court did not find
any cogent reason to set aside the
decision of the respondent Court
of Appeals
Section 4, Rule 110 of the Revised
Rules
of
Court,
specifically
provides that "all criminal action
either commenced by complaint or
by information shall be prosecuted
under the direction and control of
the fiscal.
Indeed, how can the prosecuting
fiscal secure the conviction of an
accused on evidence beyond
reasonable doubt when he himself
is not convinced that he has a
prima facie case against the
petitioners. The better procedure
would be to appeal the Fiscal's
decision to the Ministry of Justice
and/or ask for a special prosecutor.
Section 4, Rule 110 of the
Revised
Rules
of
Court,
specifically provides that "all
criminal
action
either
commenced by complaint or by
information shall be prosecuted
under the direction and control
of the fiscal.

Indeed,
how
can
the
prosecuting fiscal secure the
conviction of an accused on
evidence beyond reasonable
doubt when he himself is not
convinced that he has a prima
facie case against him.
But then, let's say that the
fiscal simply refuses to institute
a case against a respondent
even
if
the
evidence
is
sufficient to warrant the filing
of an information. As stated
above, the remedy is appeal to
the Ministry of Justice (then
Department of Justice) and, if
there
is
evidence,
administrative
complaint
against the prosecuting officer
for ignorance of the law,
neglect
of
duty,
partiality
and/or bribery.

certiorari and prohibition with


prayer for a preliminary writ of
injunction in the Court of Appeals.
CA, restrained Judge Mogul from
proceeding with the arraignment.
On the other hand, Undersecretary
of Justice resolved the petition for
review by reversing the resolution
of the Office of the Provincial
Fiscal and directed the Fiscal for
Immediate
dismissal
of
the
information. However, respondent
Judge denied the motion and set
the arraignment. Hence, the
accused filed a petition for
certiorari,
prohibition
and
mandamus with petition for the
issuance of preliminary writ of
prohibition
and/or
temporary
restraining order in the Court of
Appeals.

CRESPO v. MOGUL
53373, JUNE 30, 1987

Whether or not the trial court


acting on a motion to dismiss a
criminal
case
filed
by
the
Provincial Fiscal upon instructions
of the Secretary of Justice to whom
the case was elevated for review,
may refuse to grant the motion
and insist on the arraignment and
trial on the merits.

NO.

L-

FACTS:
The Assistant Fiscal with the
approval of the Provincial Fiscal
filed information for estafa against
Mario Fl. Crespo in the Circuit
Criminal Court of Lucena City.
When the case was set for
arraigment the accused filed a
motion to defer arraignment on
the ground that there was a
pending petition for review filed
with the Secretary of Justice of the
resolution of the Office of the
Provincial Fiscal for the filing of
the information. However, the
presiding judge, Leodegario L.
Mogul, denied the motion. So the
accused filed a petition for

ISSUE:

HELD:
Petition was dismissed.
The rule therefore is that once a
complaint or information is filed in
the Court any disposition of the
case as its dismissal or the
conviction or acquittal of the
accused rests in the sound
discretion of the court.
The Secretary of Justice who
reviewed the action of the fiscal

may be disregarded by the trial


court, the secretary of Justice, as
far as practicable, refrain from
entertaining a petition for review
or appeal from the action of the
fiscal, when the complaint or
information has already been filed
in the court
The rule therefore is that once
a complaint or information is
filed
in
the
Court
any
disposition of the case as its
dismissal or the conviction or
acquittal of the accused rests
in the sound discretion of the
court.
The Secretary of Justice who
reviewed the action of the fiscal
may be disregarded by the trial
court, the secretary of Justice,
as far as practicable, refrain
from entertaining a petition for
review or appeal from the
action of the fiscal, when the
complaint or information has
already been filed in the court.
PADILLA v. CA L-39999, MAY
31, 1984
FACTS:
The Fiscal accused the petitioners
of the crime of grave coercion.
Confederating
and
mutually
helping one another, and acting
without any authority of law, did
then and there willfully, unlawfully,
and feloniously, by means of
threats, force and violence prevent
Antonio Vergara and his family to
close their stall. In committing the
offense,
the
accused
took
advantage of their public positions:

Roy Padilla, being the incumbent


municipal mayor, and the rest of
the accused being policemen,
except Ricardo Celestino who is a
civilian, all of Jose Panganiban,
Camarines Norte, and that it was
committed
with
evident
premeditation. The trial court finds
the
accused
guilty
beyond
reasonable doubt of the crime of
grave coercion. Hence an appeal
to the Court of Appeals, which
modified the ruling of the trial
court, and decided in favor of the
petitioners. Stating that they were
acquitted on the ground of
reasonable doubt but ordered
them to pay for the actual
damages. Still not content with the
CAs decision, the petitioners filed
a motion for reconsideration
contending that the acquittal of
the defendants-appellants as to
criminal liability results in the
extinction of their civil liability.
ISSUE:
Whether or not the respondent
court committed a reversible error
in requiring the petitioners to pay
civil indemnity to the complainants
after acquitting them from the
criminal charge
HELD:
The Supreme Court AFFIRMED
the decision of the respondent
Court of Appeals and dismiss the
petition for lack of merit.
The extinction of the penal action
does not carry with it that of the
civil,
unless
the
extinction
proceeds from a declaration in a
final judgment that the fact from
which the civil might arise did not

exist. (Rule 111, Sec. 3 (c), Rev.


Rules of Court; Laperal v. Aliza, 51
OG.R. 1311, People v. Velez, 44
OG. 1811)
Article 29 clearly and expressly
provides is a remedy for the
plaintiff in case the defendant has
been acquitted in a criminal
prosecution on the ground that his
guilt has not been proved beyond
reasonable
doubt.
It
merely
emphasizes that a civil action for
damages is not precluded by an
acquittal for the same criminal act
or omission.
The extinction of the penal
action does not carry with it
that of the civil, unless the
extinction proceeds from a
declaration in a final judgment
that the fact from which the
civil might arise did not exist.
(Rule 111, Sec. 3 (c), Rev. Rules
of Court; Laperal v. Aliza, 51
OG.R. 1311, People v. Velez, 44
OG. 1811)
A separate civil action may be
warranted
where
additional
facts have to be established or
more
evidence
must
be
adduced or where the criminal
case has been fully terminated
and a separate complaint would
be just as efficacious or even
more expedient than a timely
remand to the trial court where
the criminal action was decided
for further hearings on the civil
aspects of the case. The
offended party may, of course,
choose to file a separate action.
These do not exist in this case.
Considering
moreover
the
delays suffered by the case in
the trial, appellate, and review

stages, it would be unjust to


the complainants in this case to
require at this time a separate
civil action to be filed.
PEOPLE v. CAMBA L-36471,
NOVEMBER 19, 1980
FACTS:
Camba was found guilty of murder
qualified by treachery with assault
upon an agent of a person in
authority was sentenced to death
and to indemnify the heirs of
Gongora. However, upon the
observation of the court, the crime
committed is more of robbery with
homicide, for the reason that the
victim Pat. Gongora was killed
while responding to the call for
help of a victim of snatching.
Hencem the appellant claims that
his conviction for murder should
be set aside.
ISSUE:
Whether or not the court erred in
convicting the accused of the
offense charged despite the fact
that what was proven was a
different crime.
HELD:
It should be born in mind in this
connection that the label or
caption in the information in
respect of the crime committed is
not controlling- what matters are
the material allegations in the
information.
The trial court found the appellant
guilty of murder qualified by

treachery but without any other


modifying circumstance. The crime
was compounded, according to
the trial court, with assault upon
an agent of a person in authority.
In this light the appropriate
penalty, pursuant to Art. 48 of the
Revised Penal Code, is death.
If it be held as we do that the
crime committed was robbery with
homicide, the result will be the
same
for
the
aggravating
circumstances of treachery and
contempt of public authority will
have to be assigned.
The death penalty, however, cannot
be imposed on the appellant who
was only 20 years old when he
committed the crime for lack of the
necessary votes.
The judgment of the court a quo is
modified by reducing the sentence
on the appellant to reclusion
perpetua and consolidating the
damages
in
the
amount
of
P30,000.00.
Since
the
charge
in
the
information is robbery with
homicide and the evidence is to
the same effect, the mistake in
designating the crime both in
the information and in the
judgment is not fatal.
The second assignment of error
questions the factual findings
of the trial court as to the
appellants identity and as the
person who caused the death of
Patrolman Reynaldo Gongora.
In support of the above, the
prosecution had two witnesses,
namely:
Rodrigo
Eser
and
Eleazar
Pangilinon.
To
contradict them, the appellant

presented witnesses to support


his alibi.
BUHAT v. COURT OF APPEALS
265 SCRA 701
TOPIC: AMENDMENT OF THE
INFORMATION
FACTS:
On March 25, 1993, an information
for HOMICIDE was filed in the
RTC against petitioner Danny
Buhat, John Doe and Richard Doe.
The information alleged that on
October 16, 1992, petitioner
Danilo Buhat, armed with a knife,
unlawfully attacked and killed one
Ramon George Yu while the said
two unknown assailants held his
arms, using superior strength,
inflicting x x x mortal wounds
which were x x x the direct x x x
cause of his death.
Even before petitioner could be
arraigned, the prosecution moved
for
the
deferment
of
the
arraignment on the ground that
the private complainant in the
case, one Betty Yu, moved for the
reconsideration of the resolution of
the City Prosecutor which ordered
the filing of the aforementioned
information
for
homicide.
Petitioner however, invoking his
right to a speedy trial, opposed the
motion. Thus, petitioner was
arraigned on June 9, 1993 and,
since petitioner pleaded not guilty,
trial ensued.
On February 3, 1994, then
Secretary of Justice Franklin M.
Drilon, finding Betty Yus appeal
meritorious, ordered the City
Prosecutor of Roxas City to amend

the information by upgrading the


offense charged to MURDER and
implead therein additional accused
Herminia Altavas, Osmea Altavas
and Renato Buhat.
On March 10, 1994, the Assistant
City Prosecutor filed a motion for
leave to amend information. The
amendment as proposed was
opposed by the petitioner.
In an order, dated June 2, 1994,
the RTC denied the motion for
leave to amend information. The
denial was premised on (1) an
invocation of the trial courts
discretion in disregarding the
opinion of the Secretary of Justice
as allegedly held in Crespo v.
Mogul and (2) a conclusion
reached by the trial court that the
resolution
of
the
inquest
prosecutor is more persuasive than
that of the Secretary of Justice, the
former having actually conducted
the
preliminary
investigation
where he was able to observe the
demeanor of those he investigated
The Solicitor General promptly
elevated the matter to the Court of
Appeals. He filed a petition for
certiorari assailing the aforecited
order denying the motion for leave
to amend information. Finding the
proposed amendment as nonprejudicial to petitioners rights,
respondent court granted the
petition for certiorari.
ISSUES:
1.
Whether
or
not
the
additional allegation of conspiracy
is a substantial amendment
2.
Whether
or
not
the
upgrading of the crime charged
from homicide to the more serious
offense of murder is such a

substantial amendment that it is


disallowed if made after the
accused had pleaded "not guilty"
to the crime of homicide
HELD:
1. No. The additional allegation of
conspiracy is only a formal
amendment, hence petitioner's
participation as principal is not
affected by such amendment.
Petitioner undoubtedly is charged
as a principal in the killing of
Ramon George Yu whom he is
alleged to have stabbed while two
unknown persons held the victim's
arms. The addition of the phrase,
"conspiring, confederating and
helping one another" does not
change the nature of petitioner's
participation as principal in the
killing.
Whether under the original or the
amended information, petitioner
would have to defend himself as
the People makes a case against
him and secures for public
protection the punishment of
petitioner for stabbing to death,
using superior strength, a fellow
citizen in whose health and safety
society as a whole is interested.
Petitioner, thus, has no tenable
basis to decry the amendment in
question
2. The Court ruled in the negative.
In the case of Dimalibot v.
Salcedo,the Court ruled that the
amendment of the information so
as to change the crime charged
from homicide to murder, may be
made "even if it may result in
altering the nature of the charge
so long as it can be done without

prejudice to the rights of the


accused."
Petitioner in the case at bench
maintains that having already
pleaded "not guilty" to the crime of
homicide, the amendment of the
crime charged in the information
from homicide to murder is a
substantial amendment prejudicial
to his right to be informed of the
nature of the accusation against
him. He utterly fails to dispute,
however,
that
the
original
information
did
allege
that
petitioner stabbed his victim
"using superior strength." This
particular allegation qualifies a
killing to murder, regardless of
how such a killing is technically
designated in the information filed
by the public prosecutor, as
provided for in Article 248 of the
RPC.
The
amendment
of
the
Information
by
merely
changing
its
express
designation from homicide to
murder
is
only
a
formal
amendment and no prejudice
can be made against the rights
of the accused.
TEHANKEE JR. VS MADAYAG
207 SCRA 134
TOPIC: AMENDMENT OF THE
INFORMATION
FACTS:
Petitioner was originally charged
on July 19, 1991 in an information
for the crime of frustrated murder
of one Maureen Navarro Hultman.

After the prosecution had rested


its case, petitioner was allowed to
file a motion for leave to file a
demurrer to evidence. However,
before the said motion could be
filed, Maureen Navarro Hultman
died.
Consequently, private prosecutor
Rogelio A. Vinluan filed an
omnibus motion for leave of court
to file an amended information and
to
admit
said
amended
information.
The
amended
information, filed on October 31,
1991, reads:
That on or about the 13th day of
July, 1991, in the Municipality of
Makati, Metro Manila, Philippines
and within the jurisdiction of this
Honorable Court, the said Claudio
Teehankee, Jr. y. Javier, armed with
a handgun, with intent to kill and
evident premeditation and by
means of treachery, did then and
there willfully, unlawfully and
feloniously attack, assault and
shoot with the said handgun
Maureen Navarro Hultman who
was hit in the head, thereby
inflicting mortal wounds which
directly caused the death of said
Maureen Hultman.
Petitioner filed an opposition
thereto, as well as a rejoinder to
the reply of the prosecution. The
trial court issued the questioned
order admitting the amended
information.
Petitioner avers that the additional
allegation
in
the
amended
information
constitutes
a
substantial amendment since it
involves a change in the nature of
the offense charged, that is, from
frustrated
to
consummated
murder.

Petitioner then postulates that


since the amended information for
murder
charges
an
entirely
different offense, it is essential
that
another
preliminary
investigation on the new charge be
conducted
before
the
new
information can be admitted.
At the scheduled arraignment on
November 26, 1991, petitioner
refused to be arraigned on the
amended information for lack of a
preliminary investigation thereon.
By
reason
of
such
refusal,
respondent judge ordered that a
plea of "not guilty" be entered for
petitioner.
ISSUE:
Whether or not an amended
information involving a substantial
amendment, without preliminary
investigation, after the prosecution
has
rested
on
the
original
information, may legally and
validly be admitted
HELD:
In the present case, there is an
identity of offenses charged in
both the original and the amended
information. What is involved here
is not a variance in the nature of
different offenses charged, but
only a change in the stage of
execution of the same offense from
frustrated
to
consummated
murder. This is being the case, we
hold that an amendment of the
original information will suffice
and, consequent thereto, the filing
of the amended information for
murder is proper.

Petitioner would insist, however,


that the additional allegation on
the fact of death of the victim
Maureen
Navarro
Hultman
constitutes
a
substantial
amendment which may no longer
be allowed after a plea has been
entered.
The
proposition
is
erroneous and untenable.
Section 14 of Rule 110 provides
that an amendment, either of form
or substance, may be made at any
time before the accused enters a
plea to the charge and, thereafter,
as to all
After arraignment and during the
trial, amendments are allowed, but
only as to matters of form and
provided that no prejudice is
caused to the rights of the
accused.
An objective appraisal of the
amended
information
for
murder filed against herein
petitioner will readily show that
the nature of the offense
originally charged was not
actually changed. Instead, an
additional allegation, that is,
the supervening fact of the
death of the victim was merely
supplied to aid the trial court
in determining the proper
penalty for the crime.
The
amended
information
could not conceivably have
come as a surprise to petitioner
for the simple and obvious
reason
that
it
charges
essentially the same offense as
that charged under the original
information. Furthermore, as
we have heretofore held, if the
crime originally charged is
related to the amended charge

such that an inquiry into one


would elicit substantially the
same facts that an inquiry into
the other would reveal, a new
preliminary investigation is not
necessary.
JALANDONI v. ENDAYA
SCRA 261
TOPIC: JURISIDCTION

55

FACTS:
Petitioner instituted a criminal
complaint for libel in the Municipal
Court of the Municipality of
Batangas presided over by the
respondent Judge. The accused
named therein was Serafin D.
Cruz. There was the corresponding
preliminary examination of the
witnesses for the complainant with
respondent Judge finding that
there was reasonable ground to
believe that such offense was
committed by the person named.
After respondent Cruz posted the
corresponding bail bond for his
provisional liberty, the respondent
Judge set the case for hearing on
the merits on July 13, 1964, at 2:30
o'clock in the afternoon. When that
time came, complainant, now
petitioner,
through
counsel
manifested in open court that
under Article 360 of the Revised
Penal Code, respondent Judge was
devoid of jurisdiction to do so.
There was, as noted, a negative
response. After hearing arguments
on such motion for desistance
including memoranda submitted
by both sides, respondent Judge
issued
an
order
denying
petitioner's verbal motion to have

Criminal Case No. 801 elevated to


the Court of First Instance of
Batangas. With a motion for
reconsideration meeting the same
fate, this petition for prohibition
was filed.
ISSUE:
Whether or not respondent Judges
insistence to try a prosecution for
libel, instead of having it elevated
to the proper court of first instance
is correct
HELD:
There is no need to make mention
against that it is a court of first
instance
that
is
specifically
designated to try a libel case.
Article 360 of the Revised Penal
Code so provides. Its language is
categorical; its meaning is free
from doubt. This is one of those
statutory provisions that leaves no
room for interpretation. All that is
required is application. 4 What the
law ordains must then be followed.
It is as simple as that. It did not
appear to be so to respondent
Judge. He would go ahead. He
therefore did invite a suit of this
character bent as he was on
treading
grounds
where
his
presence was, to put it at its
mildest, unwelcome. He must be
restrained.
There is no need to make
mention against that it is a
court of first instance that is
specifically designated to try a
libel case. Article 360 of the
Revised Penal Code so provides.
Its language is categorical; its

meaning is free from doubt.


This is one of those statutory
provisions that leaves no room
for interpretation. All that is
required is application.
PEOPLE VS PINEDA 20 SCRA
748
TOPIC:
CONSOLIDATION
OF
OFFENSES
IN
ONE
INFORMATION
FACTS:
Respondents
Tomas
Narbasa,
Tambac Alindo and Rufino Borres
stand indicted before the Court of
First Instance of Lanao del Norte,
as principals, in five (5) separate
cases for murder.
On July 29, 1965, spouses Teofilo
Mendoza and Valeriana Bontilao
de Mendoza, along with their
children were attacked by private
respondents.
Teofilo
and
the
children were shot dead, leaving
only Valeriana alive but wounded.
Two of the three defendants in the
five criminal cases heretofore
listed Tomas Narbasa and
Tambak Alindo moved for a
consolidation thereof "into one (1)
criminal case." Their plea is that
"said cases arose out of the same
incident and motivated by one
impulse."
Giving the nod to defendants'
claim, respondent Judge directed
the City Fiscal to unify all the five
criminal cases, and to file one
single information in Case 1246.
He also ordered that the other four
cases, Nos. 1247, 1248, 1249 and
1250 "be dropped from the
docket."

The City Fiscal balked at the


foregoing
order,
sought
reconsideration thereof, upon the
ground that "more than one gun
was used, more than one shot was
fired and more than one victim was
killed." The defense opposed.
Respondent Judge denied the
motion to reconsider. He took the
position that the acts complained
of "stemmed out of a series of
continuing acts on the part of the
accused, not by different and
separate sets of shots, moved by
one impulse and should therefore
be treated as one crime though the
series of shots killed more than
one victim;" and that only one
information for multiple murder
should be filed, to obviate the
necessity of trying five cases
instead of one."
ISSUE:
Whether or not respondent Judge
was correct in averring that the
five cases be consolidated, and
that only one information should
be filed by the City Fiscal
HELD:
The Court ruled in the negative.
Deeply rooted is the doctrine that
when various victims expire from
separate
shots,
such
acts
constitute separate and distinct
crimes. In People v. Gatbunton, the
spouses Mariano Sebastian and
Maxima Capule who were
asleep were killed by one burst
of machine gun fire; and then, by a
second burst of machine gun fire,
two of the couple's children also
asleep were killed. The accused,

Tomas Gatbunton, was found guilty


by the trial court of quadruple
murder. On appeal, this Court
declared that "appellant must be
declared guilty of four murders."
The respondent Judge reasons out
in his order of May 31, 1966 that
consolidation of the five cases into
one would have the salutary effect
of obviating the necessity of trying
five cases instead of one. To save
time,
indeed,
is
laudable.
Nonetheless, the statute confers
upon the trial judge the power to
try these cases jointly, such that
the fear entertained by respondent
Judge could easily be remedied.
Upon the facts and the law, the
Court holds that the City Fiscal of
Iligan City correctly presented the
five separate informations four
for murder and one for frustrated
murder.
The consolidation of the five
cases into one would have the
salutary effect of obviating the
necessity of trying five cases
instead of one. To save time,
indeed,
is
laudable.
Nonetheless,
the
statute
confers upon the trial judge the
power to try these cases jointly,
such that the fear entertained
by respondent Judge could
easily be remedied.
PEOPLE v. DEVARAS 228 SCRA
482
TOPIC:
QUALIFYING
CIRCUMSTANCES
SHALL
BE
ALLEGED IN THE INFORMATION
FACTS:

A pedicab driver and his passenger


were attacked without provocation
by two men who hacked them to
death and later threw their bodies
over the bridge with the help of
another person. Investigation that
same
night
disclosed
the
participation
of
the
herein
appellants, who were subsequently
accused
of
murder
in
two
informations alleging that they
committed
the
offenses
in
conspiracy with each other and
with treachery and abuse of
superior strength.
The principal witness for the
prosecution was Raul Animos, who
claimed to have witnessed the
killing of the two victims. He said
that on July 10, 1990, at about 7
o'clock in the evening, the three
appellants were drinking with him
in the house of Devaras and that
thereafter they joined him on his
tour of duty as bantay-bayan. They
had been making the rounds in the
town for about four hours when,
while at the Daguitan bridge, they
saw
a
zigzagging
pedicab
approach. When the pedicab was
halfway
across
the
bridge,
Blademir
Devaras,
who
was
carrying a long bolo, suddenly
attacked
Efren
Verzosa,
the
pedicab driver. Efren fell from his
seat
but
Blademir
continued
hacking him with the bolo, hitting
him in the head and neck. At abut
the same time, Ronilo Caisek, who
also carrying a long bolo, attacked
Felix Verzosa, the passenger, who
tried to parry the blows with his
arms as he got out of the vehicle.
He fell, staggered and ran but was
overtaken
by
Ronilo,
who
continued striking the helpless old

man in the head, neck, chest and


shoulders.
Pablo Devaras did not participate
in the brutal slaying but later
helped Blademir throw Efren's
body over the bridge into the river
below. Ronilo himself was ordered
to help throw the body of Felix
and,
although
he
initially
hesitated, had to comply in the end
because he was threatened with
death if he refused to obey. The
body of Felix was found the
following morning under the
bridge. The body of Efren was
found about three hours later near
the seashore. The autopsy revealed
that Felix had sustained twenty
wounds and nine wounds had been
inflicted in Efren.
All
three
accused
denied
participation in the killings. After
assessing the evidence of parties,
the trial court decided in favor of
the prosecution and convicted
Blademir Devaras as principal and
Pablo Devaras as accessory in the
murder of Efren Verzosa; and
Ronilo Caisek for the murder of
Felix Verzosa.
ISSUES:
1.
Whether or not the RTC
erred in convicting them of murder
instead of homicide
2.
Whether or not the RTC
erred in convicting appellant Pablo
Devaras as an accessory
HELD:
1.
The first error assigned by
the
appellants
is
untenable
because the evidence of record
clearly shows that Blademir and

Ronilo suddenly attacked their


unarmed
victims
with
bolos,
thereby insuring the commission of
the
offense
without
risk
themselves
arising
from
the
defense the victims might make.
The killings would have been
homicide
only
without
the
attendance of treachery, which is
one
of
the
qualifying
circumstances
mentioned
in
Article 248 of the Revised Penal
Code.
We agree that there was no
conspiracy between the appellants
to justify their common conviction
for both murders. There is no
evidence that Blademir and Ronilo
had earlier come to an agreement
to kill the Verzosas; on the
contrary, it would appear that they
had
acted
on
impulse,
independently of any common
plan. The trial court was correct in
finding Blademir guilty of the
murder only of Efren Verzosa and
Ronilo guilty of the murder only of
Felix Verzosa instead of holding
them equally liable for both
murders.
2.
The second assigned error
must also be dismissed. The
determination of the persons to be
prosecuted on the basis of the
evidence
against
them
rests
primarily with the prosecutor, who
is
vested
with
quasi-judicial
discretion in the discharge of this
function. We have held that, as an
exception, the prosecutor can be
compelled by mandamus if he
abuses this discretion and refuses
to include a person as a coaccused against whom there
appears to be at least prima facie.
However, this extraordinary writ is

available only if the petitioner


shows that he has first exhausted
all remedies in the ordinary course
of law, such as a motion filed with
the trial court for the indictment of
the person or persons excluded by
the prosecutor. It does not appear
that such a motion was filed by the
appellants in the case at bar.
An accessory is defined as one
who, having knowledge of the
commission of the crime, and
without
having
participated
therein, either as principal or an
accomplice, takes part subsequent
to its commission by concealing or
destroying the body of the crime,
or the effects or instruments
thereof in order to prevent its
discovery. For having assisted in
throwing Efren's dead body into
the river, Pablo Devaras was
correctly
held
guilty
as an
accessory.
Where
no
particulars
are
known as to the manner in
which the aggression was made
or how the act which resulted
in the death of the victim began
and developed, it can in no way
be
established
from
mere
suppositions that the killing
was perpetrated by treachery.
Thus, it cannot be considered
where the lone witness did not
see the commencement of the
assault.
Accordingly, Serrano
could
only
be
liable
for
Homicide, and not Murder,
since
no
qualifying
circumstance
attended
the
killing of Teodoro Bisnar.

PEOPLE OF THE PHILIPPINES


v. NARCISO NAZARENO, ET.
AL. G.R. NO. 103964, AUGUST
1, 1996
TOPIC:
PROSECUTION
OF
OFFENSES
FACTS:
This is an appeal from the
decisionof the Regional Trial Court
of Makati (Branch 136), finding
accused-appellants
Narciso
Nazareno and Ramil Regala guilty
of murder for the killing of Romulo
"Molet" Bunye II in Muntinlupa,
Metro Manila. Two others, accused
with them, Manuel Laureaga and
Orlando Hular, were acquitted.
On December 14, 1988, between
8:00 a.m. and 9:00 a.m., Bunye
took a stainless tricycle, which
was
driven
by
Fernando
Hernandez. Unknown to Bunye
was that two men were waiting
outside his house and that the two
hailed another tricycle in order to
follow him.Bunye alighted at the
corner of T. Molina and Mendiola
Streets in Alabang, Muntinlupa
and crossed to the left side of the
street. Shortly after, the tricycle,
driven by Rogelio de Limos,
arrived and stopped in front of
Hernandez's tricycle. One of the
men jumped out of the tricycle and
shot Bunye at the back of the
head. When Bunye fell face down,
the assailant fired another shot at
Bunye's head. Then, the other man
approached Bunye and shot him
also in the head. The autopsy
report on the victim showed that
he died of gunshot wounds in the
head.

On December 28, 1988, Ramil


Regala,
Narciso
Nazareno,
Orlando
Hular
and
Manuel
Laureaga were arrested. Regala
and Nazareno were put in a police
line-up. They were identified and
pointed to as the assailants by the
tricycle drivers. Regala executed
affidavits, admitting participation
in the slaying of Bunye and
pointing to Nazareno and a certain
Rey Taling as his co-conspirators.
He claimed that they had been
hired by Hular to kill the victim
and told that they would be paid
by Laureaga. His affidavits were
corroborated by Hular who stated
that it was Laureaga who wanted
Bunye
killed,
apparently
in
connection with Bunye's job as
administrator of the public market
in Alabang.
However,
Regala
and
Hular
recanted. Regala claimed that he
had been tortured. On the other
hand, Hular claimed that, although
he was not tortured, he admitted
to the crime and signed the
affidavit because he was afraid he
would also be tortured. Nazareno
also claimed to have been tortured
to admit to the crime but refused
to sign any written statement. The
trial court ruled the confessions of
Regala
and
Hular
to
be
inadmissible. However, it held
Regala and Nazareno guilty on the
basis of their positive identification
by Hernandez and de Limos during
the police line-up and their
testimony in court.
ISSUE:

Whether or not the constitutional


right to due process of the
accused-appellants was violated.
HELD:
No, there was no violation of due
process in the prosecution and
subsequent conviction of Nazareno
and
Regala.Accused-appellants
claim that their arrests without
warrant were illegal and justify the
nullification of the proceedings of
the trial court. The warrantless
arrest
of
accused-appellant
Nazareno was upheld by this Court
in 1990 in a petition for habeas
corpus. It appears that, on January
9, 1989, Nazareno filed a motion
for bail. As the trial court denied
his motion, a petition for habeas
corpus was filed on his behalf with
this Court. It was alleged that
Nazareno's arrest was illegal
because it was made without
warrant fourteen days after the
killing of Bunye. This Court
dismissed the petition. He filed a
motion for reconsideration which
the Court also denied on the
ground that the warrantless arrest
was in accordance with Rule 113,
5(b) of the Revised Rules of
Criminal Procedure. The question
which Nazareno raises has thus
been settled long ago in a final
decision of this Court.
Furthermore,
Nazareno
and
Regala waived objections based on
the alleged irregularity of their
arrest, considering that they
pleaded not guilty to the charges
against them and participated in
the trial. Any defect in their arrest
must be deemed cured when they
voluntarily
submitted
to
the

jurisdiction of the court. For the


legality of an arrest affects only
the jurisdiction of the court over
the person of the accused.
Consequently, if objections based
on this ground are waived, the fact
that the arrest was illegal is not a
sufficient cause for setting aside
an otherwise valid
judgment
rendered after a trial, free from
error. The technicality cannot
render
the
subsequent
proceedings void and deprive the
State of its right to convict the
guilty when all the facts on record
point to the culpability of accused.
The legality of an arrest affects
only the jurisdiction of the
court over the person of the
accused.
HONORATO GALVEZ, ET. AL. v.
COURT OF APPEALS ET. AL.
G.R. NO. 114046, OCTOBER 24,
1994
TOPIC:
PROSECUTION
OF
OFFENSES; AMENDMENT OR
SUBSTITUTION OF COMPLAINT
OR INFORMATION
FACTS:
On November 12, 1993, petitioners
Honorato Galvez, the incumbent
Mayor of San Ildefonso, Bulacan,
and one Godofredo Diego were
charged
in
three
separate
informations with homicide and
two counts of frustrated homicide.
Both
accused
posted
their
respective cash bail bonds and
were subsequently released from
detention. On November 15, 1993,
Bulacan
Provincial
Prosecutor

Liberato L. Reyes filed a Motion to


Defer
Arraignment
and
Subsequent Proceedings to enable
him "to review the evidence on
record and determine once more
the proper crimes chargeable
against the accused," which was
granted
by
Judge
Villajuan.
Respondent Prosecutor Dennis M.
Villa-Ignacio
was
designated
Acting Provincial Prosecutor of
Bulacan and was instructed to
conduct a re-investigation of the
aforesaid criminal cases filed
against herein petitioners.
By virtue of a Manifestation with
Exparte
Motion
filed
by
respondent
prosecutor,
the
proceedings were again ordered
suspended by Judge Villajuan until
after the prosecution's request for
change of venue shall have been
resolved by the Supreme Court,
and the preliminary investigation
being conducted by the former
shall have been terminated. It
appears that on December 2, 1993,
private complainants had filed with
the SC a Petition for Change of
Venue, purportedly to safeguard
the lives of the victims and their
witnesses, and to prevent a
miscarriage of justice.
On December 15, 1993, before
petitioners could be arraigned,
respondent prosecutor filed an Ex
parte
Motion
to
Withdraw
Informations. This motion was
granted by Judge Villajuan and the
cases were considered withdrawn
from the docket of the court. On
the same day, Prosecutor VillaIgnacio filed four new informations
against herein petitioners for
murder, two counts of frustrated
murder,
and
violation
of

Presidential Decree No. 1866 for


illegal possession of firearms
which were subsequently raffled to
the sala of Judge Victoria Pornillos
of Branch 10, Regional Trial Court
of Malolos, Bulacan. No bail
having been recommended for the
crime of murder, Judge Pornillos
ordered the arrest of herein
petitioners.
On December 27, 1993, the
scheduled
arraignment
before
Judge Pornillos were reset due to
the
absence
of
respondent
prosecutor.
On
even
date,
petitioners filed before Judge
Villajuan
a
Motion
for
Reconsideration of his order of
December 15, 1993 which granted
the motion to withdraw the
original informations. Thereafter, a
Motion
to
Quash
the
new
informations for lack of jurisdiction
was filed by petitioners before
Judge Pornillos. At the court
session set for the arraignment of
petitioners, Judge Pornillos issued
an order denying the motion to
quash and, at the same time,
directed that a plea of not guilty be
entered for petitioners when the
latter refused to enter their plea.
In the meantime, and prior to the
arraignment of herein petitioners
before Judge Pornillos, an order
was issued by Judge Villajuan
granting
the
motion
for
reconsideration
filed
by
petitioners.
However,
the
arraignment was suspended and,
in the meanwhile, petitioners
filed a petition for certiorari ,
prohibition and mandamus with
respondent Court
of
Appeals,
assailing the order dated January
24, 1994 issued by Judge Pornillos

which denied petitioners' motion


to
quash.
Respondent
court
dismissed the petition.
ISSUE:
Whether the ex parte motion to
withdraw the original informations
is null and void on the ground that
(a) there was no notice and
hearing as required by Sections 4,
5 and 6, Rule 15 of the Rules of
Court; and (b) the appropriate
remedy which should have been
adopted by the prosecution was to
amend
the
informations
by
charging the proper offenses
pursuant to Section 14 of Rule
110.
HELD:
It is petitioners' submission that
the prosecution's failure to serve
them a copy of the motion to
withdraw the original informations
and to set said motion for hearing
constitutes a violation of their
right to be informed of the
proceedings
against
them.
Petitioners advance the theory that
respondent prosecutor should have
amended the original informations
instead of withdrawing the same
and
filing
new
ones.
They
postulate that the principle of nolle
prosequi does not apply in this
case since the withdrawal or
dismissal of an information is
addressed solely to the sound and
judicious discretion of the court
which has the option to grant or
deny it and the prosecution cannot
impose its opinion on the court.
It is further stressed that in case
there is a need to change the

nature of the offense charged, that


is, from homicide to murder, by
adding the qualifying circumstance
of treachery, the only legal and
proper remedy is through the filing
of the corresponding amended
information;
and
that
the
withdrawal of an information is
allowed only where the new
information involves a different
offense which does not include or
is not included in the offense
originally charged.
In actuality, the real grievance of
herein accused is not the dismissal
of the original three informations
but the filing of four new
informations,
three
of
which
charge graver offenses and the
fourth, an additional offense. Had
these new informations not been
filed, there would obviously have
been no cause for the instant
petition. Petitioners' contention
that the dismissal of the original
informations and the consequent
filing of the new ones substantially
affected their right to bail is too
strained and tenuous an argument.
They would want to ignore the fact
that had the original informations
been amended so as to charge the
capital offense of murder, they still
stood to likewise be deprived of
their right to bail once it was
shown that the evidence of guilt is
strong. Petitioners could not be
better
off
with
amended
informations
than
with
the
subsequent ones. It really made no
difference considering that where
a capital offense is charged and
the evidence of guilt is strong, bail
becomes a matter of discretion
under either an amended or a new
information.

The rule is now well settled


that once a complaint or
information is filed in court any
disposition of the case, whether
as to its dismissal or the
conviction or the acquittal of
the accused, rests in the sound
discretion
of
the
court.
Although
the
prosecutor
retains
the
direction
and
control of the prosecution of
criminal cases even when the
case is already in court, he
cannot impose his opinion upon
the tribunal. For while it is true
that the prosecutor has the
quasi-judicial
discretion
to
determine whether or not a
criminal case should be filed in
court, once the case had
already been brought therein
any disposition the prosecutor
may deem proper thereafter
should be addressed to the
court for its consideration and
approval. The only qualification
is that the action of the court
must not impair the substantial
rights of the accused or the
right of the People to due
process of law.
FELIX A. VELASQUEZ v. HON.
UNDERSECRETARY
OF
JUSTICE, HON. ARTEMIO G.
TUQUERO, ET. AL G.R. NO.
88442, FEBRUARY 15, 1990
TOPIC:
PROSECUTION
OF
OFFENSES;
CONTROL
OF
PROSECUTION
FACTS:

Respondent Edgardo Avila was a


Cash and Business Development
Consultant
of
the
Techtrade
Management
International
Corporation, authorized to followup business transactions, including
loan applications submitted to the
company.
On September 29, 1986, Avila
informed the company that he had
a borrower (whom he did not
identify) for P200,000 with interest
of 3%/month for a 30-day term
from September 29 to October 29,
1988. This was approved by the
company which issued to him a
pay-to-cash check for P194,000
after deducting the 3% interest of
6,000. Instead of returning the
borrowed amount on due date or
giving a satisfactory explanation
for the supposed borrower's failure
to pay the loan despite written
demands, Avila resigned from the
company on December 17, 1986
promising that: "... I shall set aside
the P200,000 upon its subsequent
collection xxx to answer for the
P100,000
portion
of
Tony's
P700,000 loan to you; please treat
the P100,000 balance, less my
unpaid professional fee and gas
expenses from November 16 to
December 15, 1986, as my
separation
and
compulsory
benefit".
On December 23, 1986, petitioner
Felix A. Velasquez, as Executive
Vice-President/Managing Director
of Techtrade, filed a complaint for
estafa against Avila in the Manila
City Fiscal's Office. Assistant
Fiscal Romulo Lopez dismissed the
complaint. However, upon review
by the Chief, Investigation Division
of the City Fiscal's Office, the

latter set aside Fiscal Lopez'


resolution and ordered the filing of
an information for estafa against
Avila in the RTC.
Avila
twice
sought
a
reconsideration of that resolution,
but both motions were denied by
the
City
Fiscal.
Before
arraignment, Avila filed on June
29, 1987 in the Department of
Justice a petition for review which
the petitioner opposed. Justice
Undersecretary Silvestre Bello III
denied the petition for review. A
motion for reconsideration of the
denial did not prosper.
On October 14, 1988, Avila filed a
second motion for reconsideration
which the Undersecretary of
Justice,
Honorable
Artemio
Tuquero granted on January 4,
1989. He directed the City
Fiscalto
conduct
a
reinvestigation of this case to
afford respondent to properly
present evidence that he was duly
authorized to pay the subject
creditors and for complainant to
rebut the same with controverting
evidence, and thereafter to resolve
the case anew on the basis of all
the
evidence
adduced.
The
complainant filed a motion for
reconsideration of that resolution
but it was denied.
ISSUE:
Whether or not the Secretary of
Justice, the State Prosecutor, or
the Fiscal interfere with the
judges disposition of the case.
HELD:
No,neither
the
Secretary
of
Justice, the State Prosecutor, nor

the Fiscal may interfere with the


judge's disposition of the case,
much less impose upon the court
their opinion regarding the guilt or
innocence of the accused, for the
court is the sole judge of that.
Once the information is filed in
court, the court acquires complete
jurisdiction over it. A motion for
reinvestigation should, after the
court had acquired jurisdiction
over the case, be addressed to the
trial judge and to him alone.
Although the fiscal retains the
direction and control of the
prosecution of criminal cases even
while the case is already in Court,
he cannot impose his opinion on
the trial court. The Court is the
best and sole judge on what to do
with the case before it. The
determination of the case is within
its exclusive jurisdiction and
competence. A motion to dismiss
the case filed by the fiscal should
be addressed to the Court who has
the option to grant or deny the
same. It does not matter if this is
done
before
or
after
the
arraignment of the accused or that
the motion was filed after a
reinvestigation
or
upon
instructions of the Secretary of
Justice who reviewed the records
of the investigation.
Once
a
complaint
or
information is filed in Court,
any disposition of the case as
its dismissal or the conviction
or acquittal of the accused
rests in the sound discretion of
the Court.

JOCEL ESPINOLA
Sta. Rosa Mining Co. vs Zabala
G.R. No. L-44723, August 31,
1987

Topic: Prosecution
(Rule 110)

of

Offenses

Facts:
Petitioner filed a complaint for
attempted theft of materials (scrap
iron)
forming
part
of
the
installations on its mining property
in Camarines Norte against private
respondents with the Office of the
Provincial Fiscal. A preliminary
investigation was then conducted
and thereafter, a resolution was
issued recommending that an
information for Attempted Theft be
filed against private respondents
on a finding of prima facie case
which resolution was approved by
the Provincial Fiscal. Private
respondents
sought
reconsideration of the resolution
but the same was denied. An
information was filed with the
Court for the same crime. Private
respondents then requested the
Secretary of Justice for a review of
the resolution of the Provincial
Fiscal. The Secretary reversed the
findings of the Secretary and
directed said prosecuting officer to
immediately move for the dismissal
of the criminal case. The motion

to dismiss that was filed by the


Fiscal and the subsequent one filed
by the private respondents were
denied, prompting the private
respondents to file a petition for
mandamus.

Issue:
Whether or not the petition for
mandamus should be granted.

Held:
No. The Supreme Court held that
once a complaint or information is
filed in Court any disposition of the
case as its dismissal or the
conviction or acquittal of the
accused rests in the sound
discretion of the Court. Although
the fiscal retains the direction and
control of the prosecution of
criminal cases even while the case
is already in Court, he cannot
impose his opinion on the trial
court. The Court is the best and
sole judge on what to do with the
case before it. The determination
of the case is within its exclusive
jurisdiction and competence. A
motion to dismiss the case filed by
the fiscal should be addressed to
the Court who has the option to
grant or deny the same. It does not
matter if this is done before or
after the arraignment of the
accused or that the motion was
filed after a reinvestigation or
upon instructions of the Secretary
of Justice who reviewed the
records of the investigation.

In order therefore to avoid such a


situation whereby the opinion of
the Secretary of Justice who
reviewed the action of the fiscal
may be disregarded by the trial
court, the Secretary of Justice
should, as far as practicable,
refrain from entertaining a petition
for review or appeal from the
action of the fiscal, when the
complaint or information has
already been filed in Court. The
matter should be left entirely for
the determination of the Court.
Baares II et al. vs Balising
G.R. No. 132624, March 13,
2000
Topic: Prosecution
(Rule 110)

of

Offenses

Facts:
Petitioners were the accused in
sixteen (16) cases for Estafa filed
by the private respondents.
After
the
petitioners
were
arraigned and entered their plea of
not guilty, they filed a Motion to
Dismiss the aforementioned cases
on the ground that the filing of the
same was premature, in view of
the failure of the parties to
undergo conciliation proceedings
before the Lupong Tagapamayapa.
The municipal trial court issued an
Order, denying petitioners Motion
to Dismiss on the ground that they
failed to seasonably invoke the
non-referral of the cases to
the Lupong
Tagapamayapa or Pangkat
ng
Tagapagkasundo. It added that

such failure to invoke non-referral


of the case to the Lupon amounted
to a waiver by petitioners of the
right to use the said ground as
basis for dismissing the cases.
The municipal trial court issued an
Order dismissing the sixteen
criminal cases against petitioners
without prejudice, pursuant to
Section 18 of the 1991 Revised
Rule on Summary Procedure.
More than two months later,
private
respondents
through
counsel, filed a Motion to Revive
the
criminal cases against
petitioners,
stating
that
the
requirement
of
referral
to
the Lupon for
conciliation
had
already been complied with.

The law grants an aggrieved party


a period of fifteen (15) days from
his receipt of the court's decision
or order disposing of the action or
proceeding to appeal or move to
reconsider the same. After the
lapse of the fifteen-day period, an
order becomes final and executory
and is beyond the power or
jurisdiction of the court which
rendered it to further amend or
revoke. The doctrine of finality of
judgments and orders also applies
in criminal cases.

Issue:
Whether
or
not
an
order
dismissing a case or action without
prejudice may attain finality if not
appealed within the reglementary
period.

Topic: Prosecution
(Rule 110)

Held:
Yes. The Court held that a "final
order" issued by a court has been
defined as one which disposes of
the subject matter in its entirety or
terminates a particular proceeding
or action, leaving nothing else to
be done but to enforce by
execution
what
has
been
determined by the court. As
distinguished
therefrom,
an
"interlocutory order" is one which
does not dispose of a case
completely, but leaves something
more to be adjudicated upon.
An order dismissing a case without
prejudice is a final order if no
motion for reconsideration or
appeal therefrom is timely filed.

Roberts vs CA
G.R. No. 11390, March 5, 1996

of

Offenses

Facts:
Petitioner cites DOJ Department
Circular
No.
70,
specifically
paragraph 2 of Section 9 thereof,
which provides that the appellant
and the trial prosecutor shall see
to it that, pending resolution of the
appeal, the proceedings in court
are held in abeyance. Somehow,
petitioner is of the opinion that the
suspension of proceedings in
court, as provided in the said
circular, includes the suspension of
the implementation of warrants of
arrest issued by the court.
Issue:
Whether
or
not
a
pending
resolution of a petition for review
filed with the Secretary of Justice
concerning a finding of probable
cause
will
suspend
the

proceedings in the trial court,


including the implementation of a
warrant of arrest.
Held:
No. The Court held that there is
nothing in Crespo v. Mogul, 151
SCRA 462 (1987), which bars the
DOJ from taking cognizance of an
appeal, by way of a petition for
review, by an accused in a criminal
case from an unfavorable ruling of
the investigating prosecutor.
Section 4, Rule 112 of the Rules of
Court recognizes the authority of
the Secretary of Justice to reverse
the resolution of the provincial or
city prosecutor or chief state
prosecutor upon petition by a
proper party.
The real and ultimate test of the
independence and integrity of the
court is not the filing of motions to
suspend proceedings and to defer
arraignment but the filing of a
motion to dismiss or to withdraw
the information on the basis of a
resolution of the petition for
review reversing the resolution of
the investigating prosecutor.

Ledesma vs. CA
G.R. No. 113216, September 5,
1997
Topic: Prosecution
(Rule 110)
Facts:

of

Offenses

A complaint for libel was filed by


Dr. Juan F. Torres, Jr. against the
petitioner. Finding sufficient legal
and factual basis, the Quezon City
Prosecutor's
Office
filed
an
Information
for
libel
against
petitioner with the Regional Trial
Court of Quezon City. A petition for
review of the resolution of the
Assistant City Prosecutor was filed
by
petitioner
before
the
Department
of
Justice.
The
Department of Justice gave due
course to the petition and directed
the Quezon City prosecutor to
move for deferment of further
proceedings and to elevate the
entire records of the case. 5
Accordingly, a Motion to Defer,
Arraignment was filed by a
Prosecutor before the court a quo.
The trial court granted the motion
and
deferred
petitioner's
arraignment
until
the
final
termination of the petition for
review. Without the consent or
approval of the trial prosecutor,
private
complainant,
through
counsel, filed a Motion to Lift the
Order and to set the Case for
Arraignment/Trial.
Issue:
Whether or not respondent Judge
committed
grave
abuse
of
discretion, amounting to lack of
jurisdiction, when he denied the
Motion to Withdraw Information.
Held:
The Supreme Court held that when
confronted with a motion to
withdraw an information on the
ground of lack of probable cause
based on a resolution of the
secretary of justice, the bounden

duty of the trial court is to make


an independent assessment of the
merits of such motion. Having
acquired jurisdiction over the case,
the trial court is not bound by such
resolution but is required to
evaluate it before proceeding
further with the trial. While the
secretarys ruling is persuasive, it
is not binding on courts. A trial
court, however, commits reversible
error or even grave abuse of
discretion if it refuses/neglects to
evaluate such recommendation
and simply insists on proceeding
with the trial on the mere pretext
of
having
already
acquired
jurisdiction over the criminal
action.
Solar Entertainment, Inc. vs.
Hon. How
G.R. No. 140863, August 22,
2000
Topic: Prosecution
(Rule 110)

of

Offenses

Facts:
The City Prosecutor of Paraaque
filed an Information for Estafa
against private respondent Ma. Fe
Barreiro
based
on
the
complaint filed by Solar Team
Entertainment, Inc.
Before the scheduled arraignment
of private respondent could take
place, respondent court issued an
Order, resetting the arraignment
of private respondent on the
ground that private respondent
had filed an appeal with the
Department of Justice. Respondent
court further
deferred
the

arraignment of private respondent


until such time that the appeal
with the Secretary of Justice was
resolved.
Issue:
Whether or not the trial court can
indefinitely
suspend
the
arraignment of the accused until
the petition for review with the
Secretary of Justice has been
resolved.
Held:
Yes. The Court held that after the
filing of the information, the court
is in complete control of the case
and any disposition therein is
subject to its sound discretion. The
decision to suspend arraignment to
await the resolution of an appeal
with the Secretary of Justice is an
exercise of such discretion. A court
can defer to the authority of the
prosecution arm to resolve, once
and for all, the issue of whether or
not sufficient ground existed to file
the information.
A. AVERMENTS
COMPLAINT

IN

THE

People vs. Banihit


G.R. No. 132045, August 25,
2000
Topic: Averments in Complaint or
Information
Facts:
Accused-appellant was convicted
of rape and was sentenced to
death.

Accused-appellant and his niece,


Glaiza, rode a motorcycle and went
to a grassy lot in a place called Del
Pilar. There, accused-appellant
undressed Glaiza and made her lie
down on a big tire. Accusedappellant took off his own clothes,
went on top of Glaiza and had
sexual intercourse with her. Glaiza
was pinned between the big tire
and accused-appellant. She felt
pain but was afraid to cry. After
that, Glaiza put on her clothes, and
she and accused-appellant walked
home.
Issue:
Whether or not the accusedappellant was properly informed of
the nature and cause of accusation
against him.
Held:
Yes. The Court held that the real
nature of the criminal charge is
determined not from the caption or
preamble of the information nor
from the specification of the
provision of the law alleged to
have been violated, they being
conclusions of law which in no way
affect the legal aspects of the
information, but from the actual
recital of facts as alleged in the
body of the information.
People vs. Quiachon
G.R. No. 170236, August 31,
2006
Topic: Averments in Complaint or
Information
Facts:
The appellant Roberto Quiachon
had
sexual
intercourse
with

Rowena Quiachon, his daughter, 8years old, and a deaf-mute minor,


against her will and consent. The
Regional Trial Court of Pasig City
found the appellant guilty of the
crime of qualified rape and
sentenced him to suffer the
maximum penalty of death which
was reduced to reclusion perpetua
by virtue of R.A. 9346.
The defense presented the lone
testimony of appellant Roberto
Quiachon and interposed a defense
of denial.
Issue:
Whether or not denial would
suffice as a defense against the
charge.
Held:
No. The Court held that a mere
denial, just like alibi, constitutes a
self-serving
negative
evidence
which cannot be accorded greater
evidentiary
weight
than
the
declaration of credible witnesses
who testify on affirmative matters.
Viewed against
the
damning
evidence
of
the
prosecution,
appellants simple denial of the
charge
against
him
must
necessarily fail. The defense of
denial is inherently weak. A mere
denial, just like alibi, constitutes a
self-serving
negative
evidence
which cannot be accorded greater
evidentiary
weight
than
the
declaration of credible witnesses
who testify on affirmative matters.
People vs. Barcena
G.R. No. 168737, February 16,
2006

Topic: Averments in Complaint or


Information

G.R. No. 130599-600, April 12,


1999

Facts:
Accused-appellant was convicted
by the RTC for the crime of
qualified rape. He raped the 15year old daughter of his commonlaw spouse.
The victim was alone in their
house when appellant suddenly
embraced her from behind and
dragged her towards the bedroom
despite her vigorous attempts to
free herself from his clutches.
Appellant
repeatedly
slapped
Estrella, forcibly stripped her of
her clothing, mounted her and
then inserted his penis into her
vagina. Estrella could not tell how
long the appellant stayed on top of
her but after a while, he put on his
clothes and ordered her to do the
same. He warned her not to tell
anybody about the assault or he
would kill her. Four days later, she
revealed the harrowing experience
to her cousin who accompanied
her to the barangay captain.
Appellant raised the defenses of
denial
and
alibi.

Topic: Averments in Complaint or


Information

Issue:
Whether or not minority and
relationship should be alleged.
Held:
The Court held that Minority and
relationship
constitute
special
qualifying circumstances, which, in
accordance with the settled rule,
must be alleged in the information
and proven during trial.
People vs. Manggasin

Facts:
Accused-appellant Juan Manggasin
was found guilty of two (2) counts
of rape by the RTC.
The complainant was coming back
from the river to get her brothers
clothes to do some laundry.
When she arrived in their house,
she was allured/hypnotized by
accused-appellant,
which
rendered her unconscious. When
she woke up, she felt some pain in
her
vagina,
which
was
bleeding. When
she
asked accused-appellant what had
happened to her, he warned her
not to tell anyone what had been
done to her, otherwise he would
kill her and her mother. She also
claimed that accused-appellant
then dragged her and embraced
her so tightly that both of them fell
down. He then touched her private
parts and inserted his penis into
her vagina. After he was through,
accused-appellant
told her
she
would be killed if she told anyone
about the incident.
Complainant claimed that she had
been sexually assaulted several
times.
Issue:
Whether or not the factual findings
of the trial court is binding upon
the appellate court.
Held:
Yes. The Court held that the
evaluation of the testimonies of the

witnesses by the trial court is


binding upon the appellate court in
the absence of a clear showing
that it was reached arbitrarily or
that the trial court had plainly
overlooked
certain
facts
of
substance or value which, if
considered, might affect the result
of
the
case;
Guidelines
in
Evaluation of Evidence in Rape
Cases.

investigation does not impair the


validity of the information or
otherwise
render
the
same
defective and neither does it affect
the jurisdiction of the court over
the case or constitute a ground for
quashing the information.
Ilagan vs. CA
G.R. No. 110617, De. 29, 1994
Topic: Duplicity of Offenses

B. DUPLICITY OF OFFENSES
Sanchez vs. Demetriou
G.R. Nos.111771-77, November
9, 1993
Topic: Duplicity of Offenses
Facts:
Informations were filed against sev
eral
persons, including Mayor Antonio
L.Sanchez of Calauan, Laguna, in c
onnection with the rape-slay
of Mary Eileen Sarmenta and the
killing of Allan Gomez. Upon
invitation
to
appear
for
investigation,
Sanchez
was
identified as one of the principals
of the crime charged and was then
placed on arrest status after
which a warrant of arrest was
issued against him.
Issue:
Whether or not the absence of a
Preliminary Investigation impair
the validity of the information or
the jurisdiction of the court

Facts:
Eight (8) informations were filed
with the Regional Trial Court of
Kalookan City, charging petitioners
Geruncio H. Ilagan, Claro Pion
and
Rosendo
Pion
as
coconspirators in the crime of estafa.
They
moved
to
quash
the
informations in Criminal Cases
Nos. C-40483 to C-40489 on the
ground of duplicity of offenses
charged therein. The same was
dismissed by the trial court.
Issue:
Whether or not double jeopardy
would set in.
Held:
No. The Court held that where the
ground of double jeopardy was not
raised in the motion to quash
before the trial court, then it is
unpardonably absurd to claim that
its
non-application
would
constitute
grave
abuse
of
discretion.
C. CONTINUING CRIMES

Held:
No. The
absence

Court held that the


of
a
preliminary

Santiago vs Garchitorena

G.R. No.109266, December 2,


1993

G.R. No.
28, 1975

Topic: Continuing Crimes

Topic: Continuing Crimes

Facts:
Petitioner filed a petition for
certiorari and prohibition to enjoin
the
Sandiganbayan
from
proceeding with Criminal Case No.
16698 on the ground that said case
was intended solely to harass her
as she was then a presidential
candidate.
Public
prosecutors
filed
32
Amended Informations
against
petitioner, after manifesting to the
Sandiganbayan that they would
only file one amended information.

Facts:
Private respondent Benjamin Lu
Hayco was a former employee of
petitioner company in its optical
supply business. One hundred
twenty-four (124) complaints of
estafa under Article 315, par. 1-b
of the Revised Penal Code were
filed against him by the petitioner
company with the Office of the City
Fiscal
of
Manila.
After
the
procedural
preliminary
investigation, the Office of the City
Fiscal filed seventy-five (75) cases
of
estafa
against
private
respondent before the City Court
of Manila.

Issue:
Whether or not the concept of a
continuing crime is applicable in
crimes penalized under special
laws.
Held:
The Court held that, technically,
there was only one crime that was
committed in petitioner's case, and
hence, there should only be one
information to be file against her.
The concept of Delito Continuado
is applicable to crimes penalized
under special laws. The concept of
delito continuado, although an
outcrop of the Spanish Penal Code,
has been applied to crimes
penalized under special laws.
Where only one single criminal act
was committed on the same period
of time, several informations
should be consolidated into only
one.
Gamboa vs. CA

L-41054,

November

Issue:
Whether
or
not
the
basic
accusations contained in the
seventy-five
(75)
informations
against
private
respondent
constitute but a single crime of
estafa.
Held:
The Court held that Delito
continuado or continuous crime
is a single crime consisting of a
series acts arising from a single
criminal resolution or intent not
susceptible of division. For Cuello
Calon, when the actor, there being
unity of purpose and of right
violated, commits diverse acts,
each of which although of a
delitual
character,
merely
constitutes a partial execution of a
single particular delict, such

concurrence or delictual acts is


called a delito continuado.
In order that it may exist, there
should be plurality of acts
performed separately during a
period of time; unity of penal
provision
infringed
upon
or
violated and unity of criminal
intent and purpose, which means
that two or more violations of the
same penal provision are united in
one and the same intent leading to
the perpetration of the same
criminal purpose or aim.
People vs Hubilo
G.R. No. 101741, March 23,
1993
Topic: Continuing Crimes
Facts:
Appellant Adly Hubilo was found
guilty by the trial court of
"multiple (triple) murder" and of
frustrated murder. Hermogenia
Cacayurin, Cesario Gamiz and
Ferdinand Gamiz were riding on a
tricycle driven by Rogelio Antonio,
proceeding to thir residence. As
they approached a cemetery,
gunfire greeted them and driver
Rogelio, being hit fell off the
tricycle. Ferdinand Gamiz who had
been seated behind the tricycle
driver jumped off the tricycle and
ran. He was met or confronted by
an armed man whose face was
covered by handkerchief. The

masked
gunman
fired
at
Ferdinand. As the gunman fired,
the cover on his face fell off.
Ferdinand recognized the assailant
as Adly Hubilo, also a resident of
the same locality and known to
him since he reached the age of
reason. Ferdinand pleaded for his
life. Hubilo, however, squeezed the
trigger again but this time the gun
did not fire Ferdinand seized his
chance to escape death and ran
away. Hubilo tried to pursue
Ferdinand. Ferdinand was able to
elude him, and saw Hubilo
approach fire at the tricycle many
times.
Issue:
Whether or not there is a complex
crime.
Held:
No.
The
Court
held
that
Irregularity
in
preliminary
investigation is waived by posting
bail
and
submitting
to
arraignment.
Where several shots hurt several
victims, the crime is not a complex
crime.
When
the
evidence
presented by the prosecution did
not show that a single shot had
slain three (3) different persons,
appellant is properly held liable for
three (3) separate murders.

D. COMPLEX CRIME
PEOPLE V. VICTOR, 181 SCRA 818
Facts:
When Victor went to Boljoon to
visit his wife, Guneda met him at
the market place and proposed to
him a plan to rob the residence of
an American named Myles Castle
and the cottage of Charles Turner,
an
American
Peace
Corps
Volunteer
assigned
in
the
Municipality of Boljoon The two
agreed to execute their plan on
Oct. 16, 1984. As planned, Victor
went to Boljoon in the afternoon of
October 16, 1984, together with
Montebon, who brought with him a
.38 caliber homemade revolver.
They proceeded to the store of a
certain Josefina Romero along
McKinley Street, where Victor
used to leave his things intended
for his wife everytime he came to
Boljoon. They passed the time
there drinking "tuba" and eating
bread while they were conversing
in subdued voices. At past 6:00
o'clock that afternoon,Victor and
Montebon left and proceeded
towards a bridge where they were
subsequently joined by Guneda,
who led them to the house of
Myles Castle along Rodriguez
Street. As they entered the gate of
Castle's house, a maid went out to
get the clothes which were being
hanged
to
dry.
Forthwith,
Montebon held her and said:

"Don't move this is a hold up!"


Frightened the maid instinctively
shouted, causing the three to
scamper away and return to the
bridge where they stayed for a
while.From
there,
the
trio
proceeded to rob Charles Turner in
his rented cottage located at
Lusapon Beach in the outskirts of
Boljoon Poblacion. However, upon
entering Charles Turner's cottage,
Montebon shot him at the back of
his head upon instruction of
Guneda, who wanted the American
killed because the latter knew him.
After killing Turner, the trio
ransacked the cottage of Turner's
personal belongings, and returned
to the culvert near the bridge
where they left the things which
they found to be unimportant. At
about 6:30 o'clock in the morning
of October 17, 1984, a report was
made to the Police Force of Boljoon
that Charles Turner was seen
lifeless in his cottage. Forthwith,
Patrolman
Marcos
Florida,
Archimedes Villanueva and Romulo
Medida proceeded to the cottage
of the Peace Corps. Volunteer
where he was found dead while
sitting on a chair with his head
stooping towards a table. That
same day at about 2:00 o'clock in
the afternoon, a six-man PC-INP
Team proceeded to Cebu City to
track down the suspect. Upon
apprehension
Victor
verbally

admitted being one of those who


killed the American Peace Corps
Volunteer in Boljoon, Cebu. Victor
even voluntarily turned over to the
PC-INP team a size 40 corduroy
jacket owned by the victim. The
team also recovered an empty .38
caliber revolver shell inside the
house of Victor who explained that
said empty shell was left by his coaccused Roberto Montebon while
they were drinking liquor inside
the house immediately after the
incident From there,Victor led the
team to the house of Montebon in
Inawayan
Pardo,
Cebu
City.
Montebon was lying down inside
his house when the PC-INP team
arrived at about 11:00 o'clock in
the morning that same day. Then
and there, Captain Barias placed
Montebon under arrest. The team
also retrieved the items taken by
the suspects from the victim's
cottage in Boljoon. Regional Trial
Court found: a) Both Roberto
Montebon and Ceferino Guneda
guilty beyond doubt of the crime of
Robbery with Homicide. They were
both sentenced to suffer Reclusion
Perpetua,
and
the
accessory
penalties provided for by law; and
b) Roger Victor guilty beyond
doubt of the offense of simple
Robbery, with two mitigating
circumstances of plea of guilty to
Robbery and testifying as to true
facts in favor of the State. It is

argued that Guneda cannot be


convicted of the special complex
crime of Robbery with Homicide as
the information filed against him
failed to allege that the Homicide
was committed "by reason of or on
the occasion of the robbery," and
that neither may one infer from
that charge alone that the alleged.
Homicide was done for purposes of
committing the alleged Robbery,
thus violating Guneda's right to be
informed of the nature and cause
of the accusation against him
Issue:
WON Guneda can be convicted of
the special complex crime of
Robbery with Homicide as the
information filed against him failed
to allege that the Homicide was
committed "by reason of or on the
occasion of the robbery,"
Held:
Failure to sate in the information
that the killing of the victim was
committed by reason of or on
occasion of robbery, does not bar
conviction of accused of the
special complex crime of robbery
with homicide. Information will
reveal that it has sufficiently
alleged
the
proper
offense
committed which is that of
Robbery with Homicide. The
precise language of the statute
need not be used in alleging the
commission of the crime as long as
in charging the commission of a

complex offense like that of


Robbery
with
Homicide,
the
Information alleges each element
of the component offenses with the
same precision that would be
necessary if they were made the
subject of a separate prosecution.
Although the phrase "by reason or
on occasion of the robbery", as
provided for by the Revised Penal
Code, was not literally used in the
recital of facts alleging the
commission of the crime of
Robbery
with
Homicide,
the
Information, as filed, sufficiently
and
distinctly
alleges
the
commission of the two crimes of
"Robbery" and "Homicide" and
adequately informs the accused of
the offense charged.

PEOPLE V. FELOTEO, 290 SCRA


627
Facts:
SONNY SOTTO, and his friends,
ARNEL ABELEDA and JOHNNY
ABREA, were walking along the
highway in Barangay Bintuan,
Coron, Province of Palawan. They
had a few drinks earlier that day
and were on their way home to
Sitio Nagbaril. Abrea walked
ahead of the group, about thirteen
meters away from Sotto, followed
by Abeleda. They were in a lively
mood as Abeleda playfully walked
backwards, facing Sotto.
The
accused,
WILFREDO
FELOTEO,
appeared
on
the
opposite side of the road and
walked past Abrea and Abeleda.
He was armed with an armalite
rifle.
Abeleda
and
Abrea
recognized the accused, their
barriomate, as the moon was
shining brightly. They did not pay
much attention to the accused as
Abeleda was playing "habulan"
with Sotto. Without uttering a
word, the accused aimed the
armalite at Sotto and pressed its
trigger. Sotto was hit above the left
chest and fell on the ground, face
down.
Abeleda
and
Abrea
scampered away to find help, while
the accused fled from the crime
scene. Ten (10) minutes later,
Abeleda and Abrea, accompanied
by Barangay Tanod Tito Abrina and

a certain Inyong Adion, returned to


the locus criminis. They found
Sotto dead.
After trial, the accused was found
guilty
as
charged.
He
was
sentenced to suffer the penalties of
reclusion perpetua, for murder,
and imprisonment of twenty (20)
years, for illegal possession of
firearm. He was further ordered to
pay the heirs of Sotto the amount
of
fifty
thousand
pesos
(P50,000.00), as civil indemnity.
The appellant now contends that
the
trial
court
erred
in
appreciating
the
qualifying
circumstance of treachery as
attending the commission of the
crime alleged and in holding
accused-appellant guilty of murder
in the killing of Sonny Sotto.
Issue:
WON the trial court erred in
appreciating
the
qualifying
circumstance of treachery as
attending the commission of the
crime alleged and in holding
accused-appellant guilty of murder
in the killing of Sonny Sotto.
Held:
The Information was sufficient and
properly alleged that the crime
was committed with aggravating
circumstance of treachery. The
Information contained all the
necessary details needed. Under
par. 16, Article 14 of the Revised
Penal
Code,
the
qualifying

circumstance of treachery
is
present when the offender employs
means, methods, or forms in the
execution of the crime which tend
directly and especially to insure its
execution without risk to himself
arising from any defensive or
retaliatory act which the victim
might make. The settled rule is
that treachery can exist even if the
attack is frontal if it is sudden and
unexpected, giving the victim no
opportunity to repel it or defend
himself. What is decisive is that the
execution of the attack, without
the slightest provocation from a
victim who is unarmed, made it
impossible for the victim to defend
himself or to retaliate.
In the case at bar, treachery is
present for there was a sudden
attack against the unarmed Sotto.
When Sotto and his friends
encountered appellant on the road,
they were in a "jovial mood" as
they just came from a drinking
spree. Although they saw appellant
carrying an armalite, they did not
suspect anything untoward to
happen. However, without any
provocation, appellant shot Sotto.
The fact that the attack was frontal
cannot negate treachery. The
shooting was unexpected. There is
no showing that the alleged
warning given by appellant to
Sotto afforded the latter sufficient
time to defend himself. Indeed,

Sotto could not defend himself as


he was unarmed and a bit drunk
as observed by the appellant
himself, the victim was walking in
a zigzag manner. There was no
way for Sotto to avoid the armalite
bullets.

PEOPLE V. VALDEZ, 304 SCRA


611
Facts:
William Montano (16 years old),
Randy Tibule (17 years old), Jean
Maria
Garcia,
Willie
Acosta,
Sandra Montano and Ramon
Garcia, Jr. were at the house of
Randy
Tibule
in
Manaoag,
Pangasinan. They were discussing
how to go to the wedding party of
Jean Marie's cousin in Sitio
Cabaoangan. After discussion, they
rode in the tricycle driven by
Ramon
Garcia
going
to
Cabaoangan. Behind Garcia were
Tibule and Willie. Jean was seated
inside the side car with Sandra and
William Montano. After making a
turn along the barangay road
leading to Sitio Cabaoangan they
met appellant Rolando Valdez and
his companions who were armed
with guns. The tricycle's headlight
flashed on their faces. Without
warning, they pointed their guns
and fired at Montano's group.
Thereafter, after uttering the
words, "nataydan, mapan tayon"
(They are already dead. Let us go),
Valdez and companions left. The
shooting incident left Ramon
Garcia, Jean Marie Garcia, Sandra
Montano and Willie Acosta dead.
The trial court held that the
accused Ronaldo Valdez is guilty
beyond reasonable doubt of the
crime of MULTIPLE MURDER

WITH
DOUBLE
FRUSTRATED
MURDER defined and penalized
under Republic Act No. 7659
otherwise known as the Heinous
Crime Law, the offense having
been a complex crime the penalty
of which is in the maximum, and
with the attendant aggravating
circumstances
of
evident
premeditation
and
abuse
of
superior
strength,
hereby
sentences
him
the
ultimum
suplicum of DEATH to be executed
pursuant to Republic Act No. 8177
known as the Lethal Injection Law,
to pay the heirs of the deceased
RAMON GARCIA, JR., WILLIE
ACOSTA, JEMARIE GARCIA and
SANDRA MONTANO and the
injured
victims
WILLIAM
MONTANO and RANDY TIBULE.
Issue:
WON the trial court erred when it
allowed itself to be carried away
by the erroneous Information filed
by the Office of the Provincial
Prosecutor charging the complex
crime of multiple murder and
double frustrated murder.
Held:
When a single act constitutes two
or more grave or less grave
felonies or when an offense is a
necessary means for committing
the other, the penalty for the most
serious crime shall be imposed, the
same to be applied in its maximum
period. The case at bar does not

fall under any of the two instances


defined above. The Office of the
Provincial
Prosecutor
of
Pangasinan
erroneously
considered the case as falling
under the first. It is clear from the
evidence on record, however, that
the four crimes of murder resulted
not from a single act but from
several individual and distinct acts.
For one thing, the evidence
indicates that there was more than
one gunman involved, and the act
of each gunman is distinct from
that of the other. It cannot be said
therefore, that there is but a single
act of firing a single firearm. There
were also several empty bullet
shell recovered from the scene of
the crime. This confirms the fact
that several shots were fired.
Furthermore,
considering
the
relative position of the gunmen
and their victims, some of whom
were riding the motorized tricycle
itself while the others were seated
inside the sidecar thereof, it was
absolutely impossible for the four
victims to have been hit and killed
by a single bullet. Each act by each
gunman pulling the trigger of their
respective firearms, aiming each
particular moment at different
persons constitute distinct and
individual acts which cannot give
rise to the complex crime of
multiple murder.

REODICA V. CA, 292 SCRA 87


Facts:
Isabelita Reodica was allegedly
recklessly driving a van and hit
Bonsolcausing
him
physical
injuries and damage to property
amounting to P 8,542.00. Three
days after the accident a complaint
was filed before the fiscals office
against the petitioner. She was
charged of "Reckless Imprudence
Resulting in Damage to Property
with Slight Physical Injury."After
pleading not guilty trial ensued.
RTC of Makati rendered the
decisionconvicting petitioner of
"quasi
offense
of
reckless
imprudence, resulting indamage to
property with slight physical
injuries" with arresto mayor of
6months imprisonment and a fine
of P 13,542.00. Petitioner made an
appealbefore the CA which reaffirmed
the
lower
courts
decision.
In
its
motion
for
reconsideration, petitioner now
assails that the court erred in
giving its penalty on complex
damage to property and slight
physical injuries both being light
offenses over which the RTC has
no jurisdiction and it cannot
impose penalty in excess to what
the law authorizes. And that
reversal of decision is still possible
on ground of prescription or lack
of jurisdiction.
Issues:

1. Whether or not the duplicity


of the information may be
questioned for the first time
on appeal.
2. Whether or not the duplicity
of the information may be
questioned for the first time
on appeal.
Held:
1.When a reckless, imprudent or
negligent act results in two or
more grave or less grave felonies,
a complex crime is committed.
Applying article 48, it follows that
if one offense is light, there is no
complex crime. The resulting
offenses may be treated as
separate or the light felony may be
absorbed by the grave felony.
Thus, the light felonies of damage
to property and slight physical
injuries, both resulting from a
single act of imprudence, do not
constitute a complex crime. They
cannot
be
charged
in
one
information. They are separate
offenses
subject
to
distinct
penalties. However, petitioner may
no longer question, at this stage,
the duplicitous character of the
information, i.e., charging two
separate
offenses
in
one
information.
This
defect
was
deemed waived by her failure to
raise it in a motion to quash before
she pleaded to the information.
Under Section 3, Rule 120 of the

Rules of Court, when two or more


offenses are charged in a single
complaint or information and the
accused fails to object to it before
trial, the court may convict the
accused of as many offenses as are
charged and proved and impose on
him the penalty for each of them.
2. Prescription in criminal cases is
a matter of substantive law.
Pursuant to Section 5(5), Article
VIII of the Constitution, this Court,
in the exercise of its rule-making
power, is not allowed to diminish,
increase or modify substantive
rights. Hence, in case of conflict
between the Rule on Summary
Procedure and the Revised Penal
Code, the latter prevails. The
petitioners violation is governed
by the RPC hence the prescriptive
period shall follow that of the RPC.
The
prescriptive
period
was
interrupted by the filing of the
complaint with the fiscals office
three days after the accident. It is
only in violations of municipal
ordinance that the prescriptive
period is interrupted by the
institution of judicial proceedings

RULE 111
PROSECUTION
ACTION

OF

CIVIL

PHIL. RABBIT BUS LINES INC. V.


PEOPLE, G.R. NO. 147703, APRIL
14, 2004
Facts:
The accused was found guilty and
convicted of the crime of reckless
imprudence resulting to triple
homicide,
multiple
physical
injuries and damage to property.
The court further ruled that in the
event of the insolvency of accused,
Phil. Rabbit Bus Line shall be
liable for the civil liabilities of the
accused. Evidently, the judgment
against accused had become final
and executory. The accused had
jumped bail and remained at-large.
Counsel for accused filed a notice
of appeal which was denied by the
trial court. Section 8, Rule 124 of
the Rules of Court authorizes the
dismissal of appeal when appellant
jumps bail. Petitioner filed its
notice of appeal from the judgment
of the trial court. The OSG moved
to
be
excused
from
filing
respondents brief on the ground
that the OSGs authority to
represent People is confined to
criminal cases on appeal. The
motion was however denied. The
CA ruled that the institution of a
criminal
case
implied
the
institution also of the civil action
arising from the offense. Thus,
once determined in the criminal
case
against
the
accusedemployee,
the
employers
subsidiary civil liability as set forth
in Article 103 of the Revised Penal

Code becomes conclusive and


enforceable. The appellate court
further held that to allow an
employer to dispute independently
the civil liability fixed in the
criminal case against the accusedemployee would be to amend,
nullify or defeat a final judgment.
Since the notice of appeal filed by
the accused had already been
dismissed by the CA, then the
judgment of conviction and the
award of civil liability became final
and executory. Included in the civil
liability of the accused was the
employers subsidiary liability.
Issue:
WON an employer, who dutifully
participated in the defense of its
accused employee, may appeal the
judgment
of
conviction
independently of the accused.
Held:
Only the civil liability of the
accused arising from the crime
charged is deemed impliedly
instituted in a criminal action, that
is, unless the offended party
waives the civil action, reserves
the right to institute it separately,
or institutes it prior to the criminal
action. Hence, the subsidiary civil
liability of the employer under
Article 103 of the Revised Penal
Code
may
be
enforced
by
execution on the basis of the
judgment of conviction meted out
to the employee. It is clear that the
2000
Rules
deleted
the
requirement
of
reserving
independent civil actions and
allowed
these
to
proceed
separately from criminal actions.
Thus, the civil actions referred to
in Articles 32, 33, 34 and 2176 of
the Civil Code shall remain

separate, distinct and independent


of any criminal prosecution based
on the same act. What is deemed
instituted
in
every
criminal
prosecution is the civil liability
arising from the crime or delict per
se (civil liability ex delicto), but not
those liabilities arising from quasidelicts,
contracts
or
quasicontracts. In fact, even if a civil
action is filed separately, the ex
delicto civil liability in the criminal
prosecution remains, and the
offended party may -- subject to
the control of the prosecutor -- still
intervene in the criminal action, in
order to protect the remaining civil
interest therein. This discussion is
completely in accord with the
Revised Penal Code, which states
that every person criminally liable
for a felony is also civilly
liable.Undisputedly, petitioner is
not a direct party to the criminal
case, which was filed solely against
Napoleon M. Roman, its employee.
In its Memorandum, petitioner
cited a comprehensive list of cases
dealing with the subsidiary liability
of employers. Thereafter, it noted
that none can be applied to it,
because in all those cases, the
accuseds
employer
did
not
interpose an appeal. Indeed,
petitioner cannot cite any single
case in which the employer
appealed, precisely because an
appeal in such circumstances is
not possible.
The cases dealing with the
subsidiary liability of employers
uniformly declare that, strictly
speaking, they are not parties to
the
criminal
cases
instituted
against their employees. Although
in substance and in effect, they

have an interest therein, this fact


should be viewed in the light of
their subsidiary liability. While
they may assist their employees to
the extent of supplying the latters
lawyers, as in the present case, the
former cannot act independently
on their own behalf, but can only
defend the accused.

JAVIER V. IAC, G.R. NO. 75379,


MARCH 31, 1989
Facts:
It is not disputed that the private
respondent
issued
to
the
petitioners a check that was
subsequently dishonored and not
made good despite the required
notice of dishonor. For this he has
been charged with estafa under
B.P. Blg. 22 in the Regional Trial
Court of Makati. The information
against Leon S. Gutierrez, Jr. was
filed on April 1, 1985, and
docketed as Criminal Case No.
15581 in the Regional Trial Court
of Makati. The civil case was not
reserved. On September 5, 1985,
Gutierrez filed a complaint for
damages against the petitioners in
the Regional Trial Court of
Catarman, Northern Samar. In this
complaint, the defendants were
charged with having inveigled
Gutierrez into signing the very
check subject of the criminal case
in the Makati court. The complaint
in effect explains why he issued
the check for which he is now
facing prosecution. The petitioners
filed a motion to dismiss the case
against them on the grounds of
lack of a cause of action and litis
pendentia but was denied. The
private respondent moved to
suspend proceedings in Criminal
Case No. 15581 pending the
resolution of what was claimed to
be the prejudicial question raised
in the civil case. The petitioners
filed an opposition. The motion
was also to be denied later.
Issue:
WON he can raise that reason fir
his issuance of the check in
another court, in a separate civil

action for damages filed by him


against the petitioners
Held:
As the civil action was not
reserved by the petitioners, it was
deemed impliedly instituted with
the criminal case in the Regional
Trial Court of
Makati.
The
applicable provision is Rule 111,
Section 1, of the Rules of Court. It
was before the Makati court that
the
private
respondent,
as
defendant in the criminal charge of
violation of B.P. Blg. 22, could
explain why he had issued the
bouncing check. As the civil action
based on the same act was also
deemed filed there, it was also
before that same court that he
could offer evidence to refute the
claim for damages made by the
petitioners. This he should have
done in the form of a counterclaim
for damages for his alleged
deception by the petitioners. In
fact,
the
counterclaim
was
compulsory and should have been
filed by the private respondent
upon the implied institution of the
civil action for damages in the
criminal action.
This being so, it was improper for
the private respondent to file his
civil complaint in the Regional
Trial Court of Northern Samar
alleging the very defense he
should be making in the Regional
Trial Court of Makati. It is, of
course, not possible for him now to
invoke a different defense there
because he would be contradicting
his own verified complaint in the
Regional Trial Court in Northern
Samar. In effect, therefore, he is
arguing that both courts have
jurisdiction to consider the same

claim of deception he is making in


connection
with
the
same
transaction and involving the same
parties.
The Court suspects that not having
set it up against the civil claim for
damages in the Regional Trial
Court of Makati, the private
respondent is now seeking to make
amends by filing a separate civil
action based on the same matter in
the Regional Trial Court of
Northern Samar. That is bad
enough. But what could be worse
is that he may have filed the civil
complaint in the second court for
the more censurable purpose of
deliberately delaying the trial of
the criminal case, which has been
deferred long enough as it is. That
should not be permitted.

CASUPANAN V. CA, LAROYA, G.R.


NO. 145391, AUGUST 26, 2002
Facts:
As a result of a vehicular accident
between two vehicles, one driven
by Mario Llavore Laroya and the
other owned by Roberto Capitulo
and driven by Avelino Casupanan,
two cases were filed before the
MCTC of Capas, Tarlac. Laroya
filed a criminal case against
Casupanan
for
reckless
imprudence resulting in damage to
property. This case was on its
preliminary investigation stage
when Casupanan and Capitulo
filed a civil case against Laroya for
quasi-delict. However, upon motion
of Laroya on the ground of forumshopping, the MCTC dismissed the
civil
case.
On
Motion
for
Reconsideration, Casupanan and
Capitulo insisted that the civil case
is a separate civil action which can
proceed independently of the
criminal case. Casupana nand
Capitulo then filed a petition for
certiorari before the RTC of
Capas,Tarlac. But the RTC ruled
that the order of dismissal issued
by the MCTC is a final order which
disposes of the case and therefore,
the proper remedy should have
been an appeal. Hence,Casupanan
and Capitulo filed this petition.
Casupanan and Capitulo contends
that if the accused in a criminal
case has a counter claim against
the private complainant, he may
file the counterclaim in a separate
civil action at the proper time.
They contend that an action on
quasi-delict is different from an
action resulting from the crime of
reckless imprudence, and an
accused in a criminal case can be

an aggrieved party in a civil case


arising from the same incident.
They maintain that under Articles
31 and 2176 of the Civil Code, the
civil
case
can
proceed
independently of the criminal
action. Finally, they point out that
Casupanan was not the only one
who filed the independent civil
action based on quasi-delict but
also Capitulo, the owner-operator
of the vehicle, who was not a party
in the criminal case.
ISSUE:
WON an accused in a pending
criminal
case
for
reckless
imprudence
can
validly
file,
simultaneously and independently,
a separate civil action for quasidelict
against
the
private
complainant in the criminal case.
Held:
Under Section 1 of the present
Rule 111, the independent civil
action in Articles 32, 33, 34 and
2176 of the Civil Code is not
deemed
instituted
with
the
criminal action but may be filed
separately by the offended party
even without reservation. The
commencement of the criminal
action does not suspend the
prosecution of the independent
civil action under these articles of
the Civil Code. The suspension in
Section 2 of the present Rule 111
refers only to the civil action
arising from the crime, if such civil
action is reserved or filed before
the commencement of the criminal
action. Thus, the offended party
can file two separate suits for the
same act or omission. The first a
criminal case where the civil
action to recover civil liability exdelicto is deemed instituted, and

the other a civil case for quasidelict - without violating the rule
on non-forum shopping. The two
cases can proceed simultaneously
and independently of each other.
The commencement or prosecution
of the criminal action will not
suspend the civil action for quasidelict. The only limitation is that
the offended party cannot recover
damages twice for the same act or
omission of the defendant. In most
cases, the offended party will have
no reason to file a second civil
action since he cannot recover
damages twice for the same act or
omission of the accused. In some
instances, the accused may be
insolvent, necessitating the filing
of another case against his
employer or guardians.
Similarly, the accused can file a
civil action for quasi-delict for the
same act or omission he is accused
of in the criminal case. This is
expressly allowed in paragraph 6,
Section 1 of the present Rule 111
which states that the counterclaim
of the accused may be litigated in
a separate civil action. This is only
fair for two reasons. First, the
accused is prohibited from setting
up any counterclaim in the civil
aspect that is deemed instituted in
the criminal case. The accused is
therefore
forced
to
litigate
separately
his
counterclaim
against the offended party. If the
accused does not file a separate
civil action for quasi-delict, the
prescriptive period may set in
since the period continues to run
until the civil action for quasidelict is filed. Second, the accused,
who is presumed innocent, has a
right to invoke Article 2177 of the

Civil Code, in the same way that


the offended party can avail of this
remedy which is independent of
the criminal action. To disallow the
accused from filing a separate civil
action
for
quasi-delict,
while
refusing
to
recognize
his
counterclaim in the criminal case,
is to deny him due process of law,
access to the courts, and equal
protection of the law.
Thus, the civil action based on
quasi-delict filed separately by
Casupanan and Capitulo is proper.
The order of dismissal by the
MCTC on the ground of forumshopping is erroneous

GENERAL V. CLARAVALL, G.R.


NO. 96724, MARCH 22, 1991
Facts:
Benneth Thelmo filed with the
Office of the Public Prosecutor of
Rizal a sworn complaint accusing
Honesto General and another
person of libel, and alleged that by
reason of the offense he (Thelmo)
had suffered actual, moral and
exemplary damages in the total
sum
of
P100
million.
The
information for libel subsequently
filed with the RTC at Pasig, after
preliminary investigation, did not
however contain any allegation
respecting the damages due the
offended party. At the trial, the
defense raised the issue of nonpayment of the docket fees
corresponding to the claim of
damages contained in Thelmo's
sworn complaint before the fiscal,
as a bar to Thelmo's pursuing his
civil action therefor. The trial
Court overruled the objection, by
Order dated March 28, 1990. It
also denied the defendants' motion
for reconsideration and motion for
suspension of proceedings, by
another Order dated May 17,
1990. General and his co-accused
are now before this Court applying
for a writ of certiorari to annul the
aforesaid Orders of the Trial Court
on the theory that they had been
rendered with grave abuse of
discretion. He now questions
whether or not Manchester vs. CA,
149 SCRA 562 (1987) is no longer
binding
Issue:
WON the rule should now be that
the filing fees for the civil action
for the recovery of civil liability
arising from the offense should

first be paid in order that said civil


action may be deemed to have
been impliedly instituted with the
criminal and prosecuted in due
course
Held:
Manchester laid down the doctrine
the specific amounts of claims of
damages must be alleged both in
the body and the prayer of the
complaint, and the filing fees
corresponding thereto paid at the
time of the filing of the complaint;
that if these requisites were not
fulfilled, jurisdiction could not be
acquired by the trial court; and
that amendment of the complaint
could
not
"thereby
vest
jurisdiction. Under the 1985
Rules, the filing fees for the civil
action impliedly instituted with the
criminal had to be paid first to the
Clerk of the court where the
criminal action was commenced,
without regard to whether the
claim for such damages was set
out in the information or not.
Under the 1988 Rules, however, it
is only when "the amount of
damages, other than actual, is
alleged in the complaint or
information
(that)
the
corresponding filing fees shall be
paid by the offended party upon
the filing thereof in court for trial."
In any other casei.e., when the
amount of damages other than
actual is NOT alleged in the
complaint
or
informationthe
filing fees for the civil action "to
enforce civil liability against the
accused by way of moral, nominal,
temperate or exemplary damages .
. . shall (merely) constitute a first
lien on the judgment except in an
award for actual damages."

This Court's plain intentto make


the Manchester doctrine, requiring
payment of filing fees at the time
of the commencement of an action
applicable to impliedly instituted
civil actions under Section 1, Rule
111 only when "the amount of
damages, other than actual, is
alleged in the complaint or
informationhas thus been made
manifest by the language of the
amendatory provisions.
In any event, the Court now makes
that intent plainer, and in the
interests of clarity and certainty,
categorically declares for the
guidance of all concerned that
when a civil action is deemed
impliedly
instituted
with
the
criminal
in
accordance
with
Section 1, Rule 111 of the Rules of
Courtbecause the offended party
has NOT waived the civil action, or
reserved the right to institute it
separately, or instituted the civil
action prior to the criminal action
the rule is as follows:
1)
when
"the
amount
of
damages, other than actual, is
alleged in the complaint or
information" filed in court, then
"the corresponding filing fees shall
be paid by the offended party upon
the filing thereof in court for trial;"
2)
in any other case, however
i.e., when the amount of damages
is not so alleged in the complaint
or information filed in court, the
corresponding filing fees need not
be
paid
and
shall
simply
"constitute a first lien on the
judgment, except in an award for
actual damages.

LIM V. CA, G.R. NO. 130038,


SEPTEMBER 18, 2008
Facts:
On August 25, 1990, petitioner
bought various kinds of jewelry
worth P300,000.00 from Maria
Antonia Seguan. She wrote out a
check with the same amount,
dated August 25, 1990, payable to
cash drawn on Metrobank and
gave the check to Seguan. The
next day, petitioner again went to
Seguans store and purchased
jewelry valued at P241,668.00.
Petitioner issued another check
payable to cash dated August 16,
1990 drawn on Metrobank in the
amount of P241,668.007 and sent
the check to Seguan through a
certain Aurelia Nadera. Seguan
deposited the two checks with her
bank. The checks were returned
with
a
notice
of
dishonor.
Petitioners account in the bank
from which the checks were drawn
was
closed.
Upon
demand,
petitioner promised to pay Seguan
the amounts of the two dishonored
checks, but she never did. On June
5,
1991,
an
Assistant
City
Prosecutor of Cebu filed with the
RTC, Cebu City, Branch 23, two
informations against petitioner for
violations of BP No. 22. After due
trial, on December 29, 1992, the
trial court rendered a decision in
the two cases convicting petitioner.
Petitioner appealed to the CA, but
the same was dismissed by the CA
in its October 15, 1996 Decision
wherein it affirmed in toto the
RTCs Decision
Issue:
WON Lim violated B.P. No. 22
Held:

The gravamen of B.P. No. 22 is the


act of making and issuing a
worthless check or one that is
dishonored upon its presentment
for payment. And the accused
failed to satisfy the amount of the
check or make arrangement for its
payment within 5 banking days
from notice of dishonor. The act is
malum prohibitum, pernicious and
inimical to public welfare. Laws
are created to achieve a goal
intended and to guide and prevent
against an evil or mischief. Why
and to whom the check was issued,
and the terms & conditions
surrounding the issuance of the
checks,
are
irrelevant
in
determining culpability.
Under BP No. 22, one need not
prove that the check was issued in
payment of an obligation, or that
there was damage. Since the act
isa mala prohibita, it is not
necessary that Lim acted with
criminal intent. The intent is
immaterial. This case is a perfect
example of an act mala prohibita.
The first and last elements of the
offense are admittedly present.
B.P. No. 22, Section 2 creates a
presumption juris tantum that the
second element prima facie exists
when the first and third elements
of the offense are present. If not
rebutted, it suffices to sustain a
conviction. To escape liability, she
must prove that the second
element was absent. Petitioner
failed to rebut this presumption
and she failed to pay the amount of
the checks or make arrangement
for its payment within 5 banking
days from receipt of notice of
dishonor. B.P. No. 22 was clearly
violated. Hoc quidem per quam

durum est sed ita lex scripta est.


The law may be exceedingly hard
but so the law is written.

TAN V. MENDEZ, G.R. NO.


138669, JUNE 6, 2002
Facts:
Petitioners
Steve
Tan
and
Marciano Tan are the owners of
Master
Tours
and
Travel
Corporation and operators of
Philippine Lawin Bus Co., Inc.,
while respondent Fabian Mendez,
Jr. is the owner of three gasoline
stations in Iriga City, Ligao, Albay,
and
Sipocot,
Camarines
Sur.
Petitioners opened a credit line for
their buses lubricants and fuel
consumption with respondent. At
the same time, the latter was also
designated by petitioners as the
booking and ticketing agent of
Philippine Lawin Bus Co. in Iriga
City.
Petitioners issued several checks
to respondent as payment for oil
and fuel products. One of the
checks was dishonored by the
bank
upon
presentment
for
payment for being drawn against
insufficient funds. Respondent sent
a
demand
letter
dated
to
petitioners demanding that they
make good the check or pay the
amount thereof, to no avail.
However, petitioners failed to pay
the amount thereof. Hence, an
information for violation of B.P. 22
was filed against petitioners before
the RTC.
Petitioners argue that he cannot be
held liable for violation of B.P. 22
because the amount subject of the
check
had
already
been
extinguished
by
offset
or
compensation
against
the
collection from ticket sales from
the booking offices. He presented
a memorandum showing the return
to
respondent
of
various

unencashed checks in the total


amount
of
Php
66,839.25
representing remittance of ticket
sales that were earlier sent by
respondent. After the alleged
offset, there remains a balance of
P226,785.83.
On cross-examination, Marciano
admitted to have drawn the
subject check to pay private
respondents gasoline station and
that it was not covered by
sufficient funds at the time of its
issuance
due
to
uncollected
receivables. Upon query by the
court, he claimed that he did not
talk to private complainant and
could not tell if the latter agreed to
offset
the
checks
with
the
remittances.
Respondent disputed petitioners
claim of payment through offset or
compensation:
1.
the amount of the four
unencashed
checks
totaling
P66,839.25 could not have offset
the amount of the dishonored
checks since petitioners total
obligations at that time had
already reached P906,000; and
2.
the compensation did not
take place as there was no
application of payment made by
the
petitioners
in
their
memorandum dated June 10,1991.
The trial court ruled against the
petitioners,
convicting
the
petitioners for violation of B.P. 22.
On appeal, the Court of Appeals
affirmed the decision of the trial
court
Issue:
WON
payment
through
compensation
can
offset
or
preclude prosecution for violation
of B.P. 22.

Held:
The law has made the mere act of
issuing a bum check a malum
prohibitum, an act proscribed by
legislature for being deemed
pernicious and inimical to public
welfare. The gravamen of the
offense under this law is the act of
issuing a worthless check or a
check that is dishonored upon its
presentment for payment. Thus,
even if there had been payment,
through compensation or some
other means, there could still be
prosecution for violation of B.P. 22.
No application of payment
was made as to which check was
to be paid. These factual findings
should be accorded respect and
finality as the trial court is in the
best position to assess and
evaluate questions of fact. These
findings will not be disturbed on
appeal in the absence of any clear
showing that the trial court
overlooked
certain
facts
or
circumstances
that
would
substantially affect the disposition
of the case

REPUBLIC V. BELLO, G.R. NO. L34906; JANUARY 27, 1983


Facts:
Private respondent Arceo, in his
capacity as Cashier and Disbursing
Officer of Capiz Agricultural and
Fishery School, was charged for
malversation of public funds in the
amount of Php6,619.34 which he
supposedly failed to produce or to
make proper accounting thereof
after repeated demands. After due
trial, the respondent Court of First
Instance of Capiz, finding the
evidence of the prosecution not
sufficient to establish the guilt of
the accused beyond reasonable
doubt,
rendered
a
decision
acquitting Arceo on the following
grounds:
1.
undisputed facts clearly and
unmistakably show lack of criminal
intent on accused's part;
2.
absence of proof that the
accused benefited personally from
his disbursements nor has it been
shown that he was inexcusably
negligent in the administration of
public
funds
and
properties
entrusted to his care;
3.
it has not been shown and
proven
that
the
government
suffered damage or prejudice as
the accused's disbursements were
for the benefit of the Capiz
Agricultural and Fishery School;
and
4.
the funds claimed to be
missing
in
the
amount
of
Php6,619.34 is not really missing
for the accused demonstrated that
said amounts were spent for and in
the
interest
of
the
Capiz
Agricultural and Fishery School.
After the acquittal of Arceo, the
Provincial Fiscal filed a civil case

against Arceo for the recovery of


the total sum of Php13,790.71
which
represented
the
accountability of Arceo due to his
failure to issue official receipts and
to immediately deposit said funds
with the National Treasury.
Arceo filed a motion to dismiss
the complaint in the said civil case
alleging that the petitioner had no
cause of action against him
inasmuch as "the cause of action
had been decided in a prior
judgment."
Issue:
WON the acquittal of Arceo in the
criminal case bars the filing of the
civil action against him.
Held:
No. The Court ruled that the
decision did not absolve Arceo or
free him from responsibility insofar
as his accountability as Cashier
and
Disbursing
Officer
is
concerned.
A judgment of acquittal on the
ground that accused had no
criminal intent and that the
evidence of the prosecution was
not sufficient to establish the guilt
of the accused does not bar a civil
action for recovery of government
funds disturbed without prior
approval by the auditor. Failure to
establish
that
the
accused
committed the crime beyond
reasonable doubt does not mean
he should not be civilly liable. Only
preponderance of evidence is
needed for the civil aspect in
contrast of the proof beyond
reasonable doubt needed for the
criminal aspect
It is to be noted that the subject
subject-matter of the malversation
case
was
the
amount
of

Php6,619.34, the sum sought to be


recovered in the civil action
totalled
Php13,790.70
which
included the additional sum of
Php7,170.31
representing
the
income of the school from its
various projects for which the
accused failed to issue official
receipts.
Even insofar as the amount of
P6,619.34
is
concerned,
the
finding by the respondent court is
not a declaration that the fact
upon which the civil case is based
does not exist. The civil action
barred by such a declaration is the
civil liability arising from the
offense charged, which is the one
impliedly
instituted
with
the
criminal action. Such a declaration
would not bar a civil action filed
against an accused who had been
acquitted in the criminal case if
the criminal action is predicated
on factual or legal considerations
other than the commission of the
offense charged.
Therefore, Arceo cannot invoke
the provision of Section 3(c) of
Rule 111 of the Rules of Court to
dismiss the civil case filed against
him.

DELA CRUZ V. EJERCITO, G.R.


NO. L-40895; NOVEMBER 6, 1975
Facts:
On May 20, 1974, the first
husband of Milagros Dela Cruz
filed a complaint in the Court of
First
Instance
of
Pampanga
charging the Dela Cruz with
bigamy
for
having
married
Sergeant Dominick L. Gaccino on
September 15, 1973. On August 1,
1974, Dela Cruz filed in the same
court
a
complaint
for
the
annulment of her marriage with
Gaccino on the ground of duress.
Since the court did not receive an
answer from Gaccino and there
was no collusion on the part of
both parties, Judge Castaneda
rendered a decision annulling the
marriage of Dela Cruz and
Gaccino. The decision became
final. On January 27, 1975, in view
of the annulment of her second
marriage, Dela Cruz filed a motion
to dismiss the bigamy charge
against her. Judge Ejercito denied
the motion to dismiss on the
ground that the decision in the
annulment case is not controlling
in the criminal case because the
parties and the issues in the two
cases are not the same.
Issue:
WON the bigamy case became
moot or untenable after the second
marriage was annulled.
Ruling:
Yes.
It
is
necessary
in
a
prosecution for bigamy that the
second marriage be declared valid
if its validity is questioned in a civil
action. The Court ruled that the
finding in the annulment case that
the second marriage contracted by
Dela Cruz with Sergeant Gaccino

was a nullity is determinative of


her innocence and precludes the
rendition of a verdict that she
committed bigamy. To try the
criminal case in the face of such a
finding would be unwarranted.
Furthermore, the Court said that
even supposing that the decree
annulling the second marriage was
questionable or erroneous because
it was issued in a judgment by
default, still that would not prevent
the decree from having legal
effect. "An erroneous judgment is
not a void judgment."

DONATO V. LUNA, G.R NO. L53642; APRIL 15, 1988


Facts:
On January 23, 1979, the City
Fiscal
of
Manila
filed
an
information for bigamy against
Leonilo C. Donato with the Court
of First Instance of Manila based
on the complaint of private
respondent
Paz
Abayan.
On
September 28, 1979, before the
petitioner's arraignment, private
respondent filed with the Juvenile
and Domestic Relations Court of
Manila
a
civil
action
for
declaration of nullity of her
marriage with petitioner on the
ground that the private respondent
had no previous knowledge that
the petitioner was already married
to a certain Rosalinda R. Maluping.
Prior to the date set for the trial on
the merits of the bigamy case,
petitioner filed a motion to
suspend the proceedings of said
case on the ground that the civil
case seeking the annulment of his
second
marriage
raises
a
prejudicial question which must
first be determined or decided
before the criminal case can
proceed.
Hon. Artemon D. Luna denied the
motion to suspend the proceedings
for bigamy based on the ruling laid
down in the case of Landicho vs.
Relova. Petitioner filed a motion
for reconsideration citing as one of
his grounds for suspension of
proceedings the ruling laid down
in the case of De la Cruz vs.
Ejercito. The motion was likewise
denied due to lack of merit.
Issue:
WON a criminal case for bigamy
pending before the lower court be

suspended in view of a civil case


for annulment of marriage pending
before the juvenile and domestic
relations court on the ground that
latter constitutes a prejudicial
question.
Ruling:
No. The requisites of a prejudicial
question is not present in the case
at bar.
A prejudicial question has been
defined to be one which arises in a
case, the resolution of which
question is a logical antecedent of
the issue involved in said case, and
the cognizance of which pertains
to another tribunal. It is one based
on a fact distinct and separate
from the crime but so intimately
connected
with
it
that
it
determines the guilt or innocence
of the accused, and for it to
suspend the criminal action, it
must appear not only that said
case involves facts intimately
related to those upon which the
criminal prosecution would be
based but also that in the
resolution of the issue or issues
raised in the civil case, the guilt or
innocence of the accused would
necessarily be determined.
The mere fact that there are
actions to annul the marriages
entered into by the accused in a
bigamy case does not mean that
"prejudicial
questions"
are
automatically raised in civil actions
as to warrant the suspension of the
case. In order that the case of
annulment
of
marriage
be
considered a prejudicial question
to the bigamy case against the
accused, it must be shown that the
petitioner's
consent
to
such
marriage must be the one that was

obtained by means of duress, force


and intimidation to show that his
act in the second marriage must be
involuntary and cannot be the
basis of his conviction for the
crime of bigamy. There is no
prejudicial question for the case
does not have an issue intimately
related and determinative in the
criminal case of bigamy because
he who contracts a second
marriage
before
the
judicial
declaration of nullity of the first
marriage assumes the risk of being
prosecuted for bigamy.
In the case at bar, petitioner has
not even sufficiently shown that his
consent to the second marriage
has been obtained by the use of
threats, force and intimidation.
With regard to the contention of
the petitioner alleging that the
case of Dela Cruz vs. Ejercito
should be applied to the case at
bar, the Court said that the Dela
Cruz case and the case at bar are
markedly different due to the
following reasons:
1.Dela Cruz, the party who was
accused of bigamy was the one
who filed an action or annulment
of the second marriage while in
the case at bar, it was the private
respondent who filed an action for
the annulment of their marriage;
and
2.A
judgment
was
already
rendered in the civil case that the
second marriage of De la Cruz was
null and void, thus determinative
of the guilt or innocence of the
accused in the criminal case. In
the present case, there is as yet no
such judgment in the civil case.

IAN PANGAN

Ras vs. Rasul


G.R. 50441-42, September 18,
1980
FACTS: Luis Pichel filed a civil
complaint
against
petitioner
Alejandro Ras and a certain
Bienvenido Martin praying for the
nullification of the deed of sale
executed by Alejandro Ras in favor
of his co-defendant Bienvenido
Martin and for the declaration of
the prior deed of sale allegedly
executed in his favor by the
defendant Alejandro Ras as valid.
In their answer, the defendants
(the Ras spouses) alleged that they
never sold the property to Pichel
and that the signatures appearing
in the deed of sale in favor of
plaintiff Pichel (in Civil Case No.
73) were forgeries and that
therefore the alleged deed of sale
in Pichels favor sought to be
declared valid was fictitious and
inexistent.
While
the
aforementioned civil complaint
was being tried, an Information for
Estafa against Alejandro Ras was
filed arising from the same alleged
double sale subject matter of the
civil complaint filed by Pichel. Ras,
through counsel moved for the
suspension
of
the
criminal
complaint on the grounds of
prejudicial question praying that
the criminal complaint for estafa
be first suspended as the pendency
of
the
civil
complaint
aforementioned is determinative of
his guilt in the estafa case.

ISSUE: Is there a prejudicial


question?
HELD: Yes. There is prejudicial
question when the civil case not
only involves the same facts upon
which the criminal prosecution is
based, but also that the resolution
of the issues raised in said civil
action
would
be
necessarily
determinative of the guilt or
innocence of the accused. Criminal
action for estafa (for alleged
double sale of property) is a
prejudicial question to a civil
action for nullity of the alleged
deed of sale and defense of the
alleged vendors of forgeries of
their signatures in the deed
Salazar vs. People
G.R. No. 15193, September 23,
2003
FACTS: An Information for estafa
was
filed
against
petitioner
Anamer D. Salazar and co-accused
Nena
Jaucian
Timario.
This
criminal case for estafa originated
after petitioners issued in favour of
JY Brothers Marketing Corporation
a check with an amount of P214,
000 representing the payment of
300 cavans despite knowing fully
that said check has no sufficient
funds.
After
the
prosecution
presented its evidence, petitioner
filed a demurrer to evidence to to
prove that she is not liable for the
civil liability. Nevertheless, the
trial court rendered judgment
acquitting the petitioners of the
crime charged but in its judgement
automatically ordered her to remit
to the private complainant the
amount of said check without

further
hearing.
Hence,
this
petition.
ISSUE: Is the award proper?
HELD:
No.
In
automatically
granting the civil aspect of the
case by ordering petitioner to pay
for
her
purchases
from
complainant JY Brothers even
before they could adduce evidence
thereon, she is patently denied of
her right to due process.

Maniago vs. Court of Appeals


253 SCRA 674
FACTS: Petitoner Maniago owns
shuttle buses used in transporting
employees of Texas Instruments,
Inc from Baguio City to its plant
site located at Export Processing
Authority. In 1990, one of his buses
encountered a vehicular accident
with a passenger jeepney owned
by private respondent Boado.
Thereafter, a criminal case for
reckless imprudence resulting to
damage to property was filed
against the driver of Andava, the
driver of Maniago. A month later, a
civil complaint for damages was
also filed by Boado against
petitioner
Maniago.
Petitioner
Maniago moved for the suspension
of the proceedings in the civil
complaint on the ground that no
reservation was made in the

criminal case for independent civil


action.
ISSUE: Should the civil case be
dismissed on the ground that there
was no reservation?
HELD: Yes. . The right to bring an
action for damages under the Civil
Code must be reserved. Rule 111,
Section
1
requires
that
a
reservation must be made to
institute separately all civil actions
for the recovery of civil liability,
otherwise, they will be deemed to
have been instituted in the
criminal case. Such civil actions
are not limited to those which
arise from the offence charged.
In other words the right of the
injured party to sue separately for
the recovery of the civil liability
whether arising from crimes (ex
delicto) or from quasi delict under
Art. 2176 of the Civil Code must be
reserved otherwise they will be
deemed
instituted
with
the
criminal action. as required by
Rule 111, 1, otherwise it should
be dismissed.
Western Institute of Technology
vs. Salas
278 SCRA 216, August 21, 1997
FACTS: Private respondents Salas,
et. al. are the majority and
controlling members of the Board
of Directors of petitioner Western
Institute of Technology (WIT), a
stock corporation operating as an
educational
institution.
These
respondents were charged of
falsification of public documents
and estafa. The charge for
falsification of public document
was anchored on the private
respondents submission of WITs

income statement for the fiscal


year 1985-1986 with the Securities
and Exchange Commission (SEC)
reflecting
therein
the
disbursement of corporate funds
making it appear that the same
was passed by the board on March
30, 1986, when in truth, the same
was actually passed on June 1,
1986, a date not covered by the
corporations fiscal year 19851986. After a full-blown hearing TC
handed down a verdict of acquittal
on both counts without imposing
any civil liability against the
accused therein. This is after the
finding of the RTC that the
accused respondents did not
commit the act complained of. The
present petition was brought by
petitioners
arguing
that
respondents should be held liable
for payment of civil liability.
ISSUE: Are respondents liable for
civil liability?
HELD: No. Acquittal in a criminal
action bars the civil action arising
therefrom where the judgement of
acquittal holds that the accused
did not commit the criminal acts
imputed them. In the present case,
the acquittal of the accused is not
merely based on reasonable doubt
but rather on finding that the
accused-private respondents did
not commit the criminal acts
complained of. Thus, any civil
action ex delicto cannot prosper.
Elcano vs. Hill
77 SCRA 98
FACTS: Reginald Hill, a minor,
was prosecuted criminally for
killing Agapito Elcano, the son of
herein petitioners. The lower court
rendered judgement acquitting

Reginald Hill on the ground that


the killing was made with lack of
intent coupled with mistake.
Elcanos filed the present action to
recover damages from the father
of Reginald Hill, Atty. Hill. In the
reply of the latter, he argued that
the action to recover damages had
already been barred by the
acquittal of his son and that his
civil liability as a parent had
already been extinguished on the
ground that his son is already an
emancipated minor by reason of
his marriage.
ISSUE: Is the argument of Atty.
Hill correct?
HELD: No. A separate civil action
lies against the offender in a
criminal act, whether or not he is
criminally prosecuted and found
guilty or acquitted , provided that
the victim do not recover damages
on both scores. The vicarious
liability of the parents on account
of a delict committed by their
minor child is not extinguished by
the fact that said child is married.
Emancipation by marriage does
not carry with it the freedom to do
any act that can give rise to
judicial
litigation.
Otherwise
stated, the marriage of a minor
child does not relieve the parents
of the duty to see to it that the
child, while still a minor, does not
give cause to any litigation, in the
same manner that the parents are
answerable for the borrowings of
money
and
alienation
or
encumbering of real property
which cannot be done by their
minor without their consent.
RULE 112

PRELIMINARY
INVESTIGATION
Bautista vs. Sarmiento e. al.
L-45137, September 23, 1985
FACTS: An information charging
Fe Bautista and Teresita Vergere
with Estafa was filed before the
sala of Judge Sarmiento. Teresita
Vergere was granted separate
trial. To prove its case, the
prosecution presented in the trial
private complainant Leticia Yap as
the only witness. The petitioners
moved for the dismissal of the case
by way of demurrer to evidence.
Petitioners
alleged
that
they
receive the jewelries on a purchase
and sale arrangement and that the
prosecution
failed
to
prove
misappropriation made by them.
These allegations however were
controverted by the prosecution by
presenting the demand letter of
private complainant Leticia Yap.
Thus, respondent judge Sarmiento
denied the demurrer. In view of the
denial
of
the demurrer, Fe
Bautista, et. al. brought the
present petition for certiorari
claiming that there was only prima
facie case of estafa against them
and there was no proof beyond
reasonable doubt.
ISSUE: Considering that the
denial of petitioners demurrer is
merely interlocutory, is the present
petition for certiorari proper?
HELD:
No.
The
respondent
judges order denying petitioners
motion to dismiss the complaint by
way of demurrer to evidence is
merely an interlocutory order. It
cannot be the subject of a petition
for certiorari. What they could

have done was to continue with the


trial of the case and had the
decision been in adverse, to raise
the issue on appeal. When a prima
facie case is established by the
prosecution in a criminal case, the
burden of proof does not shif to the
defense. It remains throughout the
trial with the prosecution. It is the
burden of evidence which shift
from party to party depending on
the exigencies of the case. This
burden of going forward with the
evidence which balances that
introduced by the prosecution.
Then burden shifts back.
Talusan vs. Ofiana
L-31028, June 29, 1972
FACTS: Petitioner Talusan filed a
criminal complaint for murder
against
private
respondents
Ventura
Bartolome,
Renato
Valdecantos, Alfredo Valdecantos,
Valentin Valdecantos and Pito
Valdecantos in the MTC San
Rafael, Bulacan (Criminal Case No.
1112). Preliminary investigation
was conducted and a warrant of
arrest was issued against Ventura
Bartolome, Alfredo Valdecantos,
Renato Valdecantos, and Pito
Valdecantos. On the other hand,
respondents Valentin and Alfredo
Valdecantos also filed two charges
of attempted murder (I.S. Nos3607
and 3607-A) against petitioner
arising from the same incident
used as basis for Talusan in filing a
murder case against them
Second stage for preliminary
investigation of Criminal Case No.
1112 was conducted but private
respondent waived this and prayed
that the case be remanded to CFI

Baliwag, Bulacan. After the case


was remanded to the latter court,
private respondents asked that a
reinvestigation be done. This was
opposed by petitioner Talusan.
Nevertheless, the provincial fiscal
granted
the
prayer
for
reinvestigation.
ISSUE: Considering that MTC San
Rafael, Bulaca already conducted
preliminary investigation, is the
reinvestigation by the provincial
fiscal proper when the case was
remanded to CFI Baliwag?
HELD: Yes, it is proper. When a
Fiscal or prosecuting attorney
receives a criminal case, elevated
to the CFI by the Justice of the
Peace Court which has conducted
the
corresponding
preliminary
investigation, and on the ground
that there was a probable cause,
the said fiscal has the right to
conduct his own investigation to
convince himself of the sufficiency
of
said
evidences
for
the
prosecution. The power of the
provincial Fiscal to conduct his
own investigation of a case already
elevated to the CFI by the
municipal judge or justice of peace
who
conducted
preliminary
investigation thereon, in order to
determine his own course of action
as a prosecuting officer , is
particularly true in the present
case since counter-charges for
attempted murder have also been
filed against petitioner based on
the same incident . The Fiscal
certainly could not be expected to
proceed without first satisfying
himself
who
was
the
real
aggressor, for the combatant
parties
could
not
be

simultaneously both aggressors


and victims of the same event.

already been rendered functus


officio by the 1987 Constitution.

Ponsica vs. Ignalaga


G.R. No. 72301, July 31, 1987
FACTS: An attempt was made by
firemen and soldiers to disperse a
crowd of demonstrators massed in
front of Municipal Building of
Escalante, Negros Occidental with
the use of water and tear gas.
Eventually, there was a gun fire
and within moments, rallyists lay
dead on and by the National Road.
In view of the absence of the
MCTC judge in the area, Braulio
Lumayno, the town mayor of
Escalante , took cognizance of the
complaint filed by the Military
Station commander charging some
of the rallyists of inciting to
sedition
.
Thereafter,
Mayor
Lumayno conducted preliminary
investigation and ordered the
arrest of the demonstrators.
ISSUE: Does the municipal mayor
has
the
power
to
conduct
preliminary investigation in light of
the 1985 amendments of the rules
of
court
governing
criminal
procedure,
HELD: No. The 1985 Rules on
Criminal Procedure did not include
the mayor in the enumeration of
the officers authorized to conduct
preliminary investigation, those
listed being judges of the MTC and
MCTC; city or provincial fiscals
and their assistants; national and
regional state prosecutors; and
such other officers as may be
authorized by law. Furthermore,
the LGC granting the mayor the
power to conduct preliminary
investigation and order arrest had

People vs Villanueva
L-56443, December 19, 1981
FACTS: Rogelim Yee, with the
deliberate intent of bringing one
Ofelia Torralba, a 4th year student,
into disrepute wilfully attacked
and assaulted her and inflicted
contusions in her face in the
presence of her visitors and
classmates
to
her
great
embarrassment. Having conducted
the
required
preliminary
investigation and having found
probable
case
or
reasonable
ground to believe that there exist
probable cause for serious slander
by deed, the city fiscal of Butuan
City filed in the city court an
information or the said offence.
The
respondent
judge,
instead of issuing a warrant of
arrest conducted his ex parte
preliminary
investigation
to
determine once more the existence
of probable cause and thereafter
concluded
that
the
offense
committed is either slight slander
by deed of slight physical injuries
ISSUE: Whether or not the fiscals
preliminary
investigation
precludes
the
judge
from
exercising his legal duty to
exercise his judicial power of
determining before issuing the
corresponding warrant of arrest
the existence of probable cause.
HELD: No. The respondent judge
is clothed with the prerogative of
ascertaining probable cause as
provided in the Bill of Rights which
provides that no warrant of arrest
shall issue except upon probable
cause to be determined by the

judge This power however of


the judge does not include the
authority to dismiss outright the
information of the judge believes
that there is no probable cause.
The judge should require the fiscal
to present additional evidence to
show probable cause.
Solicitor General vs Garrido
L-28535, October 10, 1980
FACTS:
Petitioner
Abundio
Garrindo file a criminal complaint
against
respondent
Serafin
Quiason, Director of National
Library and co-respondent Manuel
Portugal for violation Civil Service
Law and the Revised Penal Code.
The Solicitor General authorised
Solicitor Raquel-Santos as counsel
fro Director Quiason in the
preliminary investigation to be
conducted by the office of the City
Fiscal. During the preliminary
investigation held by the Assisttant
Fiscal,
the
counsel
for
the
complainant
objected
to
the
appearance of Solicitor RaquelSantos in representation of the
Solicitor General contending that
the latter is not authorised by law
to appear on behalf of public
official who is criminally accused
in his private capacity.
ISSUE: Can the Solicitor General
represent a public official during
preliminary investigation in a
criminal case where the public
official is involved?
HELD: Yes. Section 1661 of the
Revised Administrative Code states
that :As principal law officer of the
government, the solicitor general
shall have authority to act for and
represent the Government of the
Philippines, its officers and agents

in
any
official
investigation,
proceeding or matter requiring the
services of a lawyer.
Urbano vs. Chavez
L-87977, March 19, 1990
FACTS: Petitoners Urbano and
Acapulco filed a criminal case with
the Ombudsman against Secretary
Luis Santos of the Department of
Local Government for alleged
violation of the Anti Graft and
Corrupt
Practices
Act.
OSG
appeared as counsel for the
respondents
as
far
as
the
preliminary investigation of the
case is concerned. Petitioners
sought to enjoin the Solicitor
General and his associate from
acting
as
counsel
for
said
respondents in the course of the
preliminary
investigation
contending that in the event that
the corresponding information is
filed against the respondents with
the
Sandiganbayan
and
a
judgement
of
conviction
is
rendered by the said court, the
appearance of OSG on behalf of
respondents
during
the
preliminary investigation will be in
conflict with its role as appellate
counsel of the People of the
Philippines.
ISSUE: Can a the OSG represent a
public officer in the preliminary
investigation of a criminal action
against him or in a civil action
against him?
HELD: No. There will be a clear
conflict of interest, where the OSG
as counsel for the public official,
defends
the
latter
in
the
preliminary investigation stage of
a criminal case, and where the
same office, as appellate counsel of

the People of the Philippines,


represents the prosecution when
the case is brought on appeal. This
anomalous situation could not have
been contemplated and allowed by
law.
Placer vs. Villanueva
L-60349, December 29, 1983
FACTS: Petitioners filed in the City
Court
of
Butuan
City
an
information for the issuance of
warrant
of
arrest
with
the
certification
that
preliminary
investigation had been conducted
and that there exist a probable
cause. Upon receipt of said
information,
respondent
judge
ordered an hearing to determine
whether the issuance of the same
is proper. After said hearing,
respondent
Judge
required
petitioners to submit to the court
the affidavits of the prosecution
witness and other documentary
evidence
in
support
of
the
information to aid him in the
exercise of his power of judicial
review of the findings of probable
cause by petitioners. However,
petitioners contended that under
PD 77 and 911, they are
authorized
to
determine
the
existence of probable cause in a
preliminary investigation and that
their findings to the existence
thereof constitute sufficient basis
for the issuance of warrants of
arrest by the court, which the
judge is bound to do.
ISSUE: Is a certification by the
fiscal in the information as to the
existence
of
probable
cause
obligates respondent judge to
issue warrant of arrest?

HELD: No. The issuance of a


warrant of arrest is not a mere
ministerial function. It calls for the
exercise of judicial discretion on
the part of the issuing magistrate.
Judge must issue the warrant of
arrest of he is satisfied from the
preliminary
investigation
conducted by him or by the
investigating officer that the
offense complained of has been
committed and that there is
reasonable ground to believe that
the accused has committed it. This
means that a judge must satisfy
himself of the existence of
probable cause before issuing a
warrant or order od arrest. If on
the face of the information the
judge finds no probable cause, he
may
disregard
the
fiscals
certification
and
require
submission of affidavit of witnesses
to aid him in arriving at a
conclusion as to the existence of
probable cause. In the present
case, the judge found inadequate
bases for the determination of
probable cause, which led him to
require the submission of the
complainant and of his witness to
enable the court to determine
whether to dismiss the case
outright or to require further
proceedings.
Abdula vs. Guiani
GR No. 118821, February 18, 2000
FACTS: A complaint for murder
was filed before the ARMMs
Criminal Investigation Command
against petitioners and six other
persons in connection with the
death of one Abdul Dimalen.
Acting on this complaint , the
provincial prosecutor Salick Panda

dismissed the charges on the


finding that there was no probable
cause
for
murder
against
petitioners. However, Prosecutor
Panda recommended the filing of
filing
of
an
information
to
respondents, a certain Kasan
Mama. Thereafter, an information
for murder murder was filed
against Kasan Mama before the
sala of the respondent judge. The
latter ordered the case to be
returned
to
the
provincial
prosecutor
for
further
investigation. It was then assigned
to 2nd assistant prosecutor Enoc
Dimaraw
for
further
reinvestigation. In addition to the
evidence presented during the
initial investigation of the murder
charge, two new affidavits of
witnesses were submitted to
support the charge of murder
against petitioners and the other
respondents
and
thus
recommended the filing of charges
against them with a notation from
the Provincial Prosecutor Panda
stating that he was inhibiting
himself
and
authorising
the
investigating prosecutor to dispose
of the case without his approval.
Respondent Judge then issued a
warrant
for
the
arrest
of
petitioners.
Petitioners alleged that the
information charging them with
murder is null and void because it
was filed without the authority of
the Provincial Prosecutor Panda
and it was signed only by the
investigating prosecutor Dimaraw.
They prayed that the warrant of
arrest issued them be recalled
considering that the respondent
judge did not personally examine

the evidence nor did he call the


complainant and his witness in the
face of their incredible accounts.
ISSUES:
1.
Is
the
second
information for murder valid?
2. Is the warrant of arrest
issued against petitioners valid?
HELD:
1. Yes. A complaint or
information can only be filed if it is
approved or authorised by the
provincial or city fiscal or chief
state
prosecutor.
While
the
Information were not approved by
the Provincial Prosecutor Panda,
the filing of the same even without
his approval was authorised. It
must be stressed that that the
Rules of Court speaks of authority
or approval by the provincial, city
or chief state prosecutor. The
notation made
by Prosecutor
Panda clearly shows that the
Investigating Prosecutor Dimaraw
was authorised to dispose of the
case without his approval. In
issuing the information and in
filing
the
information,
the
investigating
prosecutor
was
acting well within the authority
granted to him by the provincial
prosecutor.
2. No, the warrant of arrest
is void. The Constitution provides
that no search warrant or warrant
of arrest shall be issued except
upon probable cause to be
determined personally by the
judge after examination under oath
or affirmation of the complainant
and the witnessed he may produce.
In the case at bar, respondent
judge admits that he issued the
questioned warrant as there was
no reason for him to doubt the
validity of the certification made
by the Assistant Prosecutor that a

preliminary
investigation
was
conducted and that probable cause
was found to exist as against those
charged in the information filed.
This Statement is an admission
that the respondent judge relied
solely to the certification made by
the fiscal that probable cause
exists as against those charged in
the information and issued the
challenged warrant of arrest on
the sole basis of the prosecutors
findings and recommendations.
Therefore, respondent judge has
abdicated his duty under the
constitution to determine on his
own the issue of the existence of
probable cause prior to issuing
warrant.

PASTRANA SHARMANE MAE


N. T. HASHIM v. MARCELO T.
BONCAN, THE CITY FISCAL OF
MANILA G. R. NO. 47777;
JANUARY 13, 1941
TOPIC:
PRELIMINARY
INVESTIGATION (RULE 112)
FACTS: Petitioner Hashim was
caught red- handed in possession
of counterfeit treasury certificates
of the Commonwealth of the
Philippines. A complaint was filed
against him with the Office of the
City Fiscal and a warrant for his
arrest was issued by Judge Sixto
de la Costa on the strength of the
respondent
Fiscals
sworn
statement that he had conducted a
preliminary investigation and that
he had examined the witnesses
under oath.

Counsel for the petitioner asked


that the warrant of arrest issued in
the case be cancelled and insisted
that the Court conduct the
preliminary
investigation.
Respondent
Fiscal
filed
an
objection on the ground that "there
is no necessity for this Honorable
Court to conduct a preliminary
investigation in this case because
the
substitute
therefore
had
already
been
performed
in
accordance with law by the office
of the fiscal of the City of Manila."
However, it is further contended
that
existing
legislation
authorizing the City Fiscal to
conduct preliminary investigations
should be deemed repealed and
supplanted by the new Rules of
Court. Otherwise, there would be
no uniformity in said Rules as
ordained by the Constitution.
ISSUE:
Whether or not the
existing legislation under which
the
City
Fiscal
conducts
preliminary
investigations
has
been repealed and supplanted by
the New Rules of Court.
HELD: No, the right to a
preliminary
investigation
is
statutory, not constitutional. Its
purpose is to secure the innocent
against hasty, malicious, and
oppressive prosecutions, and to
protect him from open and public
accusation of crime, from the
trouble, expenses and anxiety of a
public trial, and also to protect the
State from useless and expensive
prosecutions. The new Rules were
drafted in the light of the Court's
experience with cases where
preliminary
investigations
had
dragged on for weeks and even
months. The Court had intended to

make a preliminary investigation


as simple and as speedy as is
consistent with the substantial
rights of the accused.
In
this
case,
a
preliminary
investigation
was
already
conducted by the respondent
Fiscal at which evidence was
adduced warranting the filing of an
information against the petitioner.
Upon the strength of the said
preliminary
investigation
and
sworn information, the presiding
Judge issued a warrant for the
arrest of the petitioner. By asking
for the abstract of testimony for no
other purpose than to scrutinize
the
same
evidence
which
convinced the respondent Fiscal
and the presiding Judge that there
was probable ground to proceed
against the petitioner, is, in effect,
to ask for another preliminary
investigation.
PEOPLE OF THE PHILIPPINES
v. ROBERT POCULAN G.R. NOS.
70565-67; NOVEMBER 9, 1988
TOPIC:
PRELIMINARY
INVESTIGATION (RULE 112)
FACTS: Three separate sworn
complaints were filed by Conchita
Rone charging the accused, Robert
Poculan, Municipal Mayor of Rizal,
Zamboanga del Norte, with Rape.
Three
Identical
Information
differing only as to the date, time
and place of commission were
subsequently filed before the Court
of First Instance of Zamboanga del
Norte. In a joint Decision, the Trial
Court convicted the accused.
Hence, this appeal. One of the
contentions of the defense was
that the trial Court fatally erred in
holding that there had been a
preliminary investigation of these

complaints of the complainant and


in proceeding with the trial and
convictions subjects of this appeal
without
such
preliminary
investigation. The defense further
maintained that the rejection of
the second request denied him the
opportunity
to
present
controverting evidence.
ISSUE: Whether or not there had
been a preliminary investigation
HELD: Yes. The records disclose
that the accused was given all the
opportunity
to
submit
countervailing evidence.
When preliminary investigation
has already been conducted, as
where the accused was given the
right to present countervailing
evidence, there is no need for
reinvestigation. The purpose of a
preliminary investigation or a
previous inquiry of some kind,
before an accused person is placed
upon trial is to secure the innocent
against
hasty,
malicious
and
oppressive prosecution, and to
protect him from an open and
public accusation of a crime, from
the trouble, expense and anxiety of
a public trial. This purpose has
been satisfied in these cases.
Meanwhile, the rejection of the
second request did notdeny the
defense the opportunity to present
controverting evidence.What was
rejected was the accused's plea to
be released from detention so he
could go home to secure affidavits
of his witnesses. However, that did
not bar him nor his lawyers from
still presenting counterbalancing
evidence as, in fact, the defense
was still given the opportunity to
do so.

ARULA v ESPINO G.R. No. L28949, June 23, 1969


TITLE:
PRELIMINARY
INVESTIGATION
FACTS: The petitioner Arula was
recruited by one Capt. Teodoro R.
Facelo of the Armed Forces of the
Philippines at Simunul, Sulu, to
undergo training. He, together
with other recruits, was taken to
Corregidor
island,
where
a
shooting incident occurred at
Corregidor, resulting in, among
other things, the infliction of
serious physical injuries upon the
petitioner. Despite his wounds he
succeeded in fleeing Corregidor.
He filed, a criminal complaint with
the city fiscal of Cavite City for
frustrated murder against military
officials. Acting on the criminal
complaint, the city fiscal sent
subpoenas to the accused, advising
them
that
the
preliminary
investigation was set and requiring
them to appear at his office on the
same date and time.
Meanwhile,
the
respondent
General
Espino
directed
the
conduct of a pre-trial investigation
to pinpoint responsibility therefor.
All of the army personnel, except
two, supposedly involved in the
hapless incident had already been
placed under technical arrest and
restricted to camp limits.
Respondent General Espino issued
Special Order 208, appointing a
General court-martial to try the
case against the army personnel
involved.
Charges
and
specifications for violations of
articles of war 94 and 97 were
filed with the general courtmartial. Armed Forces lawyers
moved to dismiss the complaint

filed with the city fiscal of Cavite


upon the ground that the civil
courts had lost jurisdiction over
the case because a court- martial
had been convened.
It is here pertinent to note that
before the petitioner Arula filed his
criminal complaint with the city
fiscal of Cavite, the President had
already ordered an investigation of
the Corregidor incident and the
convening
of
a
court-martial
relative thereto.
ISSUE:
Whether or not courtmartial has acquired jurisdiction
over the offense committed to
petitioner
even
when
no
preliminary
investigation
was
conducted.
HELD: Yes. Failure to conduct a
pre-trial investigation does not
deprive a general court- martial of
jurisdiction. The better accepted
concept of pre-trial investigation is
that it is directory, not mandatory,
and in no way affects the
jurisdiction of a court-martial.
A trial before a general courtmartial convened without any
pretrial investigation under article
of war 71 would of course be
altogether irregular but the courtmartial might nevertheless have
jurisdiction. Significantly, this rule
is similar to the one obtaining in
criminal procedure in the civil
courts to the effect that absence of
preliminary investigation does not
go into the jurisdiction of the court
but merely to the regularity of the
proceedings.
TANDOC V RESULTAN G.R. NO.
59241-44, JULY 5, 1989
TOPIC:
PRELIMINARY
INVESTIGATION

FACTS:
This controversy arose
from a heated altercation and
physical
assaults
amongst
neighbors. Based on the collated
complaints of both parties, in
October 1980, at the house of
PacitaTandoc,
respondents
Cancino,
Arnulfo
Payopay,
ConradoPayopay, Sr. and several
others intruded the sari-sari store
and house of the former and an
altercation ensued. In the middle
of the verbal joust, Arnulfo and
Beda Acosta picked up stones and
hurled them unto Pacita, though,
the projectiles instead hit the
latters helpers who sustained
physical injuries. Four days later,
respondents
filed
complaints
against Tandocs party with the
same fiscals office, however, the
latter found them merely as
belated countercharges meriting
dismissal, except the trespass to
dwelling charged against Pedro
Tandoc.
Displeased
with
the
fiscals resolution, in July 1981,
Payopays party directly lodged
their complaints with City Court
San Carlos (CCSC), where the
criminal cases initiated by the
Tandocs against them are pending.
Subsequently, the CCSC issued
several Orders which are the
subject
of
this
Petition
for
Certiorari, whereby the said court,
after
conducting
preliminary
examination
of
Payopays
complaints
found
reasonable
ground to believe that the offenses
charged may have been committed
by the accused, herein petitioners.
The
Tandocs
moved
for
reconsideration
and
reinvestigation of the complaints by
the city fiscal, insisting that the

latter had already evaluated the


same and found no prima facie
case.
ISSUE: Whether or not the CCSC
had the power and authority to
conduct
anew
a
preliminary
examination of charges, which
already went thru a preliminary
investigation (PI) by the city fiscal
who ordered their dismissal.
HELD: No. The policy objective
for the conduct of a preliminary
investigation is to protect the
accused from the inconvenience,
expense and burden of defending
himself in a formal trial unless
reasonable probability of his guilt
shall have been first ascertained in
a fairly summary proceeding by a
competent officer. Re-investigation
by the Office of the City Fiscal
applies only to instances where a
case is cognizable by the Court of
First Instance but filed with the
City
Court
for
purposes
of
preliminary investigation only and
thereafter dismissed by the latter
on the ground that no prima facie
case exists. However, for cases
cognizable by inferior courts and
filed with the same not only for
purposes
of
preliminary
investigation but for trial on the
merits, the Office of the City Fiscal
has no authority to re- investigate.
In the case at bar, the
offenses
charged
against
petitioners
for
"Trespass
to
Dwelling", "Grave Threats" and
"Physical Injuries" were all within
the jurisdiction of the CCSC.
Under the circumstances, the
complaints could be filed directly
with the City Court which is
empowered
to
conduct
a
preliminary
examination
for

purposes of issuance of warrants


of arrest, and thereafter to
proceed with the trial of the cases
on the merits. The PI proper
conducted by the Office of the City
Fiscal could have been dispensed
with. Neither did the earlier order
of dismissal of the complaints by
the investigating fiscal bar the
filing of said complaints with the
city court on the ground of double
jeopardy.
PILAPIL V SANDIGANBAYAN
G.R. NO. 101978, APRIL 7,
1993
TOPIC:
PRELIMINARY
INVESTIGATION
FACTS:
The Philippine Charity
Sweepstakes
Office
(PCSO)
donated
one
ambulance
(a
Mitsubishi
L-300)
to
the
Municipality of Tigaon, Camarines
Sur.
Petitioner,
who
is
the
Congressman of the 3rd District of
Camarines
Sur,
received
the
ambulance in behalf of the
municipality. However, he did not
deliver the ambulance to said
municipality.
Unaware of the donation, the
Sangguniang
Bayan
of
the
municipality passed a resolution
requesting
PCSO
for
an
ambulance.
Presiding
Justice
Francis Garchitorena, who is from
the said municipality, contacted
the PCSO and learned about the
ambulance previously donated by
the latter to Tigaon through
petitioner.
He
accordingly
informed Mayor Lelis that the
municipality's request cannot be
favorably acted upon in view of the
previous donation but the Mayor
said that no vehicle from the

PCSO. He reiterated the request to


PCSO.
Upon
verification
of
the
whereabouts of the Mitsubishi L300 by the PCSO from the
petitioner, the latter indicated his
willingness
to
return
the
ambulance and requested that said
vehicle be donated instead to the
Municipality of Tinambac, same
province. With such return, the
Municipality of Tigaon, through
Mayor Lelis, finally received a
brand new Besta Kia Ambulance
unit complete with all accessories.
A letter-complaint (Malversation of
Public Property) was filed against
petitioner. In his counter-affidavit,
petitioner denied the imputation of
said offense alleging that he made
personal
representations
with
PAGCOR for the latter to shoulder
the expenses of the installation.
The Ombudsman ruled that there
is no malversation but a violation
of the Anti-Graft and Corrupt
Practices Act (Section 3(e) of
Republic Act. 3019). A warrant of
arrest was, then, issued against
petitioner.
Petitioner filed a motion to quash
on the ground that respondent
Sandiganbayan has no jurisdiction
over his person because the
information was filed without
probable cause since there is
absolutely no proof adduced in the
preliminary investigation of any of
the elements of the crime defined
in Section 3(e) R.A. 3019. The
Sandiganbayan denied the motion.
ISSUE:
Whether or not the
Sandiganbayan committed grave
abuse of discretion in denying the
petitioners motion to quash.

HELD: No. The absence of a


preliminary investigation is not a
ground to quash a complaint or
information under Section 3, Rule
117 of the Rules of Court. The
proper procedure in case of lack of
preliminary investigation is to hold
in abeyance the proceedings upon
such information and the case
remanded to the Office of the
Provincial
Fiscal
or
the
Ombudsman, for that matter, for
him or the Special Prosecutor to
conduct
a
preliminary
investigation.
Likewise,
the
absence
of
preliminary
investigation does not affect the
court's jurisdiction over the case
In the case at bar, there is no old
or new information. Only one
information was filed as a result of
the
preliminary
investigation
conducted by the office of the
Ombudsman.
Even
on
the
assumption that no preliminary
investigation was conducted for
the information filed, petitioner
waived his right thereto for failure
to ask the Sandiganbayan or the
Ombudsman for a new preliminary
investigation.
It is well-settled that the right to a
preliminary investigation is not a
fundamental right and may be
waived expressly or by silence.
Failure of accused to invoke his
right to a preliminary investigation
constituted a waiver of such right.
UY V SANDIGANBAYAN G.R.
NOS. 105965-70, MARCH 20,
2001
TOPIC:
PRELIMINARY
INVESTIGATION
FACTS: Before the Court is the
Motion for Further Clarification of
the Court's
ruling that the

prosecutory
power
of
the
Ombudsman extends only to cases
cognizable by the Sandiganbayan
and that the Ombudsman has no
authority to prosecute cases falling
within the jurisdiction of regular
courts.
ISSUE:
WON Ombudsman can
conduct preliminary investigation
HELD: Yes. The law recognizes
the concurrence of jurisdiction
between
the
Office
of
the
Ombudsman
and
other
investigative
agencies
of
government in the prosecution of
cases cognizable by regular courts.
The Ombudsman is therefore
clothed with authority to conduct
preliminary investigation and to
prosecute
all
criminal
cases
involving
public
officers
and
employees, not only those within
the
jurisdiction
of
the
Sandiganbayan, but those within
the jurisdiction of the regular
courts as well.
The
Office
of
the
Special
Prosecutor is merely a component
of the Office of the Ombudsman
and may only act under the
supervision and control and upon
authority of the Ombudsman. Its
power to conduct preliminary
investigation and to prosecute is
limited to criminal cases within the
jurisdiction of the Sandiganbayan.
It must be clarified that the
authority of the Ombudsman to
prosecute cases involving public
officers and employees before the
regular courts does not conflict
with the power of the regular
prosecutors under the Department
of Justice to control and direct the
prosecution of all criminal actions

under Rule 110 of the Revised


Rules of Criminal Procedure.
HONASAN II V PANEL OF
INVESTIGATING
PROSECUTORS
OF
THE
DEPARTMENT OF JUSICE G.R.
NO. 159747, APRIL 13, 2004
TOPIC:
PRELIMINARY
INVESTIGATION
FACTS: The CIDG-PNP/P Director
EdguardoMatillano
filed
an
affidavit-complaint
with
the
Department of Justice (DOJ) on the
crime of coup d etat committed by
military personnel and Senator
Gregorio Gringo Honasan II,
who occupied Oakwood Hotel.
Prior
to
the
occupation
of
Oakwood, Capt. Gerardo Gambala,
in behalf of the military rebels,
made a public statement aired on
national television, stating their
withdrawal of support to the chain
of command of the Armed Forces
of
the
Philippines
and
the
Government of President Gloria
Macapagal-Arroyo. Willing to risk
their lives to achieve the National
Recovery
Agenda
of
Senator
Honasan, which they believe is the
only program that would solve the
ills of society.
The
Panel
of
Investigating
Prosecutors of the Department of
Justice, in taking cognizance of the
preliminary
investigation
on
charges of coup d'etat against
petitioner
Gregorio
Honasan,
relies on OMB-DOJ Circular No.
95- 001. Consequently, Honasan
questioned
said
OMBDOJ
circular.
ISSUE: Whether or not, the office
of the Ombudsman should deputize
the prosecutors of the DOJ to

conduct
the
preliminary
investigation.
HELD: No. The authority of the
Ombudsman
to
investigate
offenses involving public officers
or employees is not exclusive but is
concurrent with other similarly
authorized
agencies
of
the
government. The
salary
grade
position does not by itself remove
from the DOJ Panel the authority
to investigate the charge of coup
d'etat
which falls under the
Revised Penal Code. Thus, the DOJ
Panel need not be authorized nor
deputized by the Ombudsman to
conduct
the
preliminary
investigation for complaints filed
with it. Thus, there is not even a
need to delegate the conduct of
the preliminary investigation to an
agency, which has the jurisdiction
to do so in the first place.
However, the Ombudsman may
assert its primary jurisdiction at
any stage of the investigation.
OMB-DOJ Joint Circulars no. 95001 is merely an internal circular
between the DOJ and the office of
the
Ombudsman,
Outlining
authority
and
responsibilities
among prosecutors of the DOJ and
of the office of the Ombudsman in
the
conduct
of
preliminary
investigation.
Moreover, the Constitution, the
Ombudsman
Act
of
1989,
Administrative Order No. 8 of the
Office
of
the
Ombudsman,
prevailing jurisprudence, and the
Revised
Rules
on
Criminal
Procedure
all
recognize
and
uphold the concurrent jurisdiction
of the Ombudsman and the DOJ to
conduct preliminary investigation

on charges filed against public


officers and employees.
PEOPLE OF THE PHILIPPINES,
v. JESSIE B. CASTILLO AND
FELICITO R. MEJIA, G.R. NO.
171188 JANUARY 19, 2009
TOPIC:
PRELIMINARY
INVESTIGATION
FACTS: Complainant Cesar Sarino
in one of the registered owners of
a piece of land. The property is
leased to Pepito B. Aquino and
Adriano G. Samoy who are
subleasing
it
to
several
stallholders. Respondent Felicito
R. Mejia, Municipal Building
Official
of
Bacoor,
sent
to
stallholders Notices of Violation of
the National Building Code on the
grounds that the structures they
were occupying were erected
without building permits and
occupied by them without the
necessary
certificates
of
occupancy
having
been
first
secured. The task force from the
Bacoor Municipal Hall effected the
closure if the stalls. Lessees
Aquino and Samoy filed before the
Office of the Ombudsman a
complaint
against
respondents
Castillo, Mejia and two other
municipal officials for violation of
Anti-Graft and Corrupt Practices
Act. The Office of Ombudsman
dismissed the complaint ruling
that the respondent local officials
acted in good faith in effecting the
closure of the stalls. Sarino filed a
case againt Castillo and Mejia
before
the
Office
of
the
Ombudsman
charging
them
criminally for violation of Section
3(e) and (f) of R.A. No. 3019 and
R.A. No. 6713 and administratively

for oppression, grave misconduct


and for committing acts contrary
to law. Ombudsman dismissed the
administrative complaint for being
moot and academic due to
Castillos re-election as Mayor
because the act complained of
happened more than one year ago
before the complaint was filed.
In a resolution, the Sandiganbayan
declared that probable cause
exists against respondents for
violation
of
Section
3(e).
Respondents
voluntarily
surrendered to the Sandiganbayan
and posted their respective bonds
for
their
provisional
liberty.
Respondents
moved
for
the
reinvestigation of the case and
after which the Office of the
Special Prosecutor, filed a Motion
for Leave to Admit Attached
Amended
Information.
The
respondents then filed a Comment
thereon with the Motion for
Judicial Determination of Probable
Cause.
The
Sandiganbayan
reversed
its
resolution
and
dismissed
the
case.
The
Sandiganbayan likewise set aside
the arrest warrants it previously
issued. It held that the instant
criminal case is a mere rehash of
the previously dismissed criminal
case.
ISSUE:
Whether
the
Sandiganbayan
erred
in
overturning
the
Ombudsmans
determination of probable cause
resulting in the dismissal of the
case of respondents.
HELD: Yes. In this case, there is
no question that both the original
and amended information were
valid on their face because they
complied with Section 6 Rule 110

of the Rules of Court. Hence, as


the amended Information was valid
on its face and there is no manifest
error or arbitrariness on the part
of
the
Ombudsman.
The
Sandiganbayan erred in making an
executive
determination
of
probable cause when it overturned
the
Ombudsmans
own
determination. The Sandiganbayan
and all courts should always
remember the judiciarys standing
policy on non-interference in the
Office
of
the
Ombudsmans
exercise of its constitutionally
mandated powers. Sandiganbayan
cannot
make
an
executive
determination of probable cause
by overturning the Ombudsmans
own determination. It could only
revoke a previously issued arrest
warrants
and
require
the
Ombudsman to submit additional
evidence for the purpose of issuing
the arrest warrants based on the
amended Information. Moreover, it
was clearly premature on the part
of the Sandiganbayan to make a
determinative finding prior to the
parties
presentation
of
their
respective evidence that there was
no bad faith and manifest partiality
on the respondents part and undue
injury on the part of the
complainant. Also, it would be
unfair to expect the prosecution to
present all the evidence needed to
secure the conviction of the
accused upon the filing of the
information against the latter. The
reason is found in the nature and
objective
of
a
preliminary
investigation.
PEOPLE OF THE PHILIPPINES,
VS.HONORABLE ENRIQUE B.

INTING G.R. NO. 88919 JULY


25, 1990
TOPIC:
PRELIMINARY
INVESTIGATION
FACTS: Mrs. Editha Barba filed a
letter- complaint against OICMayor Dominador Regalado with
COMELEC
for
allegedly
transferring her from the Office of
the Municipal Mayor to a very
remote barangay and without
obtaining prior permission or
clearance from COMELEC as
required
by
law.
COMELEC
directed Atty. Gerardo Lituanas to
conduct preliminary investigation
pursuant to COMELEC Resolution
No. 1572.
After a preliminary
investigation of Barbas case,
Atty .Lituanas found a prima facie
case thus he filed with respondent
court a criminal case against the
OIC- Mayor. The trial court set
aside its order of warrant of arrest
on the ground that Atty. Lituanas is
not
authorize
to
determine
probable cause.The court stated
that it "will give due course to the
information filed in this case if the
same has the written approval of
the Provincial Fiscalafter which
the prosecution of the case shall
be under the supervision and
control of the latter."
In another order dated November
22, 1988, the court gave Atty.
Lituanas fifteen (15) days from
receipt to file another information
charging the same offense with the
written approval of the Provincial
Fiscal. However Atty. Lituanas
failed to comply with the order.
Hence,
in
an
order
dated
December 8, 1988, the trial court
quashed the information. A motion
for reconsideration was denied.

ISSUE: Whether or not the


approval of the Provincial Fiscal is
necessary before the information
filed by the Provincial Election
Supervisor may be give due course
by the trial court.
HELD:
No.
First,
the
determination of probable cause is
a function of the Judge. It is not for
the Provincial Fiscal or Prosecutor
nor for the Election Supervisor to
ascertain. Only the Judge and the
Judge
alone
makes
this
determination.
Second,
the
preliminary inquiry made by a
Prosecutor does not bind the
Judge. It merely assists him to
make
the
determination
of
probable cause. The Judge does
not have to follow what the
Prosecutor presents to him. By
itself, the Prosecutor's certification
of probable cause is ineffectual.
And third, the determination of
probable cause for the warrant of
arrest is made by the Judge. The
preliminary investigation properwhether or not there is reasonable
ground to believe that the accused
is guilty of the offense charged
and, therefore, whether or not he
should be subjected to the
expense,
rigors
and
embarrassment of trial is the
function of the Prosecutor.
The 1988 Amendments to the 1985
Rules on Criminal Procedure, did
not restore that authority to
Judges of RTC; said amendments
did not in fact deal at all with the
officers or courts having authority
to
conduct
preliminary
investigations. While an RTC Judge
may no longer conduct preliminary
investigations to ascertain whether
there is sufficient ground for the

filing of a criminal complaint or


information,
he
retains
the
authority, when such a pleading is
filed with his court, to determine
whether there is probable
cause justifying the issuance of a
warrant of arrest. COMELEC is
empowered to conduct preliminary
investigations in cases involving
election offenses for the purpose of
helping the Judge determine
probable cause and for filing an
information in court. This power is
exclusive with COMELEC. Hence,
the Provincial Fiscal, as such,
assumes no role in the prosecution
of election offenses. If the Fiscal or
Prosecutor files an information
charging an election offense or
prosecutes a violation of election
law, it is because he has been
deputized by the COMELEC. He
does not do so under the sole
authority of his office. (People v.
Basilla, et al., G.R. Nos. 83938-40,
November 6, 1989).
It is only after a preliminary
examination conducted by the
COMELEC through its officials or
its deputies that section 2, Article
III of the 1987 Constitution comes
in. This is so, because, when the
application for a warrant of arrest
is made and the information is filed
with the court, the judge will then
determine whether or not a
probable cause exists for the
issuance of a warrant of arrest.
PAUL G. ROBERTS, JR., ET AL.
v. CA G.R. NO. 113930, 5
MARCH 1996
TOPIC: ARREST
FACTS:
Petitioners, who are
corporate officers and members of
the Board of Pepsi Cola Products

Phils., Inc. were prosecuted in


connection
with
the
Pepsi
Number Fever promotion by
handlers
of
the
supposedly
winning 349 Pepsi crowns. Of
the four cases filed against the
petitioners, probable cause was
found
by
the
investigating
prosecutor only for the crime of
estafa, but not for the other
alleged offenses. On 12 April 1993,
the information was filed with the
trial
court
without
anything
accompanying it. A copy of the
investigating prosecutors Joint
Resolution was forwarded to and
received by the trial court only on
22 April 1993. However, no
affidavits
of
the
witnesses,
transcripts of stenographic notes
of the proceedings during the
preliminary investigation, or other
documents submitted in the course
thereof were found in the records
of the case as of 19 May 1993. On
15 April 1993, petitioners Roberts,
et al. filed a petition for review to
the Department of Justice seeking
the reversal of the finding of
probable
cause
by
the
investigating prosecutor. They also
moved for the suspension of the
proceedings and the holding in
abeyance of the issuance of
warrants of arrest against them.
Meanwhile, the public prosecutor
also
moved
to
defer
the
arraignment
of
the
accusedappellants
pending
the
final
disposition of the appeal to the
Department of Justice.
On 17 May 1993, respondent
Judge
Asuncion
issued
the
challenged order (1) denying, on
the basis of Crespo v. Mogul, the
foregoing motions respectively

filed by the petitioners and the


public prosecutor, and directing
the issuance of the warrants of
arrest after June 1993 and
setting the arraignment on 28 June
1993. In part, respondent judge
stated in his order that since the
case is already pending in this
Court for trial, following whatever
opinion the Secretary of Justice
may have on the matter would
undermine the independence and
integrity his court. To justify his
order, he quoted the ruling of the
Supreme Court in Crespo, which
stated: In order therefor to avoid
such a situation whereby the
opinion of the Secretary of Justice
who reviewed the action of the
fiscal may be disregarded by the
trial court, the Secretary of Justice
should, as far as practicable,
refrain from entertaining a petition
for review or appeal from the
action of the fiscal, when the
complaint or information has
already been filed in Court. The
matter should be left entirely for
the determination of the Court.
Petitioners went to the Court of
Appeals (CA), arguing that the
respondent judge had not the
slightest
basis
at
all
for
determining probable cause when
he ordered the issuance of
warrants of arrest. After finding
that a copy of the public
prosecutors Joint Resolution had
in fact been forwarded to, and
received by, the trial court on 22
April
1993,
the
CA
denied
petitioners application for writ of
preliminary injunction. The CA
ruled that the Joint Resolution
was sufficient in itself to have
been relied upon by respondent

Judge in convincing himself that


probable cause indeed exists for
the
purpose
of
issuing
the
corresponding warrants of arrest
and that the mere silence of the
records or the absence of any
express
declaration
in
the
questioned order as to the basis of
such finding does not give rise to
an adverse inference, for the
respondent Judge enjoys in his
favor the presumption of regularity
in the performance of his official
duty. Roberts, et al. sought
reconsideration, but meanwhile,
the DOJ affirmed the finding of
probable
cause
by
the
investigating prosecutor. The CA
therefore dismissed the petition
for mootness.
ISSUES:
1.
Did Judge Asuncion commit
grave abuse of discretion in
denying, on the basis of Crespo v.
Mogul, the motions to suspend
proceedings and hold in abeyance
the issuance of warrants of arrest
and to defer arraignment until
after the petition for review filed
with the DOJ shall have been
resolved?
2.
Did Judge Asuncion commit
grave abuse of discretion in
ordering the issuance of warrants
of arrest without examining the
records
of
the
preliminary
investigation? 3.
May the
Supreme Court determine in this
[sic] proceedings the existence of
probable cause either for the
issuance of warrants of arrest
against the petitioners or for their
prosecution for the crime of
estafa?
HELD: [The Court, in a 7-5-2 vote,
GRANTED the petition. It SET

ASIDE the decision and resolution


of the CA, the resolutions of the
DOJ 349 Committee, and the order
of respondent judge.] 1.
YES,
Judge Asuncion committed grave
abuse of discretion in denying, on
the basis of Crespo v. Mogul, the
motions to suspend proceedings
and hold in abeyance the issuance
of warrants of arrest and to defer
arraignment until after the petition
for review filed with the DOJ shall
have been resolved.
There is nothing in Crespo v.
Mogul which bars the DOJ from
taking cognizance of an appeal, by
way of a petition for review, by an
accused in a criminal case from an
unfavorable
ruling
of
the
investigating prosecutor. It merely
advised the DOJ to, as far as
practicable,
refrain
from
entertaining a petition for review
or appeal from the action of the
fiscal, when the complaint or
information has already been filed
in Court.
Whether the DOJ would affirm or
reverse
the
challenged
Joint
Resolution is still a matter of
guesswork. Accordingly, it was
premature for respondent Judge
Asuncion to deny the motions to
suspend proceedings and to defer
arraignment on the following
grounds: This case is already
pending in this Court for trial. To
follow
whatever
opinion
the
Secretary of Justice may have on
the matter would undermine the
independence and integrity of this
Court. This Court is still capable of
administering justice. The real and
ultimate test of the independence
and integrity of this court is not
the filing of the aforementioned

motions [to suspend proceedings


and issuance of warrants of arrest
and to defer arraignment] at that
stage but the filing of a motion to
dismiss or to withdraw the
information on the basis of a
resolution of the petition for
review
reversing
the
Joint
Resolution of the investigating
prosecutor.
However,
once
a
motion to dismiss or withdraw the
information is filed the trial judge
may grant or deny it, not out of
subservience to the Secretary of
Justice, but in faithful exercise of
judicial prerogative.
2.
YES, Judge Asuncion
committed
grave
abuse
of
discretion in ordering the issuance
of warrants of arrest without
examining the records of the
preliminary
investigation.
The
teachings then of Soliven, Inting,
Lim, Allado, and Webb reject the
proposition that the investigating
prosecutors certification in an
information or his resolution which
is made the basis for the filing of
the information, or both, would
suffice
in
the
judicial
determination of probable cause
for the issuance of a warrant of
arrest. In Webb, this Court
assumed that since the respondent
Judges had before them not only
the 26-page resolution of the
investigating panel but also the
affidavits
of
the
prosecution
witnesses and even the counteraffidavits of the respondents, they
(judges) made personal evaluation
of the evidence attached to the
records of the case.
In this case, nothing accompanied
the information upon its filing on
12 April 1993 with the trial court.

A copy of the Joint Resolution was


forwarded to, and received by, the
trial court only on 22 April 1993.
And
as
revealed
by
the
certification of respondent judges
clerk of court, no affidavits of the
witnesses,
transcripts
of
stenographic
notes
of
the
proceedings
during
the
preliminary investigation, or other
documents submitted in the course
thereof were found in the records
of this case as of 19 May 1993.
Clearly, when respondent Judge
Asuncion issued the assailed order
of 17 May 1993 directing, among
other things, the issuance of
warrants of arrest, he had only the
information, amended information,
and Joint Resolution as bases
thereof. He did not have the
records or evidence supporting the
prosecutor's finding of probable
cause. And strangely enough, he
made no specific finding of
probable cause; he merely directed
the issuance of warrants of arrest
after June 21, 1993. It may,
however, be argued that the
directive presupposes a finding of
probable
cause.
But
then
compliance with a constitutional
requirement for the protection of
individual liberty cannot be left to
presupposition, conjecture, or even
convincing logic. 3.
NO, the
Supreme
Court
MAY
NOT
determine in this [sic] proceedings
the existence of probable cause
either for the issuance of warrants
of arrest against the petitioners or
for their prosecution for the crime
of
estafa.
Ordinarily,
the
determination of probable cause is
not lodged with this Court. Its duty
in an appropriate case is confined

to the issue of whether the


executive
or
judicial
determination, as the case may be,
of probable cause was done
without or in excess of jurisdiction
or with grave abuse of discretion
amounting to want of jurisdiction.
This is consistent with the general
rule that criminal prosecutions
may not be restrained or stayed by
injunction, preliminary or final.
There are, however, exceptions to
the foregoing rule. But the Court
refused to reevaluate the evidence
to determine if indeed there is
probable cause for the issuance of
warrants of arrest in this case. For
the respondent judge did not, in
fact, find that probable cause
exists, and if he did he did not
have the basis therefor. Moreover,
the records of the preliminary
investigation in this case are not
with the Court. They were
forwarded by the Office of the City
Prosecutor of Quezon City to the
DOJ in compliance with the latter's
1st Indorsement of 21 April 1993.
The trial court and the DOJ must
be required to perform their duty.
RULE 113 ARREST
CUSTOMHOUSE, PASAY CITY,
v. DISTRICT, STATIONED AT
PASIG, RIZAL, AND CESAR T.
MAKAPUGAY, (CONSOLIDATED
CASES) G.R. NO. L-34038 JUNE
18, 1976
TOPIC:
PRELIMINARY
INVESTIGATION
FACTS:
Petitioner Collector of
Customs, Salvador T. Mascardo
filed against Cesar T. Makapugay, a
letter complaint with respondent

Judge of the Circuit Criminal Court


for violation of: (a) Section 174 of
the National Internal Revenue
Code, (b) Cantral Bank Circular
No. 265, and (c) Section 3601 and
3602 of the Republic Act No. 1937.
Respondent
Judge
assumed
jurisdiction to conduct and did
conduct
the
preliminary
investigation
and
issued
the
challenged order dismissing "the
case with prejudice and ordering
the return to private respondent
the amount of P2,280.00, his
passport No. Ag-2456 FA - No.
B103813, and one (1) box of airconditioning evaporator only, as
well as the forfeiture of forty (40)
cartons of untaxed blue seal Salem
cigarettes and five (5) bottles of
Johnny Walker Scotch Whiskey" (p.
13, rec.). Petitioner Collector of
Customs refused to obey the order
due to the "prior institution of
seizure proceedings thereon." The
refusal
prompted
respondent
Makapugay to file a complaint for
"Open Disobedience" under Article
231 of the Revised Penal Code,
before the City Fiscal of Pasay City.
ISSUE: Whether or not respondent
Circuit Criminal Courts possess
the power to conduct preliminary
investigation.
HELD: Yes. Circuit
Criminal
Courts are of limited jurisdiction,
only because they cannot try and
decide all criminal cases falling
under the jurisdiction of the Courts
of First Instance as courts of
general jurisdiction. They can only
take cognizance of cases expressly
specified in Section 1 of Republic
Act No. 5179, as amended by
Presidential Decree No. 126.
Nevertheless, they have the same

powers and functions as those


conferred upon regular Courts of
First
Instance
necessary
to
effectively exercise such special
and limited jurisdiction. The twofold purpose for the creation of
Circuit Criminal Courts is to
alleviate the burden of the regular
Courts of First Instance and to
accelerate the disposition of the
cases therein as well as stem the
tide of criminality. As such, it is
only logical that the authority
vested in the judges of the Courts
of First Instance is likewise
conferred on Circuit Criminal
Courts. Otherwise, the Courts of
First Instance would still be
carrying the burden of conducting
preliminary investigations in those
cases where Circuit Criminal
Courts have jurisdiction which
would
delay
the
trial
and
disposition
of
criminal
cases
pending before such Courts of
First Instance.The Congress, in
enacting Republic Act No. 5179
clearly intended, by Sections 3 and
6 thereof, to clothe the Circuit
Criminal Court with all the powers
vested in regular Courts of First
Instance including the authority to
conduct preliminary examinations
and investigations.
ROBERTO UMIL v. FIDEL v.
RAMOS
G.R.
NO.
81567
OCTOBER 3, 1991
TOPIC: ARREST
FACTS: This consolidated case of
eight (8) petitions for habeas
corpus assails the validity of the
arrest and searches made by the
military on the petitioners. The
arrest relied on the confidential
information that the authorities

received. All cases were charged


with subversion for being a
member of the New Peoples Army,
except for one where inciting to
sedition (Espiritu v. Lim) was
charged. Petitioners contend that
their detention was unlawful since
they were arrested with no
warrant
and
no
preliminary
investigation was conducted.
ISSUE: Whether or not the arrests
of the petitioners are all valid and
legal.
HELD:
Umil v. Ramos

They
were
arrested
in
connection with the killing of 2
capcom soldiers

Dural
was
captured
and
identified 1 day after the
incident because he needed
medical care

FEB 6, 1988 petition for


habeas corpus was filed with
the court on behalf of Umil,
Dural and Villanueva

FEB 26, 1988 Umil and


Villanueva posted bail before
RTC Pasay where charges for
violation of the Anti- Subversion
Act, they were released. Hence,
petition for Habeas Corpus of
Umil and Villanueva is already
moot and academic.

Although
Dural
was
not
arrested DURING the shooting,
he was arrested the day after.
BUT court said the Dural was

arrested for being a member of


the NPA an outlawed subversive
organization. Since Subversion
is a continuing offense, the
arrest
without
warrant
is
justified. Furthermore, evidence
shows that the case against
Dural was tried in court
wherein
they
were
found
GUILTY and now serving the
sentence, hence, petition for
habeas corpus cannot lie.

Roque and Buenaobra were


charged with violation of antisubversion act

BUENAOBRA - At the hearing


manifested his desire to stay in
Crame = Moot and academic
ROQUES
WARRANTLESS
ARREST IS JUSTIFIED. As
officers and/or members of the
NUFC-CPP, their arrest, without
warrant, was justified for the
same reasons earlier stated visa-vis
Rolando
Dural,
additionally justified as she was,
at the time of apprehension, in
possession
of
ammunitions
without license to possess them.

Roque v. De Villa

They were found through a


snitch (Ramos) who pointed to a
certain house occupied by
Constantino used as a
safehouse of the NUFC of the
CPP- NPA where Amelia Roque
and Wilfredo Buenaobra were
apprehended

Buenaobra admitted that he


was an NPA courier and he had
with him letters to Constantino
and other members

Roque was a member of the


National
United
Front
Commission,
in
charge
of
finance,
and
admitted
ownership
of
subversive
documents found in the house
of her sister in Caloocan City

She was also in possession of


ammunition and a frag grenade
with no permit

Anonuevo v. Ramos

Domingo Anonuevo and Ramon


Casiple are admittedly members
of the standing committee of
the NUFC apprehended in the
house of Constatino They had
a bag containing subversive
materials, and both carried
firearms and ammunition (no
permit to carry)

DURING COURSE OF HOUSE


SURVEILANCE; the military
agents noticed bulging objects
on their waist lines, hence, they
frisked them and found them to
be loaded guns.

SC heldAnonuevo and Casiple


were
carrying
unlicensed
firearms
and
ammo
when

arrested.
Hence,
warrantless arrest

lawful

There is also no merit in the


contention that the informations
filed against them are null and
void for want of a preliminary
investigation.
The filing of an information,
without
a
preliminary
investigation having been first
conducted, is sanctioned by the
Rules. Sec. 7, Rule 112 of the
ROC
Anonuevo and Casiple, refused
to sign a waiver of the
provisions of RPC Art 125
(Delay in delivery of detained
persons to the proper judicial
authorities) PETITIONERS DID
NOT ASK FOR A PRELIMINARY
INVESTIGATION AFTER INFOS
FIELD AGAINST THEM IN
COURT

for a .45 cal. pistol were found


in the car of Vicky Ocaya. They
were brought to the PC
Headquarters for investigation.

Ocaya could not produce any


permit
to
possess
the
ammunition information filed
for violation of PD 1866 in RTC
Pasig. Danny Rivera, on the
other hand, was released from
custody.

SC held that Ocaya was


arrested in flagranti delicto so
that her arrest without a
warrant is justified.

No preliminary investigation
was conducted because she was
arrested without a warrant and
she refused to waive the
provisions of Article 125 RPC.
Ocaya, Anonuevo, Casiple and
Roque claim that the firearms,
ammo and subversive docs were
all planted illegal arrest.

SC says NO EVIDENCE and no


attributable evil motive or illwill on part of arresting officers.

SOLGEN says arrests were part


of in- depth surveillance of NPA
safehouses pointed to by former
NPA members.

Ocaya v. Aguirre

12 May 1988, agents of the PC


Intelligence and Investigation of
the Rizal PC-INP Command,
armed with a search warrant
conducted a search of a house
in Marikina occupied by Benito
Tiamson, head of the CPP-NPA.
During search Ocaya arrived in
a car driven by Danny Rivera.
Subversive
documents
and
several rounds of ammunition

Espiritu v. Lim

Petitioner
Secretary

is

the
of

General
the

PinagkaisahangSamahanngTsup
er at Operators Nationwide
(PISTON), an association of
drivers and operators of public
service
vehicles
in
the
Philippines, organized for their
mutual aid and protection.

He claims that one day while


sleeping, a group of men
wanted to hire his jeepney, but
upon
surprise
these
men
arrested him without warrant of
arrest. He was detained at
Police
Station
No.
8,
Blumentritt.

The
respondents
claim,
however, that the detention was
justified
in
view
of
the
information filed against him
before the RTC of Manila,
charging him with inciting to
sedition.

Since
the
arrest
of
the
petitioner without a warrant
was in accordance with the
provisions of Rule 113, Sec. 5(b)
of the Rules of Court and that
the petitioner is detained by
virtue of a valid information
filed with the competent court,
he may not be released on
habeas corpus.

Nazareno
Commander

v.

Station

Nazareno was pointed by one


Ramil Regal, who testified that

the former was his companion


in the killing of one Romulo
Bunye II. Therefore, police
officers without a warrant
picked
up
Nazareno
and
brought
him
to
police
headquarters for questioning.
Now, a petition for habeas
corpus was filed by Nazareno,
which was denied by the RTC
hearing the case.

At the conclusion of the


hearing, or on 1 February 1989,
the Presiding Judge of the
Regional Trial Court of Bian,
Laguna issued a resolution
denying the petition for habeas
corpus, it appearing that the
said NarcisoNazareno is in the
custody of the respondents by
reason of an information filed
against him with the Regional
Trial Court of Makati, Metro
Manila
which
had
taken
cognizance of said case and
had, in fact, denied the motion
for
bail
filed
by
said
NarcisoNazareno (presumably
because of the strength of the
evidence against him).

As a general rule, no peace officer


or person has the power or
authority to arrest anyone without
a warrant of arrest, except in those
cases express authorized by law.
Thus, when the arrest is lawful,
writ of habeas corpus as a speedy
and effective remedy to relieve
persons from unlawful restraint is
unavailing. All arrests herein

stated were justified warrantless


arrests based on the attending
circumstances of each case and
because there is legal and valid
reasons as to why they are
arrested and detained the writ of
habeas corpus cannot be granted
in their favor. The rule is that if a
person alleged to be restrained of
his liberty is in the custody of an
officer under process issued by a
court judge, and that the court or
judge had jurisdiction to issue the
process or make the order, of if
such person is charged before any
court, the writ of habeas corpus
will not be allowed.
MALACAT
v.
COURT
OF
APPEALS G.R. NO. 123595,
DECEMBER 12, 1997
TOPIC: ARREST (RULE 113)
FACTS:
Plaza
Miranda
is
historically notorious for being a
favorite bombsite, but it is not an
excuse in violating the rights of the
accused. On or about August 27,
1990 in Plaza Miranda, Quiapo,
Manila, a certain Sammy Malacat
was apprehended by a police
officer named Rodolfo Yu for the
allegedly carrying an unlicensed
hand grenade.
Police officer Yu narrated that two
days prior to the arrest, they
received information of a plot to
bomb Plaza Miranda, and while on
patrol, they observed a group of
Muslims,
including
Malacat
carrying a hand grenade. They
chased the group but failed to
apprehend anyone. Two days later,
Police officer Yu while on patrol
with other officers spotted Malacat
standing at the corner near
Mercury Drug, Quiapo, Manila

with a hand grenade tucked to his


waist. They searched him and
arrested him for allegedly carrying
a hand grenade. During custodial
investigation, Malacat admitted
the commission of a crime and
willingly answered the questions
asked without the presence of a
counsel.
During the trial, petitioner
Malacat claimed that he was
threatened by the police officers to
admit the crime and that the
grenade admitted to evidence was
his first time to see it.The RTC
convicted Malacat and the CA
upheld the decision.
ISSUES: Whether or not there
was a valid warrantless search and
a valid warrantless arrest
HELD: The Court ruled to acquit
Sammy Malacat on the grounds of
lack of jurisdiction, for violating
his rights as an accused, and
violating the chain of custody rule.
As for violating his rights as an
accused, Malacat was denied the
right to remain silent and counsel
during custodial investigation. The
court declared as inadmissible the
statements and admission made by
Malacat absent the written waiver
of his rights to remain silent and
counsel.
Also, the court declared that the
warrantless
search
and
warrantless arrest conducted by
the police were invalid since they
failed to prove that the search and
arrest
fell
among
the
circumstances allowed by law.
Section 5, rule 113 of the Rules of
Court provide that there is a valid
warrantless arrest when: (1) the
accused is caught in flagrante
delicto
(2)
accused
is

apprehended during hot pursuit


(3) or when he is a prisoner
escapee. The court also stated that
warrantless searches are limited to
the following circumstances: (1)
custom searches, (2) search of
moving vehicles, (3) seizure of
evidence in plain view, (4) consent
searches, (5) search incidental to a
lawful arrest, (6) stop and frisk.
The court said that Malacat was
merely standing at a corner when
the police started searching him
and arresting him which does not
qualify
under
any
of
the
circumstances permitted by law.
Lastly, the court also said that the
grenade was not properly recorded
and managed when it was acquired
by the police, it being a violation of
the chain of custody rule. Hence,
the
petition
was
granted,
acquitting the accused. For failure
to prove that the warrantless
arrest falls under the exception
provided for by Section 5, Rule 113
of the Rules of Court and all under
instances where warrantless arrest
is allowed, the accused may be
acquitted

SAGAUINIT FERDIMAR
PEOPLE OF THE PHILIPPINES
v. MARI MUSA
G.R. NO. 96177, JANUARY 27,
1993
TOPIC: ARREST (RULE 113)
FACTS:

A civilian informer reported


to
the
Narcotics
Command
(NARCOM)
of
Calarian,
Zamboanga City that a certain
Mari
Musa
of
Sutterville,
Zamboanga City was involved in
the sale of marijuana within the
said place. T/Sgt. Jesus Belarga,
team
leader
of
NARCOM,
instructed Sgt. Amado Ani to
conduct a surveillance and a test
buy against the suspect to confirm
the
information.
The
civilian
informer led Sgt. Ani to the house
of Musa and particularly described
his appearance. Sgt. Ani then
pretended to purchase marijuana
which he was able to accomplish
and reported to his superior. The
following day, a buy-bust operation
was planned and executed. Upon
arriving at the location, Sgt. Ani
proceeded inside the house of
Musa while the rest of the team
strategically positioned themselves
outside the house. The wife and
child of Musa, along with a
manicurist were also inside the
house
where
operation
was
done.T/Sgt. Belarga from his
position was able to see what was
going on inside the house. When
the transaction was made, Sgt. Ani
upon receiving the marijuana gave
Musa the marked money of P20.00
and went outside giving the signal
to arrest Musa. The agents entered
Musas home and apprehended
him but the marked money was
gone since he gave to his wife who
had escaped. The agents while
searching for the marked money
entered the kitchen and found a
red plastic bag. They asked Musa
what was in it but to no avail.

They opened it and found more


marijuana.
During
the
trial,
the
prosecution claimed that the
pieces of evidence were validly
acquired and documented though
without a warrant since the
seizure was incidental to a lawful
arrest and were in plain view of
the agents.
Musa, on his defense claimed
that he did not know Sgt. Ani, no
search warrant was presented
upon entry to the house, he was
threatened to admit the crime, and
that he could not have committed
such a crime in the presence of his
wife and son.
The RTC convicted the
accused
Musa
affirming
the
validity of the evidence acquired
absent a search warrant.
ISSUE:
Whether or not the evidence
acquired by the agents are
admissible in court absent a search
warrant and that there was a
lawful arrest.
HELD:
The
court
affirmed
the
decision of the RTC in convicting
Musa but declared that the
evidence found in the kitchen is
not admissible since it was not
acquired in plain view of the
agents or was it incidental to the
arrest. The court explained that it
was not in plain view since the
agents had to enter and search the
kitchen for the reason of finding

the marked money and that it was


wrapped in plastic but was opened
without consent or warrant. But
due to the fact that the agents
acquired separate evidence during
the buy bust operation which was
properly recorded and admitted,
the conviction of the accused was
upheld.
PEOPLE OF THE PHILIPPINES
v. RUBEN BURGOS
G.R. NO. L-68955, SEPTEMBER
4, 1986
TOPIC: ARREST (RULE 113)
FACTS:
The accused Ruben Burgos
was convicted by the Regional
Trial Court for the crime of illegal
possession of firearms. On May 12,
1982 at Digos, Davao Del Sur, a
certain Cesar Masamlok personally
and voluntarily surrendered to the
authorities at the Constabulary
Headquarters, admitting he was
forcibly recruited by the accused
Ruben Burgos as a member of the
New Peoples Army, while pointing
a gun at him threating his life if he
refused. He was also asked to
make monthly contributions from
his harvest and 1 peso.
Upon
receipt
of
the
information, Captain Bargio and
his men along with Pat. Bioco and
Sgt. Bucanlan went to the home of
the accused and questioned him
regarding the possession of a gun.
The accused Burgos, who at that
time was cultivating his farm,
denied the allegation. But when his

wife was questioned, she pointed


the location of the gun which was
buried to the ground. Pat. Bioco
then verified the location and dug
the ground finding a caliber .38
revolver along with subversive
documents on communism.

and he never consented to the


search or waived his right. Hence,
the evidence against him was a
violation of the accuseds right
against self-incrimination and to
unreasonable
searches
and
seizures.

Petitioner was then arrested


and was tried and convicted by the
Trial Court saying there was a
valid warrantless arrest when he
was found to have in his
possession the firearm which his
wife surrendered.
ISSUES:

Lastly,
Burgos
was
not
informed of his right to remain
silent, to counsel, or to waive his
right
against
unreasonable
searches and seizures. Hence, the
Supreme Court reversed the
decision of the RTC and acquitted
Burgos.

Whether or not conviction is


rightful absent a warrant to search
the place for evidence, a warrant
of arrest and without informing the
accused of his rights
HELD:
The Supreme Court reversed
the conviction the RTC saying that,
though
subversion
through
unlawful means is of great
importance, the right of the
accused should never be forsaken.
In the first place, the accused
when he was question by the
constabulary was neither in the act
of committing a crime nor in hot
pursuit
since
he
was
busy
cultivating his land. A warrant of
arrest is therefore necessary to
effectively apprehend him.
As for the search conducted,
the digging conducted by the
authorities
in
acquiring
the
evidence was a form of fishing for
evidence without a valid search
warrant. He also did not admit in
owning the firearm they located

DORIS TERESA HO v. PEOPLE


OF THE PHILIPPINES AND
SANDIGANBAYAN
G.R. NO. 106632, OCTOBER 9,
1997
TOPIC: ARREST (RULE 113)
FACTS:
The
consolidated
case
stemmed from two different suits
filed against herein petitioner
Doris Teresa Ho (President of
National Marine Corporation) and
a certain Rolando Narciso (VicePresident
of
National
Steel
Corporation) for entering into a
contract with Loadstar Shipping
Company
at
a
very
disadvantageous rate compared to
other firms in violation of Sec. 3
(g) of R.A. 3019 (Anti-Graft and
Corrupt Practices Act) which
prohibits a public officer to enter
into any contract or transaction on
behalf of the government wherein
the latter is manifestly and grossly

disadvantageous, whether or not


the officer will benefit.
In the case at bar, Rolando
Narciso and Doris Teresita Ho
were recommended, by the Chief
Prosecutor and Special Prosecutor
respectively, that an information
for violating the said provision be
filed before the Sandiganbayan.
The Sandiganbayan, based
solely
on
resolution,
issued
warrants of arrest against the two
stating that the recommendation
provided by the prosecutors is
already sufficient to justify their
arrest since these were thoroughly
investigated and examined by
them.
Thereafter, petitioner Ho
questioned the issuance of the
Sandiganbayan
saying
it
committed
grave
abuse
of
discretion when it issued the
warrant
of
arrest
without
personally
examining
any
evidence submitted during the
preliminary investigation.
ISSUE:

personally
determine
after
examination
under
oath
or
affirmation of the complainant and
the witnesses he may produce and
particularly describing the place to
be searched and the persons or
things to be seized.
The Supreme Court said that
mere reliance on the resolution or
recommendation of the prosecutor
is not enough to issue a search
warrant or warrant of arrest since
the prosecutor and the judge have
different
roles
in
the
administration of justice. The
prosecutor only verifies if there is
reason to file a case against the
accused or hold him for trial while
the judge has the obligation to
determine probable cause and the
guilt of the accused. It is not
enough that a recommendation
was made by the prosecutor but at
least a personal evaluation of the
evidence by the judge is done to
determine probable cause to
satisfy
the
constitutional
requirement. Hence, the court
declared the warrant of arrest
issued against petitioner as null
and void.

Whether or not a judge may


issue a warrant of arrest by merely
relying on the recommendation of
the prosecutor without personally
examining evidence to determine
probable cause

GILBERT
ZALAMEDA
v.
PEOPLE OF THE PHILIPPINES
G.R. NO. 183656, SEPTEMBER
4, 2009

HELD:

TOPIC: ARREST (RULE 113)

No. The 1987 Constitution


provides that the judge has the
duty to issue a search warrant or
warrant of arrest only upon
probable cause which he must

FACTS:
On
about
5:15am
of
September 14, 2003, a concerned
citizen
contacted
SPO4

MignelitoObeta reporting an ongoing pot session in a house at


Brgy. Tejeros, Makati City. SPO4
Obeta immediately dispatched a
team to respond and verify the
information. The officers including
PO2 De Guzman arrived at the
destination and peeped inside the
house through a slightly opened
door and saw the petitioner Gilbert
Zalameda and his companion
Villaflor, sniffing smoke while
sitting on a bed. PO2 De Guzman
gave the signal and apprehended
the suspects for violating RA
9165(Comprehensive
Dangerous
Drugs Act). They also found
sachets of shabu in Zalamedas
pocket
and
some
drug
paraphernalia around the room.
The police handcuffed the suspects
and informed them of their rights
and the cause against them.
At the Police Station, PO2 De
Guzman immediately marked the
confiscated items and turned them
and the suspects over to SPO4
Mangulabnan
for
laboratory
examination.Police
Inspector
Palacios, the forensic chemical
officer, found the paraphernalia
and the suspects positive for
Methylamphetamine
Hydrochloride or shabu.
Petitioner
denied
the
accusation and said that they were
framed by the police, they extorted
money from them, that the arrest
was without warrant, and that the
evidence
acquired
was
inadmissible in court.

The RTC convicted the


accused and was affirmed by the
CA.
ISSUE:
Whether or not the arrest
was lawful and the chain of
custody rule was followed
HELD:
Yes. The Supreme Court said
that no evidence corroborated the
petitioners version of the story.
Mere denial is not sufficient to
disprove the allegations. In the
ordinary conduct of a public
officer, it is presumed that the
officer lawfully performed his duty
absenting evidence to the contrary.
The SC also ruled that the
warrantless arrest was valid since
the accused were caught in
flagrante delicto while smoking
shabu when the officers entered
the house.
Lastly, in evaluating the
process conducted by PO2 De
Guzman in acquiring, preserving,
marking and documenting the
pieces of evidence from the
operation, the court said that the
chain of custody rule was never
broken and was fully complied
with as stated in Sec. 21 of R.A.
9165. Hence, the Supreme Court
affirmed the decision of the Court
of Appeals.
PEOPLE v. GERENTE
G.R. NO. 95847-48, 10 MARCH
1993

TOPIC: ARREST (RULE 113)


FACTS:
On April 30, 1990 at about
7:00
a.m.,
appellant
Gabriel
Gerente, together with Fredo
Echigoren and Totoy Echigoren,
started
drinking
liquor
and
smoking marijuana in the house of
the appellant which is about six
meters away from the house of the
prosecution
witness
who
overheard the three men talking
about their intention to kill Clarito
Blace. Fredo and Totoy Echigoren
and Gerente then killed Clarito.
Thereafter, the three men dragged
Blace to a place behind the house
of Gerente. Edna Edwina Reyes,
testified that she witnessed the
killing.
At about 4:00 p.m. of the
same day, Patrolman Jaime Urrutia
of the Valenzuela Police Station
received a report from the Palo
Police
Detachment
about
a
mauling incident. He went to the
Valenzuela District Hospital where
the victim was brought. He was
informed by the hospital officials
that the victim died on arrival.
Right away, Patrolman Urrutia,
together with Police Corporal
Romeo Lima and Patrolman Alex
Umali, proceeded to Paseo de Blas
where the mauling incident took
place. They were informed by the
witness, Edna Edwina Reyes, that
she saw the killing and she pointed
to Gabriel Gerente as one of the
three men who killed Clarito. The
policemen proceeded to the house
of the appellant who was then
sleeping. They told him to come

out of the house and they


introduced
themselves
as
policemen.
Patrolman
Urrutia
frisked appellant and found a coin
purse
in
his
pocket
which
contained dried leaves wrapped in
cigarette foil. The dried leaves
were sent to the National Bureau
of Investigation for examination.
The Forensic Chemist found them
to be marijuana. Gabriel Gerente,
was apprehended by the police.
The other suspects are still at
large. Two separate informations
were filed by Assistant Provincial
Prosecutor
Benjamin
Caraig
against him for Violation of Section
8, Art. II, of R.A. 6425 also known
as the Dangerous Drugs Act of
1972, and for Murder. The
appellant pleaded not guilty to
both charges. A joint trial of the
two cases was held. The trial court
rendered a decision convicting him
of Violation of Section 8 of R.A.
6425 and of Murder. The accused
appealed from the decision of the
Regional Trial Court of Valenzuela,
Metro Manila, Branch 172.
ISSUE:
Whether or not Gerente was
lawfuly arrested and searched
even
without
the
necessary
warrants.
HELD:
Yes.
The
search
of
appellant's person and the seizure
of the marijuana leaves in his
possession were valid because they
were
incident
to
a
lawful
warrantless arrest. Paragraphs (a)
and (b), Section 5, Rule 113 of the

Revised Rules of Court provide


that a peace officer or a private
person may, without a warrant,
arrest a person when in his
presence, the person to be
arrested has committed, is actually
committing, or is attempting to
commit an offense and when an
offense has in fact just been
committed, and he has personal
knowledge of facts indicating that
the person to be arrested has
committed it. The policemen
arrested Gerente only some 3
hours after Gerente and his
companions had killed Blace. The
Police patrols saw Blace dead in
the hospital and when they
inspected the scene of the crime,
they found the instruments of
death. The witness, Edna Edwina
Reyes, reported the happening to
the policemen and pinpointed her
neighbor, Gerente, as one of the
killers. Under those circumstances,
since the policemen had personal
knowledge of the violent death of
Blace and of facts indicating that
Gerente and two others had killed
him, they could lawfully arrest
Gerente without a warrant. If they
had postponed his arrest until they
could obtain a warrant, he would
have fled the law as his two
companions did. The search of
Gerent is also valid according to
Section 12, Rule 126 of the
Revised Rules of Court which
provides that a person lawfully
arrested may be searched for
dangerous weapons or anything
which may be used as proof of the
commission of an offense, without
a search warrant.

PEOPLE v. MANLULU
G.R. NO. 102140, 22
1994

APRIL

TOPIC: ARREST (RULE 113)


FACTS:
On 29 May 1986 at around
ten o'clock in the evening, Wally
Manlapaz, a.k.a. Crisanto Meneses
and accused Dante Samson and
Rolando Manlulu were having a
drinking spree in an alley along
Quirino Avenue, Paco, Manila.
They were later joined by Agent
Gerardo Alfaro who had a .45 cal.
pistol tucked to his waist. When
Alfaro arrived he blurted out, "Dito
may kumakatalo saking tao." At 12
midnight, the group transferred in
front of the house of Manlapaz and
continued to drink. There, Samson
suddenly stabbed Alfaro in the
chest with a 6-inch double-bladed
knife while boasting, "Dapat sa iyo
manahimik na. Manlulu then
followed suit and stabbed Alfaro in
the abdomen several times with an
ice pick they used to chip ice.
Samson grabbed the .45 cal.
service pistol of Alfaro and shot
him in the neck. When Alfaro
slumped on the pavement, both
accused fled, with Samson holding
Alfaro's handgun. After a few
seconds, both accused returned
and got Alfaro's wristwatch and
wallet.
Noel
Pagco,
another
witness, recounted that at the time
of the shooting he was outside the
alley where the accused and the
deceased were drinking. After
hearing a gunshot coming from the
direction of the alley, he saw Dante
Samson and Rolando Manlulu

coming out the alley, the former


tucking a gun in his waist and
sporting a watch on his right wrist,
and the latter holding an ice pick.
Patrolman Reynaldo Perez
and some police officers recounted
that at around seven o'clock in the
evening of 30 May 1986 arrested
Manlulu on the information given
by Manlapaz. Malapaz said that he
seized from Manlulu the .45 cal.
pistol and Casio wristwatch said to
belong to Alfaro, and that Manlulu
verbally
confessed
to
the
commission of the crime.
Patrolman Perez however
admitted on cross-examination that
when he arrested Manlulu and
seized from him the handgun as
well as the wristwatch, he was not
with any warrant nor did he inform
the accused of the latter's right to
counsel. Perez added that at that
time Manlulu was under the
influence
of
liquor.
Accused
Manlulu and Samson on appeal
invoked self-defense.
ISSUES:

Whether or not Rolando


Manlulu was arrested lawfully.
HELD:
No. The police authorities
should have first obtained a
warrant for the arrest of accused
Rolando Manlulu, and for the
search and seizure of his personal
effects. The killing took place at
one o'clock in the morning. The

arrest and the consequent search


and seizure came at around seven
o'clock
that
evening,
some
nineteen hours later. This instance
cannot come within the purview of
a
valid
warrantless
arrest.
Paragraph (b), Sec. 5, Rule 113 of
the 1985 Rules on Criminal
Procedure
provides
that
the
arresting
officer
must
have
"personal knowledge" of an offense
which "has in fact just been
committed."
In the instant case, neither
did Pat. Perez has "personal
knowledge," nor was the offense
"in fact just been committed."
While Pat. Perez may have
personally
gathered
the
information which led to the arrest
of Manlulu, that is not enough. The
law
requires
"personal
knowledge." Obviously, "personal
gathering
of
information"
is
different
from
"personal
knowledge." The rule requires that
the arrest immediately follows the
commission of the offense, not
some
nineteen
hours
later.
However,
in
spite
of
the
nullification of the arrest of
accused
Manlulu,
and
the
exclusion of real evidence, as well
as his extra-judicial confession
which was taken in violation of the
provisions of the Constitution, still
the prosecution was able to prove
the guilt of the accused beyond
reasonable doubt. After all, the
illegality of the warrantless arrest
cannot deprive the state of its right
to prosecute the guilty when all
other facts on record point to their
culpability.

headquarters
incident.
PEOPLE v. DEL ROSARIO
G.R. NO. 127755, 14, APRIL
1999
TOPIC: ARREST (RULE 113)
FACTS:
Gabriel Gerente y Bullo was
charged with Violation of Section
8, Art. II of R.A. 6425, before
Regional Trial Court of Valenzuela,
Metro Manila. The conviction of
the accused was based on the
testimony of a tricycle driver Paul
Vincent Alonzo. Alonzo stopped his
tricycle by the side of Nita's
Drugstore, General Luna St.,
Cabanatuan City. Parked at a
distance of about one and a-half
meters in front of him was a
tricycle driven by accused Joseliton
del Rosario. At that point, Alonzo
saw two men thereafter named
Dodong Visaya and Jun Marquez
and a woman named Virginia
Bernas grappling for possession of
a bag. After taking hold of the bag,
Jun Marques started chasing a
man who was trying to help the
woman, while Dodong Visaya
kicked the woman sending her to
the ground. Soon after, Jun
Marquez returned and while the
woman was still on the ground he
shot her on the head. The bag was
brought to the tricycle of accused
del Rosario where Boy Santos was
inside to receive it. When the
tricycle sped away Alonzo gave
chase and was able to get the plate
number of the tricycle. He also
recognized the driver, after which
he went to the nearest police

and

reported

the

Upon finding the name of the


owner of the tricycle by the
policemen, they proceeded to
Bakod Bayan in the house of the
barangay captain where the owner
of the tricycle was summoned and
who in turn revealed the driver's
name
and
was
invited
for
interview. The driver was Joselito
del Rosario who volunteered to
name his passengers on May 13,
1996. On the way to the police
station, accused informed them of
the bag and lunch kit's location
and the place where the holduppers may be found and they
reported these findings to their
officers, Capt. Biag and Capt.
Cruz. After lunch, they proceeded
to Brgy. Dicarma. They went inside
the house where they found
Marquez dead holding a magazine
and a gun. After which they went
back to the police station. The
investigator took the statement of
the accused on May 14,1996, and
was only subscribed on May
22,1996. All the while, he was
detained in the police station as
ordered
by
the
Fiscal.
His
statements were only signed on
May 16, 1996. He also executed a
waiver of his detention. His
Sinumpaang Salaysay was done
with the assistance of Ex-judge
Talavera.
Del
Rosario
was
handcuffed by the police because
allegedly
they
had
already
gathered enough evidence against
him and they were afraid that he
might attempt to escape.
ISSUES:

I. Whether or not the


Miranda rights of the accusedappellant were violated.
II. Whether or not the
warrantless arrest of the accusedappellant was lawful.
HELD:
I. Yes. Miranda rights of the
accused-appellant were violated.
Custodial investigation is the stage
where the police investigation is
no longer a general inquiry into an
unsolved crime but has begun to
focus on a particular suspect taken
into custody by the police who
carry
out
a
process
of
interrogation that lends itself to
elicit incriminating statements. It
is well setttled that it encompasses
any question initiated by law
enforcers after a person has been
taken into custody or otherwise
deprived of his freedom of action
in any significant way. This concept
of custodial investigation has been
broadened by RA 743842 to
include "the Practice of issuing an
'invitation' to a person who is
investigated in connection with an
offense he is suspected to have
committed." Section 2 of the same
Act further provides that Any
public officer or employee, or
anyone acting under his order or in
his place, who arrests, detains or
investigates any person for the
commission of an offense shall
inform the latter, in a language
known and understood by him of
his right to remain silent and to
have competent and independent
counsel, preferably of his own

choice, who shall at all times be


allowed to confer privately with
the person arrested, detained or
under custodial investigation. If
such person cannot afford the
services of his own counsel, he
must be provided with a competent
and independent counsel by the
investigatingofficer.
From the foregoing, it is
clear
that
del
Rosario
was
deprived of his rights during
custodial investigation. From the
time
he
was
"invited"
for
questioning at the house of the
barangay captain, he was already
under
effective
custodial
investigation, but he was not
apprised nor made aware thereof
by the investigating officers. The
police already knew the name of
the tricycle driver and the latter
was already a suspect in the
robbing and senseless slaying of
Virginia
Bernas.
Since
the
prosecution failed to establish that
del Rosario had waived his right to
remain
silent,
his
verbal
admissions on his participation in
the crime even before his actual
arrest were inadmissible against
him, as the same transgressed the
safeguards provided by law and
the Bill of Rights.
II. Warrantless arrest of the
accused-appellant was unlawful.
Arrest of del Rosario is obviously
outside the purview of the
circumstance set forth in a valid
warrantless arrest since he was
arrested on the day following the
commission of the robbery with
homicide. On the other hand, Sec.
5, par. (b), Rule 113, necessitates

two (2) stringent requirements


before a warrantless arrest can be
effected: (1) an offense has just
been committed; and (2) the
person making the arrest has
personal
knowledge
of
facts
indicating that the person to be
arrested had committed it. Hence,
there must be a large measure of
immediacy between the time the
offense was committed and the
time of the arrest, and if there was
an appreciable lapse of time
between the arrest and the
commission of the crime, a
warrant of arrest must be secured.
It is also mandatory that the
person making the arrest must
have
personal
knowledge
of
certain facts indicating that the
person to be taken into custody
has committed the crime. Again,
the arrest of del Rosario does not
comply with these requirements
since, as earlier explained, the
arrest came a day after the
consummation of the crime and
not immediately thereafter. As
such, the crime had not been "just
committed" at the time the
accused was arrested. Likewise,
the arresting officers had no
personal
knowledge
of
facts
indicating that the person to be
arrested
had
committed
the
offense since they were not
present and were not actual
eyewitnesses to the crime, and
they became aware of his identity
as the driver of the getaway
tricycle only during the custodial
investigation.
Wherefore,
Del
Rosario's was acquitted on the
ground of "irresistible force" since
it has been substantiated by clear
and convincing evidence. On the

other hand, conspiracy between


him and his co-accused was not
proved beyond a whimper of a
doubt by the prosecution, thus
clearing del Rosario of any
complicity in the crime charged.

PEOPLE v. KIMURA
G.R. NO. 130805,
2004

07

APRIL

TOPIC: ARREST (RUKE 113)


FACTS:
In the morning of June 27,
1994, Maj. Anso, head of Delta
Group,
Narcotics
Command
(NARCOM) I, North Metro District
Command, Camp Karingal, Quezon
City, received information from a
confidential informant that a
certain Koichi Kishi and Rey
Plantilla were engaged in the
selling of illegal drugs at the Cash
and Carry Supermarket, Makati
City. Acting on said information,
Maj. Anso organized a team
composed of SPO4 Baldovino, Jr.,
SPO1 Cabato and PO3 Cadoy to
conduct surveillance of the area.
A buy-bust operation was
launched. At around 3:00 in the
afternoon of the same day, the
team together with the informant
arrived at the Cash and Carry
Supermarket
and
conducted
surveillance of the area. Later, the
informant was able to contact the
targets who told him that they will
be arriving at 8:00 in the evening
at the parking area Of the Cash
and Carry Supermarket. At around

8:00 in the evening, Koichi and


Rey arrived and were met by PO3
Cadoy and the informant. PO3
Cadoy gave the marked money
worth P1,000.00 to Rey and Koichi
who then handed him the "shabu".
PO3 Cadoy scratched his head as a
pre-arranged
signal
of
the
consummation of the sale. While
the team was approaching, PO3
Cadoy held Koichi by the hand
while Rey scampered away to the
direction
of
the
South
Superhighway. The team brought
Koichi to a safe area within the
Cash and Carry Supermarket and
interrogated him. They learned
from
Koichi
that
his
friends/suppliers will arrive the
same evening to fetch him.

and SPO4 Baldovino, Jr. were


approaching to check what was
inside the wrapped newspaper,
appellant Kimura ran but was
apprehended while Boy was able
to board his jeep and together with
appellant Kizaki who was seated at
the passenger seat sped off. The
police operatives then inspected
the contents of the trunk and
found packages of marijuana. They
brought Koichi and appellant
Kimura to the headquarters and
turned over the seized marijuana
to the investigator who made
markings thereon. The seized
packages were brought to the PNP
Crime Laboratory on June 29,
1994. It was positive to the test of
marijuana.

Several minutes later, a


white Nissan Sentra car driven by
appellant Kimura with his coappellant Kizaki seating at the
passenger seat arrived at the
parking area. Koichi pointed to
them as the ones who will fetch
him. Appellants remained inside
the car for about ten to fifteen
minutes. Then, a certain Boy
driving a stainless jeep, without a
plate number, arrived and parked
the jeep two to three parking
spaces away from the Sentra
car.Boy approached the Sentra car
and after a few minutes, appellants
got out of their car. Appellant
Kizaki went to the stainless jeep
and sat at the passenger seat. Boy
and appellant Kimura went to the
rear of the Sentra car and opened
its trunk. Appellant Kimura got a
package wrapped in a newspaper
and gave it to Boy who walked
back to his jeep. While Maj. Anso

On June 29, 1994, appellant


Kizaki while having dinner with his
friends
at
the
Nippon
Ichi
Restaurant located at Mabini,
Malate, Manila was arrested by
another NARCOM group led by
Maj. Jose F. Appellant Kizaki was
arrested on June 29, 1994 in the
Nippon Ichi Restaurant located at
Mabini, Manila. He was having
dinner with Lt. Col. Rodolfo Tan,
Masami
Y.
Nishino,
Anita
Takeyama and Akiyoshi Takeyama.
These witnesses executed a joint
affidavit and testified that while
they were about to leave the
restaurant, a man got near Kizaki
and asked for his passport whom
they thought was from the
Immigration. Later, they learned
that Kizaki was brought to Camp
Karingal.
Both accused, Kizaki and
Kimura, were positively identified

by prosecution witnesses SPO4


Baldomino,SPO1
Cabatu,
Maj.
Anso and PO3 Cadoy as the
persons whom they arrested for
drug trafficking in a buy-bust
operation at the Cash and Carry
Supermarket on June 27, 1994.
ISSUE:
Whether or
arrest was lawful.

not

Kimuras

HELD:
No. SPO1 Delfin, one of
those who arrested appellant
Kizaki, admitted that they did not
have a warrant of arrest when his
group arrested Kizaki on the night
of June 29, 1994. Rule 113, Section
5 of the Revised Rules of Criminal
Procedure provides that a peace
officer or a private person may,
without a warrant, arrest a person
only under the circumstances
given by law.
The alleged crime happened
on June 27, 1994 and appellant
Kizaki was arrested on June 29,
1994 or two days after the subject
incident. At the time appellant
Kizaki was arrested, he was at a
restaurant having dinner with a
group of friends, thus, he was not
committing
or
attempting
to
commit a crime. Neither was he an
escaped prisoner whose arrest
could be effected even without a
warrant. It bears stressing that
none of the arresting officers of
appellant Kizaki was present on
the night of June 27 where
appellant Kizaki allegedly sold and
transported
marijuana
and

escaped, thus the arresting officers


had no personal knowledge of facts
or circumstances that appellant
Kizaki committed the crime. None
of the exceptions enumerated
above was present to justify
appellant
Kizaki's
warrantless
arrest.
However,
notwithstanding
the unjustified warrantless arrest
of appellant Kizaki, the records
show that he did not raise such
question before he pleaded to the
offense charged. Neither did he
move to quash the information on
that ground before the trial court.
He thus waived objection to the
illegality of his arrest. Moreover,
appellant Kizaki's application for
bail which was denied by the trial
court likewise constitutes a waiver
of his right to question whatever
irregularities and defects which
attended his arrest. The decision of
the trial court in Criminal Case No.
94-5606 is hereby reversed and
appellants Tomohisa Kimura and
Akira Kizaki, are hereby acquitted
on ground of reasonable doubt.

BAIL RULE 114


LEVISTE
v.
COURT
OF
APPEALS
G.R. NO. 189122, 17 MARCH
2010
TOPIC: BAIL (RULE 114)
FACTS:

Charged with the murder of


Rafael de las Alas, petitioner Jose
Antonio Leviste was convicted by
the Regional Trial Court of Makati
City for the lesser crime of
homicide and sentenced to suffer
an indeterminate penalty of six
years and one day of prision mayor
as minimum to 12 years and one
day of reclusion temporal as
maximum.
He
appealed
his
conviction to the Court of Appeals.
Pending appeal, he filed an urgent
application for admission to bail
pending
appeal,
citing
his
advanced
age
and
health
condition,
and
claiming
the
absence of any risk or possibility of
flight on his part. The Court of
Appeals
denied
petitioner's
application for bail. It invoked the
bedrock principle in the matter of
bail pending appeal, that the
discretion to extend bail during the
course of appeal should be
exercised "with grave caution and
only for strong reasons." Citing
well-established jurisprudence, it
ruled that bail is not a sick pass for
an ailing or aged detainee or a
prisoner needing medical care
outside the prison facility. It found
that petitioner failed to show that
he suffers from ailment of such
gravity
that
his
continued
confinement during trial will
permanently impair his health or
put his life in danger.
For purposes of determining
whether petitioner's application
for bail could be allowed pending
appeal, the Court of Appeals also
considered the fact of petitioner's
conviction. It made a preliminary

evaluation of petitioner's case and


made a prima facie determination
that
there
was
no
reason
substantial enough to overturn the
evidence of petitioner's guilt.
Petitioner's
motion
for
reconsideration
was
denied.
Petitioner now questions as grave
abuse of discretion the denial of
his application for bail, considering
that none of the conditions
justifying denial of bail under the
third paragraph of Section 5, Rule
114 of the Rules of Court was
present. Petitioner's theory is that,
where the penalty imposed by the
trial court is more than six years
but not more than 20 years and the
circumstances mentioned in the
third paragraph of Section 5 are
absent, bail must be granted to an
appellant pending appeal.
ISSUE:
Whether
or
not
the
discretionary nature of the grant of
bail pending appeal mean that bail
should automatically be granted
absent any of the circumstances
mentioned in the third paragraph
of Section 5, Rule 114 of the Rules
of Court.
HELD:
No. The Court held that after
conviction by the trial court, the
presumption
of
innocence
terminates and, accordingly, the
constitutional right to bail ends.
From then on, the grant of bail is
subject to judicial discretion. At
the risk of being repetitious, such
discretion must be exercised with
grave caution and only for strong

reasons. Considering that the


accused was in fact convicted by
the trial court, allowance of bail
pending appeal should be guided
by a stringent-standards approach.
This judicial disposition finds
strong support in the history and
evolution of the rules on bail and
the language of Section 5, Rule
114 of the Rules of Court. It is
likewise consistent with the trial
courts initial determination that
the accused should be in prison.
Furthermore, letting the accused
out on bail despite his conviction
may destroy the deterrent effect of
criminal laws. This is especially
germane to bail pending appeal
because long delays often separate
sentencing in the trial court and
appellate review. In addition, at
the post-conviction stage, the
accused faces a certain prison
sentence and thus may be more
likely to flee regardless of bail
bonds or other release conditions.
Finally, permitting bail too freely in
spite of conviction invites frivolous
and time-wasting appeals which
will make a mockery of the
criminal justice system and court
processes.

DOJ v. JIMENEZ
G.R. NO. 148571, SEPTEMBER
24, 2002
TOPIC: BAIL
FACTS:
In an earlier case entitled
Secretary of Justice v. Ralph C.
Lantion,
the
United
States

Government, through the proper


notes and documents sent to the
Philippine Government, requested
the extradition of Mark B. Jimenez,
also known as Mario Batacan
Crespo.
Upon learning the request of
his extradition, Jimenez sought and
was
granted
a
temporary
restraining
order
(TRO),
prohibiting the Department of
Justice (DOJ) from filing with the
Regional Trial Court (RTC) a
petition for his extradition. The
Secretary of Justice (SOJ) assailed
the validity of the TRO but the
Supreme Court (SC) dismissed the
petition.
The Government of the
United States, represented by the
DOJ, filed the appropriate petition
for
extradition.
The
petition
alleged that Jimenez was the
subject of an arrest warrant for
certain crimes.
In order to prevent his flight,
the
petition
prayed
for
his
immediate arrest.
Jimenez filed an urgent
manifestation/ex
parte
motion,
praying for petitioners application
for arrest warrant be set for
hearing.
The RTC granted the motion
of Jimenez. In the hearing,
petitioner
manifested
its
reservations on the procedure
adopted by the trial court allowing
the accused to be heard prior to
the issuance of a warrant of arrest.

The court a quo required the


parties
to
submit
their
memoranda.
In
private
respondents memorandum, he
sought and alternative prayer, that
in case a warrant of arrest should
issue, he be allowed to post bail in
the amount of P100,000. The court
issued his warrant of arrest, fixing
his bail in the amount of one
million pesos.
ISSUE:
I.Whether or not Jimenez is
entitled to notice and hearing
before a warrant for his arrest can
be issued;
II. Whether or not he is
entitled to bail and to provisional
liberty
while
the
extradition
proceedings are pending.
HELD:
I. No. Jimenez is not entitled
to notice and hearing before a
warrant for his arrest can be
issued.
Our
Extradition
Law
provides for the immediate arrest
of the accused. The respondent
judge
gravely
abused
his
discretion when he set the matter
for hearing upon motion of
Jimenez. Respondent judge could
have already gotten an impression
from the documents sent by the
United States Government for him
to make an initial determination of
whether the accused was someone
who
should
immediately
be
arrested in order to serve the best
ends of justice.
Also, in the present case, a
hearing is absent from the

provision,
in
matter
of
the
immediate
arrest.
Extradition
proceedings are summary. The
silence of the law and the treaty
leans to the more reasonable
interpretation that there is no
intention to punctuate with a
hearing every little step in the
entire proceedings.
Section 2 of Article 3 of the 1987
Philippine Constitution does not
require a notice or hearing before
the issuance of a warrant of arrest.
II. No. He is not entitled to
bail and to provisional liberty. Bail
is generally available only for
persons arrested or detained for
the violation of Philippine criminal
laws. It does not apply to
extradition proceedings, because
extradition courts do not render
judgments
of
conviction
or
acquittal. Even if the offenses he
committed are bailable in United
States law, such is not a valid
argument to grant him one in the
present case.

PADILLA v. CA
G.R. NO. 121917,JULY 31, 1996
TOPIC: BAIL
FACTS:
Appellant Robin C. Padilla
was charged with violation of P.D.
No. 1866 for the illegal possession
of
firearms.
Pending
trial,
appellant was released on bail.
Appellant was then convicted and
meted an indeterminate penalty of
17 years, 4 months and 1 day as

minimum to 21 years of reclusion


perpetua. He appealed but was
denied by the Court of Appeals
(CA). Appellants bail bond was
cancelled and his arrest was
ordered. Appellant filed a petition
for certiorari with an application
for bail. Appellant moved for the
separate resolution of his bail
application.
ISSUE:
Whether or not appellant is
entitled to bail
HELD:
No. Appellant is not entitled
to bail. As held by the Court in
People v. Nitcha, when the accused
is charged with a capital offense,
or an offense punishable with
reclusion
perpetua
or
life
imprisonment, and evidence of
guilt is strong, bail shall be denied.
Section 7 of Rule 114 of the Rules
of Court provides that capital
offense or an offense punishable by
reclusion
perpetua
or
life
imprisonment is not bailable.

GO ET. AL. v. BONGOLAN


A.M. NO. RTJ- 99-1464, JULY
26, 1999
TOPIC: BAIL
FACTS:
On November 10, 1997, in
the province of Abra, Jaime
Balmores, Butch Reynaldo, PO1
Rolando Molina, Edgardo Cacal,

and three other John Does,


allegedly kidnapped and detained
Samuel Go. The vehicle he was in
during his transport from Abra to
Ilocos Sur was intercepted by the
PNP wherein Go was recovered.A
case was filed against the accused
for kidnapping with ransom. The
prosecution recommended no bail
for the accused as the penalty for
the crime allegedly committed is
punishable with reclusion perpetua
to death.
Jaime
Balmores
filed
a
motion, asking for the amendment
of the information from kidnapping
with ransom to simple kidnapping,
alleging
that
the
evidence
presented did not show that the
kidnapping
was
for
ransom.
Respondent
Judge
Benjamin
Bongolan of the Regional Trial
Court (RTC) denied Balmores
motion, holding that it is the State
that determines the contents of the
information and it is the State's
responsibility
to
prove
its
allegation
contained
in
the
information under the principle of
`allegata et probata'.
Rolando Molina and Edgardo
Cacal, as allowed by the RTC, filed
their separate memorandum for
motion for bail, reiterating the
claim that prosecution failed to
prove kidnapping for ransom. The
prosecution asserted that the
motion was prematurely filed since
they are still in the process of
presenting further evidence to
prove the commission of the crime.
Judge
Bongolan
subsequently granted the motions

for
bail,
holding
that
the
prosecution did not show that the
evidence of guilt is strong. He,
however, gave the prosecution 10
days to file its motion for
reconsideration.Before
such
motion could be filed, the accused
were already released, through the
order by Judge Alberto Banesa, the
designated pairing judge of Judge
Bongolan, which he issued based
on seeing that Judge Bongolan
already approved the motion for
bail. Judge Bongolan was unaware
of the accused release since he
was in the hospital. Subsequently,
Judge
Bongolan
denied
the
prosecutions
motion
for
reconsideration.
ISSUE:
Whether
or
not
Judge
Bongolan correctly granted the
accused motion for bail before
hearing the bail application
HELD:
No.
Judge
Bongolan
incorrectly granted the accused
motion
for
bail.
Complaints
involving irregular approval of
bailbond and issuance of order
release appear to be a common
offense of judges. A bail hearing is
necessary to give the prosecution
reasonable opportunity to oppose
the application by showing that the
evidence of guilt is strong.
It is true that when asked by
Judge Bongolan whether the
prosecution
would
present
additional evidence, Prosecutor
Gayao responded in the negative.

Subsequently,
however,
the
prosecution changed its mind
when it stated in its Opposition
that a resolution of the Motion for
admission to bail would be
premature since it has additional
witnesses to present.

PEOPLE v. FITZGERALD
GR NO. 149723, OCTOBER 27,
2006
TOPIC: BAIL
FACTS:
Respondent
Victor
Keith
Fitzgerald, an Australian citizen,
was charged with Violation of
Article 3, Section 5 (a)(5) of
Republic Act No. 7610 when he
allegedly
lured
and
drugged
complainant AAA, a 13 year old,
to engage in prostitution and
thereafter, having sex with her.
The Regional Trial Court (RTC)
found the accused guilty of
violating
said
law.Respondent
applied for bail, which the RTC
denied,
holding
that
the
circumstance of the accused
indicates probability of flight and
undue risk that he may commit a
similar offense. Subsequently, his
appeal to the Court of Appeals
(CA) was denied, where the CA
affirmed the RTC ruling, with
modifications on the duration of
his sentence.
Respondent filed for motion
for
new
trial
including
a
supplemental on the ground that
new and material evidence not

previously available had surfaced.


The CA granted the motion.
Respondent subsequently filed for
a
motion
to
fix
bail
with
manifestation
which
the
CA
denied, holding that, pursuant to
Section 7 of Rule 114 of the Rules
of Court, no person charged with a
capital offense, or an offense
punishable by reclusion perpetua
or
life
imprisonment
when
evidence of guilt is strong shall be
admitted to bail regardless of the
stage of the criminal prosecution.
Respondent filed a motion
for early transmittal of the records
and for the re-examination of the
penalty imposed, and a motion for
bail. The CA granted respondents
bail petition, holding that though
the evidence of guilt of the
accused is strong, his plea of
temporary
liberty
should
be
granted considering that he is of
old age and is not in the best of
health.
ISSUE:
I. Whether or not the CA had
jurisdiction, considering the case
has been set to the RTC for new
trial
II. Whether or not the CA
was correct to grant Fitzgeralds
motion for bail
HELD:
I. Yes. The CA still possess
jurisdiction over the case. Under
Section 14, Rule 124 of the Rules
of Criminal Procedure, when the
CA grants new trial, it may either

directly receive newly discovered


evidence or refer the case to the
court of origin for reception of
such evidence under Section 15. In
either case, jurisdiction over the
case is not
relinquished.
II. No. The CA was incorrect
in granting Fitzgeralds motion for
bail. Section 5, Rule 144 of the
2000 Rules of Criminal Procedure
provides
that
bail
may
be
discretionary upon conviction by
the RTC of an offense not
punishable by death, reclusion
perpetua, or life imprisonment. If
the penalty imposed is exceeding 6
years, the accused shall be denied
bail, or his bail shall be cancelled
upon
a
showing
by
the
prosecution, with notice to the
accused, of the following or other
similar circumstances: (a) That he
is a recidivist, quasi-recidivist, or
habitual
delinquent,
or
has
committed the crime aggravated
by the circumstance of reiteration;
(b) That he has previously escaped
from legal confinement, evaded
sentence,
or
violated
the
conditions of his bail without valid
justification; (c) That he committed
the offense while under probation,
parole, or conditional pardon; (d)
That the circumstances of his case
indicate the probability of flight if
released on bail; or (e) That there
is undue risk that he may commit
another crime during the pendency
of the appeal.
When the CA granted the
motion for new trial, it set aside its
own decision when it modified the
duration of the penalty set by the

RTC. Nonetheless, the


the sentence set by the
exceeds 6 years, is
Bail, at that point,
discretionary.

duration of
RTC, which
untouched.
cannot be

It is bad enough that the CA


granted bail on grounds other than
those stated in the Motion filed by
respondent; it is worse that it
granted bail on the mere claim of
the latter's illness. Bail is not a
sick pass for an ailing or aged
detainee or prisoner needing
medical care outside the prison
facility. A mere claim of illness is
not a ground for bail. It may be
that the trend now is for courts to
permit bail for prisoners who are
seriously sick.

another bail. She applied for the


cancellation of the bail she posted.
Respondent judge Hon. Reynaldo
Alhambra denied the petition,
holding that the arrest and
detainment of Gerardo for another
criminal case does not affect the
character of the cash bail posted
by Anita. Money so deposited, even
if done by a third person, takes the
nature of property in custodialegis
and is to be applied for payment or
fine and costs.
ISSUE:
Whether or not Anita can
cancel the bail she posted in favor
of Gerardo.
HELD:

ESTEBAN v. JUDGE ALHAMBRA


GR NO. 135012, SEPTEMBER 7,
2004
TOPIC: BAIL
FACTS:
Private respondent Gerardo
Esteban is the accused in the
criminal cases filed before the
Regional Trial Court (RTC). His
sister-in-law,
petitioner
Anita
Esteban, posted cash bail of
P20,000 in each case for his
temporary liberty. While out on
bail during the pendency of the
criminal proceedings, he was
subsequently charged with another
crime for which he was arrested
and detained.
Fed
up
with
Gerardos
action, Anita refused to post

No. Anita may not cancel the


bail she posted. Section 22, Rule
114 of the Revised Rules of
Criminal Procedure provides that
upon
application
of
the
bondsmen, with due notice to the
prosecutor, the bail may be
cancelled upon surrender of the
accused or proof of his death.
The cash bail cannot be
cancelled because petitioner did
not surrender the accused to the
trial court.
Moreover, the bail bond
posted for the accused was in the
form of cash deposit, which, as
mandated by Section 14 (formerly
Section 11) of the same Rule 114,
shall be applied to the payment of
fine and costs, and the excess, if
any, shall be returned to the

accused or to any person who


made the deposit
A cash bond may be posted
either by the accused or by any
person in his behalf. However, as
far as the State is concerned, the
money deposited is regarded as
the
money
of
the
accused.
Consequently, it can be applied in
payment of any fine and costs that
may be imposed by the court.
Judge Bongolan contends that it is
not necessary for the prosecution
to present all its witnesses before
he could resolve the motion for
bail. The stance cannot be
sustained.
In Borinaga v. Tamin, the
Court ruled that the prosecution
must be given an opportunity to
present its evidence within a
reasonable time whether the
motion for bail of an accused who
is in custody for a capital offense
be
resolved
in
a
summary
proceeding or in the course of a
regular trial. If the prosecution is
denied such an opportunity, there
would be a violation of procedural
due process. The records show
that the prosecution was supposed
to present its 6th and 7th
witnesses on June 4, 1998 when
Judge
Bongolan
prematurely
resolved the motion. A bail
application does not only involve
the right of the accused to
temporary liberty, but likewise the
right of the State to protect the
people and the peace of the
community
from
dangerous
elements. These two rights must
be balanced by a magistrate in the
scale of justice, hence, the

necessity for hearing to guide his


exercise of discretion.

RULE 115 RIGHTS OF THE


ACCUSED NO CASES

PEREZ PATTY
E

RUL
116

ARRAIGNMENT AND PLEA


PEOPLE v. FLORES
G.R. No. 128823-24 Dec. 27,
2002
Topic: Accuseds right to be
informed of the nature and cause
of the accusation against him
Doctrine:
The
accused
has
the
constitutional right to be informed
of the nature and cause of the
accusation against him. This right
has the following objectives: 1. To
furnish the accused with such a
description of the charge against
him as will enable him to make the
defense; 2. To avail himself of his
conviction
or
acquittal
for
protection
against
further
prosecution for the same cause; 3.
To inform the court of the facts
alleged, so that it may decide
whether they are sufficient in law
to support a conviction if one
should be had. The right cannot be
waived for reasons of public policy.

Every crime is made up of


certain acts and intent which must
be set forth in the complaint or
information
with
reasonable
particularity of time, place, names
(plaintiff and defendant), and
circumstances
The complaint must contain a
specific allegation of every fact
and circumstance necessary to
constitute the crime charged, the
accused being presumed to have
no independent knowledge of the
facts that constitute the offense.
Facts:
11-year-old Filipina Flores
claimed that on Dec. 9 and Dec 28,
1996, her father Pedro Flores, Jr.
inserted his fingers then his penis
into her vagina in the bathroom of
their house and upon waking her
from her sleep, respectively. Pedro
denied
the
accusations
and
claimed that Filipina was not at
home when the alleged rape
incidents
happened.
Both
information
filed
with
RTC
Urdaneta, Pangasinan alleged that
Pedro sexually abused Filipina.
RTC found Pedro guilty of two
counts of statutory rape.
Issue:
Whether or not Pedro was
denied the constitutional right to
be informed of the nature and
cause of the accusation against
him
Held:
Yes.
The gravamen of the crime
of rape is carnal knowledge or

sexual intercourse between a man


and a woman. To sustain a
conviction,
the
complaint
or
information must allege that the
accused had carnal knowledge of
or sexual intercourse with the
private complainant, which was
not complied with in this case. The
allegation that Pedro sexually
abused Filipina does not suffice.
The phrase sexually abuse
in the criminal complaints does not
comply with the requirement that
the complaint must contain a
specific averment of every fact
necessary
to
constitute
the
crime. Sexual abuse is not used
under RA 7610 as an elemental
fact of rape but is an altogether
separate offense.
Sec. 8 Rule 110 of the Rules
of Court provides: Designation of
the offense. The complaint or
information
shall
state
the
designation of the offense given by
the statute, aver the acts or
omissions constituting the offense,
and specify its qualifying and
aggravating
circumstances.
If
there is no designation of the
offense, reference shall be made to
the section or subsection of the
statute punishing it.
PEOPLE v. SERZO
G.R. No. 118435 June 20, 1997
Topic: Right to counsel
Doctrine:
The right of an accused to
counsel is guaranteed by the
Constitution. This right is granted
to minimize the imbalance in the
adversarial system where the

accused is pitted against the


awesome prosecutory machinery
of the state. However, the option to
secure the services of counsel de
parte is not absolute. The court
may restrict the accuseds option
to retain a counsel de parte if the
accused insists on an attorney he
cannot afford, or the chosen
counsel is not a member of the bar,
or the attorney declines to
represent the accused for a valid
reason, e.g. conflict of interest and
the like.
The right to be heard and to
reopen the case (and send it to
trial anew) could not be allowed if
doing so would sanction a plainly
dila-tory tactic and a reprehensible
trifling
with
the
orderly
administration of justice.
Facts:
On
Aug.
22,
1990
in
Antipolo, Rizal,
Alfredo
Alcantara
and
wife
Adelaida helped their neighbor
Suzana Serzo, mother of the
accused,
rescue
the
latters
grandchildren who were being
held by Mario Serzo inside their
house. They were able to bring the
children to safety, but as Alfredo
was returning to his house, Mario,
with a bladed weapon, stabbed
Alfredo at the back, which directly
caused
his
death.
At
the
arraignment on Jan. 8, 1991, Mario
appeared without a counsel. The
court appointed Atty. Wilfredo
Lina-ac as counsel de oficio. Mario
moved that the arraignment be
reset and that he be given time to
engage a counsel of his choice,
which the court granted. On Feb.

11, 1991, Mario again appeared


without a counsel de parte. He was
nonetheless arraigned and pleaded
not guilty with the assistance of
Atty. Lina-ac. Subsequent trials
were
cancelled
as
Mario
manifested his intention to secure
a counsel of his choice, but he
never did, causing the court to
appoint two more counsels de
oficio, Atty. Bella Antonano and
PAO Atty. Bonifacia Garcia. Mario
repeatedly refused to sign the
minutes of the proceedings and
declined to take the witness stand,
forcing the defense to rest its case.
Mario wrote Judge Angeles three
times and asked for the early
resolution of his case. RTC found
Mario guilty of murder on the
basis of the evidence presented by
the prosecution.
Issue:
Whether or not Mario was
denied the right to counsel
Held:
No.
While the right of the
accused to be represented by
counsel is immutable, his option to
secure the services of counsel de
parte, however, is not absolute.
The court is obliged to balance the
privilege to retain a counsel of
choice against the states and the
offended partys equally important
right to speedy and adequate
justice.
Mario had been given ample
time to secure the services of a
counsel de
parte,
but
his
subsequent appearances in court
without such counsel and his act of
allowing this situation to continue

until the presentation of his


evidence betrays his lack of
intention to do so. It even appears
that he was merely delaying his
own presentation of evidence on
purpose.
Mario
did
not
demonstrate in what way the
services
of
his
counsels de
oficio were unsatisfactory. Mario
was afforded a chance to be heard
by counsel of his own choice, but
by his own neglect or mischief, he
effectively waived such right.
PEOPLE v. AYSON
G.R. No. 85215 July 7, 1989
Topic: Accuseds right against selfincrimination; rights in custodial
interrogation (to remain silent and
to counsel, and to be informed of
such rights)
Doctrine:
The
right
against
selfincrimination is an option of
refusal to answer incriminating
questions and not a prohibition of
inquiry. It simply secures to a
witness, whether he be a party or
not, the right to refuse to answer
any
particular
incriminatory
question, i.e., one the answer to
which
has
a
tendency
to
incriminate him for some crime. It
does not give a witness the right to
disregard a subpoena, to decline to
appear before the court at the time
appointed, or to refuse to testify
altogether. Said right is not selfexecuting
or
automatically
operational. It must be claimed. If
not claimed by or in behalf of the
witness, the protection does not
come into play.

Miranda rule: the accused has


the right to remain silent, that
anything he says can be used
against him in a court of law, that
he has the right to the presence of
an attorney, and that if he cannot
afford an attorney one will be
appointed for him prior to any
questioning if he so desires.
After such warnings have been
given,
and
such
opportunity
afforded him, the individual may
knowingly and intelligently waive
these rights and agree to answer
or make a statement. The objective
is
to
prohibit
incriminating
statements without full warnings
of constitutional rights.
A person suspected of having
committed
a
crime
and
subsequently charged with its
commission in court, has the
following rights in the matter of
his
testifying
or
producing
evidence, to wit: 1) BEFORE THE
CASE IS FILED IN COURT (or with
the
public
prosecutor,
for
preliminary
investigation),
but
after having been taken into
custody or otherwise deprived of
his liberty in some significant way,
and on being interrogated by the
police: the continuing right to
remain silent and to counsel, and
to be informed thereof, not to be
subjected
to
force,
violence,
threat, intimidation or any other
means which vitiates, the free will;
and to have evidence obtained in
violation of these rights rejected;
and 2) AFTER THE CASE IS FILED
IN COURTa) to refuse to be a
witness; b) not to have any
prejudice whatsoever result to him
by such refusal; c) to testify in his

own behalf, subject to crossexamination by the prosecution; d)


WHILE TESTIFYING, to refuse to
an answer a specific question
which tends to incriminate him for
some crime other than that for
which he is then prosecuted.
Facts:
PAL Baguio City filed an
information with RTC Baguio
against Felipe Ramos charging him
with estafa for misappropriating
fare tickets of passengers for oneway and round-trip in the total
amount of P76,700.65 Before trial
and during PALs investigation,
Ramos submitted a statement and
a handwritten admission, and
offered to settle his obligation on a
staggered basis. However, no
compromise was reached between
Ramos and PAL. Ramos pleaded
not guilty to the charge. The
prosecution presented as evidence
Ramos statement and handwritten
admission. The defense objected
that
the
evidence
presented
appeared to be a confession and
that Ramos made it without being
represented by a lawyer. Judge
Ruben Ayson ruled that said
evidence was inadmissible as
Ramos was not reminded of his
constitutional rights to remain
silent and to counsel, and it does
not appear that he waived the
same.
Issue:
Whether or not Judge Ayson
committed
grave
abuse
of
discretion in excluding said pieces
of evidence

Held:
Yes.
Ramos was not in any sense
under custodial interrogation prior
to and during the administrative
inquiry
into
the
discovered
irregularities in ticket sales in
which he appeared to have had a
hand. Ramos had voluntarily
answered questions posed to him
on
the
first
day
of
the
administrative investigation and
agreed
that
the
proceedings
should be recorded. The note that
Ramos sent his superiors offering
to settle his liability was a free and
even spontaneous act on his part.
They may not be excluded on the
ground that the so-called "Miranda
rights" had not been accorded to
Ramos.
The
right
against
selfincrimination, mentioned in the
Constitution is accorded to every
person
who
gives
evidence,
whether voluntarily or under
compulsion of subpoena, in any
civil, criminal, or administrative
proceeding.
It
prescribes
an
"option of refusal to answer
incriminating questions and not a
prohibition of inquiry." It simply
secures to a witness, whether he
be a party or not, the right to
refute to answer any particular
incriminatory question. However,
the right can be claimed only when
the incriminatory question is
actually put to the witness. It does
not impose on the judge, or other
officer presiding over a trial,
hearing or investigation, any
affirmative obligation to advise a
witness of his right against selfincrimination.

The
rights
in
custodial
investigation apply to persons
"under
investigation
for
the
commission of an offense," i.e.,
"suspects" under investigation by
police authorities; and this is what
makes these rights different from
that
against
self-incrimination
which, applies to any person
testifying in any proceeding, civil,
criminal, or administrative.
A defendant on trial or under
preliminary investigation is not
under custodial interrogation. His
interrogation by the police, if any
there had been, would already
have been ended at the time of the
filing of the criminal case in court.
ANCIRO v. PEOPLE
G.R. No. L-107819 Dec. 17,
1993
Topic: Right to cross-examination
Doctrine:
The right of confrontation thus
guaranteed and secured to the
accused is a personal privilege
which can be waived.
The
fact that
the crossexamination of the complainant
was not formally terminated is not
an irregularity that would justify a
new trial. The right to confront the
witnesses may be waived by the
accused
expressly
or
by
implication.
Facts:
Efren Anciro was found
guilty beyond reasonable doubt by
the Regional Trial Court for the
rape of Gloria Dalin on February
23, 1979 in Carmona, Cavite.

Petitioner appealed the decision to


the Court of Appeals contesting
that he was convicted on the basis
of the testimony of the offended
party even if it was incomplete and
was not their partys fault.
On June 25, 2002, the Court of
Appeals affirmed the decision of
the trial court. It ruled that that
the accused was not denied his
right to cross-examination, even
though
it
was
not
entirely
completed. The records of the case
would reveal that the defense was
afforded ample opportunity for
cross-examination. After the directexamination of the offended party,
the trial court granted the motion
of the defense for a continuance of
the hearing on another date to
allow it to prepare for a thorough
examination of the offended party's
testimony. On the date set for the
continuation
of
the
crossexamination, the offended party
was present and in fact was
sufficiently interrogated by the
defense counsel.
Issue:
Whether or not the crossexamination conducted is valid
even if the testimony of the private
complainant was not finished.
Held:
Yes. The cross-examination is
considered valid.
The petitioner was not denied the
right to cross- examine the
offended party. It is clear that in
the case at bar, the witness was
ready for cross- examination,
however,
private
respondents
counsel, Atty. Loyola, moved the
date, and when he was informed

that
the
witness
(private
complainant) will be leaving the
Philippines on said date, counsel
was fully aware and was ready to
take on another witness. The
testimony
of
the
private
complainant must stand. The
petitioner had not only been given
sufficient opportunity to finish the
cross- examination of the private
complainant, by his conduct he
has, as well, waived his right to
further cross-examine her. His
constitutional right to meet the
witness face to facewas not
impaired. Furthermore, there is
enough evidence on record to
prove beyond reasonable doubt
that the petitioner committed the
crime of rape.
UY v. ADRIANOG.R NO. 159098,
OCTOBER 27, 2006
TOPIC: ARRAIGNMENT AND
PLEA (RULE 116)
Doctrine:
Speedy trial is a relative term
and necessarily a flexible concept.
In determining whether the right
of the accused to a speedy trial
was violated, the delay should be
considered, in view of the entirety
of the proceedings
The right to speedy trial cannot
be invoked where to sustain the
same would result in a clear denial
of due process to the prosecution
it should not operate in depriving
the
State
of
its
inherent
prerogative to prosecute criminal
cases generally in seeing to it that
all of those who approach the bar
of
justice
is
afforded
fair
opportunity to present their side.

The right to a speedy trial does not


preclude the peoples equally
important right to public justice.
Facts:
Spouses Henry and Rosario
Uy,
owners
of
a
business
establishment in the Municipality
of Tarlac, Tarlac, were allegedly
engaged in unfair competition by
using
the
trademark
and
substituting the contents of Marca
Pia soy sauce and selling it as
their own. A search
warrant was implemented and
fifty-five bottles of fake soy sauce
were
confiscated. A
criminal
complaint was filed against the
spouses for the violation of Article
189 (Unfair Competition) of the
Revised
Penal
Code.After
preliminary examination of the
prosecution witnesses, the court
found probable cause against
petitioners.On January 30, 1995,
the court issued a warrant of
arrest against petitioners.They
were released after posting a cash
bond. On July 10, 1995, petitioners
were
arraigned,
assisted
by
counsel, and pleaded not guilty.
Subsequently, in October 1996, the
Court issued Administrative Order
No. 104-96 providing, that the RTC
shall have exclusive jurisdiction
over violations of Articles 188 and
189 of the Revised Penal Code and
Republic Act (R.A.) No. 166 as
amended.
Despite
the
administrative order, the MTC
continued with the trial. In the
meantime, Articles 188 and 189 of
the Revised Penal Code were
amended by R.A. No. 8293,
otherwise
known
as
the
Intellectual Property Code. On

March 10, 2000, petitioners, filed a


Motion for Leave to File Demurrer
to Evidence. Petitioners argued
that there was no sufficient
evidence presented to prove that
they are guilty of the offense
charged. Also, under both R.A. No.
166, as amended, and its repealing
law, R.A. No. 8293, the RTC had
jurisdiction
over
the
crime
charged; therefore, the amended
complaint should be quashed. The
motion was granted. In response,
the prosecution maintained that,
under Batas Pambansa Blg. 129,
the MTC had jurisdiction over the
crime charged in the light of the
imposable
penalty
for
unfair
competition under Article 189 of
the Revised Penal Code. In a
resolution dated May 16, 2000, the
court ruled that the evidence
would be sufficient to warrant
conviction, however, the RTC has
exclusive
jurisdiction on the matter. The case
was forwarded to the Clerk of
Court of the RTC. The City
Prosecutor found probable cause
based on the findings of the MTC.
Petitionersfiled a Motion to Quash
the Information stating that there
has already been 6 years of delay
since the initial case was filed and
their right to a speedy trial was
violated. The court denied the
motion. The information was
amended stating that petitioners
violated R.A. No. 8293 and not
Article 189 of the Revised Penal
Code. Petitioners then filed before
the CA that respondent judge
committed
grave
abuse
of
discretion in denying their motion
to quash based on violation of their
right to a speedy trial. The CA

dismissed the petition, hence, this


petition.
Issue:
Whether or not the Judge
committed
grave
abuse
of
discretion
when
it
denied
petitioners motion to quash based
on violation of their right to a
speedy trial
Held:
No. The Judge did not
commit grave abuse of discretion
when it denied petitioners motion
to quash based on violation of their
right to a speedy trial.
In Section 1(h), Rule 115 of
the Revised Rules of Criminal
Procedure
provides
that
the
accused is entitled to a speedy,
impartial and public trial. Section
2, Rule 119 of the said Rules
provides
that
trial,
once
commenced, shall be continuous
until terminated. However, any
period of delay resulting from a
continuance granted by the court
motuproprio, or on motion of
either the accused or his counsel,
or the prosecution, if the court
granted the continuance on the
basis of its findings set forth in the
order that the ends of justice is
served by taking such action
outweigh the best interest of the
public and the accused on a
speedy trial, shall be deducted.
The accused have the burden to
prove the factual basis of the
motion to quash the Information
on the ground of denial of their
right to a speedy trial. On the
other hand, the prosecution is
required to present evidence
establishing that the delay was

reasonably
attributed
to
the
ordinary processes of justice, and
that petitioners suffered no serious
prejudice beyond
that which
ensued after an inevitable and
ordinary delay. Furthermore, the
right to speedy trial cannot be
invoked where to sustain the same
would result in a clear denial of
due process to the prosecution. In
the case at bar, the delay cannot
only
be
attributed
to
the
prosecution and that the MTC did
not have exclusive jurisdiction over
the case before it was forwarded
to the RTC. There was no showing
failed that the delay in bringing
petitioners to trial in a court of
competent
jurisdiction
caused
them any prejudice tantamount to
deprivation of their right to a
speedy trial. In asserting a delay to
their right to a speedy trial, it
should be based on specifics and
not merely vague assertions.
Failure to present evidence defeats
their claim that their right has
been violated.
People v. Nuelan
G.R. No. 123075, Oct. 8, 2001
Topic: Improvident plea of guilty
Doctrine:
An arraignment is the means of
implementing the constitutional
right of an accused to be informed
of the nature and cause of the
accusation against him so that he
may be informed as to why he was
indicted and what penal offense he
has to face.

Rule 116, Section 3 of the


Revised
Rules
on
Criminal
Procedure requires that the court
conduct a searching inquiry, which
more than informing cursorily the
accused that he faces a jail term
but also, the exact length of
imprisonment under the law and
the certainty that he will serve at
the national penitentiary or a penal
colony. After an accused withdraws
a plea of guilty, the trial court
should re-arraign him.
Facts:
RTC Daet, Camarines Norte
convicted Pedro Nuelan of 3
counts of rape committed against
his 13-year-old daughter Margie.
The crimes happened on Jan. 1,
Feb. 27 and March 4, 1994. On
Aug. 15, 1994, accused enter into
a
plea-bargaining.
With
the
consent of the prosecution, Margie
and her mother, the RTC dropped
Criminal Case No. 8209 (first case)
on condition that the accused
would plead guilty to Criminal
Cases Nos. 8210 and 8211 (last
two cases). Upon re-arraignment,
Pedro pleaded guilty to the
charges in the last 2 cases.
Margie, assisted by elder sister
Agnes,
testified
for
the
prosecution. On Aug. 17, 1994,
after a searching inquiry on the
voluntariness of the plea, RTC
found that accused did not fully
comprehend the consequences of
the plea of guilty and thought that
the charges in the last 2 cases
were only for attempted rape. RTC
allowed Pedro to withdraw his plea
and ordered the reinstatement of
the first case. RTC sentenced
Pedro to death in each of the 3

cases and ordered him to pay


P50,000 moral damages.
Issue:
Whether or not RTC erred in
failing to immediately inquire into
the voluntariness of Pedros plea of
guilty in the last 2 cases
Held:
Yes.
The trial court did not
impress
on
Pedro
full
comprehension
of
the
consequences of his plea of guilty.
Rule 116, Section 3 of the Revised
Rules on Criminal Procedure
requires that the court conduct a
searching inquiry, which more than
informing cursorily the accused
that he faces a jail term but also,
the exact length of imprisonment
under the law and the certainty
that he will serve at the national
penitentiary or a penal colony.
Also, after the accused withdrew
the plea of guilty, the trial court
did
not
re-arraign
accusedappellant. An arraignment is the
means
of
implementing
the
constitutional right of an accused
to be informed of the nature and
cause of the accusation against
him so that he may be informed as
to why he was indicted and what
penal offense he has to face.
People v. Villarama
G.R. No. 99287, June 23, 1992
Topic: Plea of guilty to a lesser
offense
Doctrine:
Sec. 2 Rule 116 of the Rules of
Court requires the accused to

obtain the consent of both the


prosecution and the offended party
for him (accused) to be allowed to
plead guilty to a lesser offense.
If the the plea of guilty to the
lesser offense was made without
the consent of the Fiscal and of the
offended party, the accused could
still be prosecuted under the
original
charge.
No
double
jeopardy exists.
Facts:
On Aug. 24, 1990, Jaime
Manuel was charged with violation
of Sec. 16, RA No. 6425 with
penalty of imprisonment ranging
from six years and one day to
twelve years and a fine ranging
from
six thousand
to
twelve
thousand pesos for possession,
custody and control of 0.08 grams
of Methamphetamin Hydrocloride
(Shabu), a regulated drug. During
arraignment, Manuel entered a
plea of not guilty. Thereafter, trial
ensued. On Nov. 21, 1990, the
prosecution rested its case. On
Jan. 9, 1991, counsel for private
respondent verbally manifested in
open court that Manuel was willing
to change his former plea of "not
guilty" to that of "guilty" to the
lesser offense of violation of
Section 17, R.A. No. 6425, with a
penalty of imprisonment ranging
from six months and one day to
four years and a fine ranging from
six hundred to four thousand
pesos. Manuel failed to obtain the
consent of the prosecution and the
offended party. Still, Judge Martin
Villarama of RTC NCR Pasig,
Metro
Manila
granted
the
accuseds
motion
and
found

Manuel guilty beyond reasonable


doubt of the crime of violation of
Sec. 17, Article III, RA 6425, the
lesser offense.
Issues:
I. Whether or not respondent judge
erred
in
granting
private
respondent's request to plead
guilty to a lesser offense because
the request was filed out of time
and the consent thereto of the
prosecutor and the offended party
was not obtained.
II. Whether or not respondent
judge erred in convicting private
respondent of the lesser offense
instead of the offense originally
charged, in view of the absence of
a valid change of plea.
Held:
I. Yes. Sec. 2 Rule 116 of the Rules
of Court requires the accused to
obtain the consent of both the
prosecution and the offended
party, in this case, the State, for
him (accused) to be allowed to
plead guilty to a lesser offense.
Manuel failed to obtain said
consent as prosecution manifestly
filed its Opposition to the Motion.
II. Yes. The trial court disregarded
the prosecutions objections that
Manuel filed his Motion after the
prosecution had already rested its
case and that the possibility of
conviction
of
the
private
respondent of the crime originally
charged (violation of Sec. 16) was
high because of the strong
evidence presented. The judgment
was based only on one objection:
waste of valuable time already

spent
by
the
court
and
prosecution. Absent any finding on
the weight of the evidence in hand,
the respondent judge's acceptance
of the private respondent's change
of plea is improper and irregular.
As to Manuels allegation of double
jeopardy, the Court ruled that it
has no basis. Sec. 7 Rule 117 of
the Rules of Court states that
conviction of the accused shall not
be a bar to another prosecution for
an offense which necessarily
includes the offense charged in the
former complaint or information
under any of the following
instances: xxx (c) the plea of guilty
to the lesser offense was made
without the consent of the Fiscal
and of the offended party. Hence,
Manuel could still be prosecuted
under the original charge.
People v. Espidol
G.R. No. 150033, Nov. 12, 2004
Doctrine:
Sec. 3 Rule 116 of the 2000
Revised
Rules
of
Criminal
Procedure
states:
When
the
accused pleads guilty to a capital
offense, the court shall conduct a
searching
inquiry
into
the
voluntariness
and
full
comprehension
of
the
consequences of his plea and shall
require the prosecution to prove
his guilt and the precise degree of
culpability. The
accused
may
present evidence in his behalf.
Even if the trial court is satisfied
that the plea of guilty was entered
with full knowledge of its meaning
and consequences, the Court must

still require the introduction of


evidence for the purpose of
establishing the guilt and the
degree of culpability of the
defendant. A plea of guilty is only a
supporting evidence or secondary
basis for a finding of culpability,
the main proof being the evidence
presented by the prosecution to
prove the accuseds guilt beyond
reasonable doubt.
Topic: Improvident plea of guilty
Facts:
Sagrado Dalacat, with coaccused Danilo Espidol, Alfredo
Tomas,
Virgilio
Corpuz,
and
Armando Aniasco were convicted
of the complex crime of robbery in
band with homicide for committing
a
hold-up
at
the
business
establishment of the family of
Hipolito Bagay in Vigan, Ilocos Sur
on Oct. 14, 1998. Bagay died of
gunshot wounds. The band carried
away P1.2M on a Mitsubishi L-300
van with plate no. CME 337. The
case against Corpuz was dismissed
for insufficiency of evidence and he
then became a state witness,
Espidol died in jail, Limos fled
from prison and remained at-large,
while Aniasco was arrested in
Cabanatuan City for another
offense. On Mar. 8, 1999, Dalacat
was arraigned and, with the
assistance of Atty. Barrios, pleaded
not guilty. On June 13, 2001,
Dalacat changed his mind and new
counsel Atty. Vitamog manifested
in open court Dalacats desire to
change his plea to guilty. A new
hearing was set on July 26, 2001 to
assess Dalacats comprehension of
his plea. The court inquired from

appellant if his new counsel, Atty.


Ascao, clearly explained to him the
legal effects and consequences of
his plea of guilty. He answered in
the affirmative. The court then
deemed the case submitted for
decision
without
asking
the
defense if it would present
evidence in its behalf.
Issue:
Whether or not Dalacats
plea of guilty was validly made to
convict him of a capital offense
Held:
No.
Sec. 3 Rule 116 of the 2000
Revised
Rules
of
Criminal
Procedure
states:
When
the
accused pleads guilty to a capital
offense, the court shall conduct a
searching
inquiry
into
the
voluntariness
and
full
comprehension
of
the
consequences of his plea and shall
require the prosecution to prove
his guilt and the precise degree of
culpability. The
accused
may
present evidence in his behalf.
Even if the trial court is satisfied
that the plea of guilty was entered
with full knowledge of its meaning
and consequences, the Court must
still require the introduction of
evidence for the purpose of
establishing the guilt and the
degree of culpability of the
defendant. A plea of guilty is only a
supporting evidence or secondary
basis for a finding of culpability,
the main proof being the evidence
presented by the prosecution to
prove the accuseds guilt beyond
reasonable doubt.

RULE 117
MOTION TO QUASH
Pacoy v. People
G.R. No. 157472, Sept. 28, 2007
Topic: Amendment or substitution
of Information; Motion to quash
Doctrine:
Pursuant to Sec. 14 Rule 110
and Sec. 19 Rule 119 of the Rules
of Court, the change of the offense
charged from Homicide to Murder
is merely a formal amendment and
not a substantial amendment or a
substitution. The only change
made was in the caption of the
case;
and
in
the
opening
paragraph or preamble of the
Information, with the crossing out
of
word
Homicide
and
its
replacement by the word Murder.
There was no change in the recital
of facts constituting the offense
charged or in the determination of
the jurisdiction of the court.
Pursuant to Sec. 3 and Sec. 7 Rule
117 of the Rules of Court,
dismissal
of
the
first
case
presupposes
a
definite
or
unconditional
dismissal,
which
terminates the case. And for the
dismissal to be a bar under the
jeopardy clause, it must have the
effect of acquittal. Judge Cajigals
Order dated Sept. 12 was for the
trial prosecutor to correct and
amend the Information but not to
dismiss the same upon the filing
of a new Information charging the
proper offense

Facts:
On or about Mar. 18, 2002 in
Tarlac, SSGT. Jose Pacoy shot his
commanding officer 2Lt. Frederick
Escueta with his armalite rifle that
caused multiple gunshots wounds
and led to Escuetas instant death.
Pacoy was charged with homicide
with aggravating circumstance of
disregard
of
rank.
Upon
arraignment on Sept. 12, 2002,
Pacoy pleaded not guilty. On the
same day and after arraignment,
Judge
Afable
Cajigal
issued
another
order
directing
the
prosecutor
to
change
the
Information from homicide to
murder as he (Cajigal) considered
disregard of rank as a qualifying
circumstance. The prosecutor then
crossed out the word homicide and
instead wrote the word murder in
the caption and opening paragraph
of
the
Information
without
changing anything in the body.
Pacoy was to be re-arraigned on
Oct. 8, 2002, but his counsel
objected that Pacoy would be
placed in double jeopardy since
the complaint for homicide was,
according to counsel, dismissed as
it was terminated without Pacoys
express consent. Pacoys counsel
filed a Motion to Inhibit and a
Motion
for
Reconsideration.
Cajigal denied the former and
granted the latter, and reinstated
the original Information charging
Pacoy of homicide. Cajigal realized
that disregard of rank is a generic
and not a qualifying aggravating
circumstance and should not
elevate the classification of crime
from homicide to murder.

Issues:
I. The respondent judge gravely
abused
his
discretion
and
exceeded
his
jurisdiction
in
ordering the amendment of the
information from homicide to
murder.
II. The respondent judge gravely
abused his discretion and violated
the law in denying the motion to
quash the information for murder.
III. The respondent judge gravely
abused
his
discretion
and
exceeded his jurisdiction and
violated the law in ordering the
reinstatement of the information
for homicide which was already
terminated
Held:
The petition has no merit.
I. Pursuant to Sec. 14 Rule 110
and Sec. 19 Rule 119 of the Rules
of Court, the change of the offense
charged from Homicide to Murder
is merely a formal amendment and
not a substantial amendment or a
substitution. The only change
made was in the caption of the
case;
and
in
the
opening
paragraph or preamble of the
Information, with the crossing out
of
word
Homicide
and
its
replacement by the word Murder.
There was no change in the recital
of facts constituting the offense
charged or in the determination of
the jurisdiction of the court.
II. Pursuant to Sec. 3 and Sec. 7
Rule 117 of the Rules of Court,
dismissal
of
the
first
case
presupposes
a
definite
or
unconditional
dismissal,
which

terminates the case. And for the


dismissal to be a bar under the
jeopardy clause, it must have the
effect of acquittal. Cajigals Order
dated Sept. 12 was for the trial
prosecutor to correct and amend
the Information but not to dismiss
the same upon the filing of a
new Information
charging
the
proper offense.
III.
Homicide
is
necessarily
included in the crime of murder;
thus, Cajigal merely ordered the
amendment of the Information and
not the dismissal of the original
Information.
Cajigal
granted
Pacoys motion for reconsideration,
not on the ground that double
jeopardy exists, but on his
realization that disregard of rank
is
a
generic
aggravating
circumstance which does not
qualify the killing of the victim to
murder. Thus, Cajigal rightly
corrected himself by reinstating
the
original
Information
for
Homicide.
Los Baos v. Joel Pedro
G.R. 173588, April 22, 2009
Topic: Provisional dismissal vs.
Quashal
Doctrine:
The delimitation of the
grounds available in a motion to
quash suggests that a motion to
quash is a class in itself. The
failure of the Rules to state that a
provisional dismissal is a bar to
further prosecution shows that the
framers did not intend a dismissal
based on a motion to quash and a
provisional
dismissal
to
be

confused with one another. If the


problem relates to an intrinsic or
extrinsic
deficiency
of
the
complaint or information, as shown
on its face, the remedy is a motion
to quash. All other reasons for
seeking the dismissal of the
complaint or information, before
arraignment and under
the
circumstances outlined in Sec. 8,
fall under provisional dismissal.
Facts:
On or about May 13, 2001 or
a day before the elections in Boac,
Marinduque,
Joel
Pedro
was
caught carrying a Revolver Cal.
357, Magnum Ruger 100 loaded
with 6 ammunitions, with Serial
No.
173-56836
outside
his
residence during the election
period, without authorization in
writing from Comelec. This was in
violation of BP 881 or the Omnibus
Election Code. Pedro filed with the
RTC a Motion to Quash, arguing
that the Information contains
averments, which, if true, would
constitute a legal excuse or
justification and/or that the facts
charged do not constitute an
offense. Pedro attached to his
motion
a
Comelec Certification dated Sept.
24, 2001 that he was exempted
from the gun ban. RTC quashed
the Information and ordered the
police and the prosecutors to
return the seized articles to Pedro.
Private prosecutor Ariel Los Baos
moved to reopen the case, as
Pedros Comelec Certification was
a falsification, and the prosecution
was deprived of due process when
the judge quashed the information
without a hearing. The RTC

reopened the case but Pedro


moved for the reconsideration of
the RTCs order primarily based on
Sec. 8 of Rule 117, arguing that
the
dismissal
had
become
permanent
and
the
public
prosecutors
lack
of
express
approval of the motion to reopen
the case. Los Baos contends that
under Sec. 6 of Rule 117, an order
sustaining a motion to quash does
not bar another prosecution for the
same offense, unless the motion
was
based on
the
grounds
specified in Sec. 3. Initially, CA
denied Pedros petition and ruled
that it was not proved when the
public prosecutor was served the
order of dismissal dated Nov. 22,
2001. However, upon motion for
reconsideration manifesting that
the public prosecutor received the
order of dismissal at 2:35 p.m. on
Dec. 1, 2001, CA reversed itself
and said that the State is barred
form reopening Pedros case.
Issue:
Whether Section 8, Rule 117
is applicable to the case, as the CA
found
Held:
No. Sec. 3 Rule 117 of the
Revised Rules of Court enumerates
the grounds for the quashal of a
complaint or information:
(a)
That the facts
charged
do
not
constitute
an
offense;
(b)
That the court
trying the case has
no jurisdiction over
the
offense
charged;

(c)

That the court


trying the case has
no jurisdiction over
the person of the
accused;
(d) That the officer
who
filed
the
information had no
authority to do so;
(e) That it does not
conform
substantially to the
prescribed form;
(f) That more than one
offense is charged
except
when
a
single punishment
for various offenses
is prescribed by
law;
(g) That the criminal
action or liability
has
been
extinguished;
(h) That it contains
averments which, if
true,
would
constitute a legal
excuse
or
justification; and
(i) That the accused
has been previously
convicted
or
acquitted of the
offense charged, or
the case against
him was dismissed
or
otherwise
terminated without
his
express
consent.
Sec.
8.
Provisional
dismissal. A case shall not be
provisionally
dismissed
except with the express
consent of the accused and

with notice to the offended


party.
The
provisional
dismissal
of
offenses
punishable by imprisonment
not exceeding six (6) years
or a fine of any amount, or
both,
shall
become
permanent one (1) year after
issuance
of
the
order
without the case having
been revived. With respect
to offenses punishable by
imprisonment of more than
six
(6)
years,
their
provisional dismissal shall
become permanent two (2)
years after issuance of the
order without the case
having been revived.
The delimitation of the grounds
available in a motion to quash
suggests that a motion to quash is
a class in itself. The failure of the
Rules to state under Sec. 6 that a
Sec. 8 provisional dismissal is a
bar to further prosecution shows
that the framers did not intend a
dismissal based on a motion to
quash and a provisional dismissal
to be confused with one another. If
the problem relates to an intrinsic
or extrinsic deficiency of the
complaint or information, as
shown on its face, the remedy is a
motion to quash. All other reasons
for seeking the dismissal of the
complaint or information, before
arraignment and under
the
circumstances outlined in Sec. 8,
fall under provisional dismissal
The grounds Pedro cited in his
motion to quash are that the
Information contains
averments

which, if true, would constitute a


legal excuse or justification [Sec.
3(h), Rule 117], and that the facts
charged do not constitute an
offense [Sec. 3(a), Rule 117]. We
find from our examination of the
records that the Information duly
charged a specific offense and
provides the details on how the
offense was committed. Thus, the
cited Sec. 3(a) ground has no
merit. COMELEC Certification is a
matter aliunde that is not an
appropriate motion to raise in, and
cannot support, a motion to quash
grounded on legal excuse or
justification found on the face of
the Information.
Thus, the RTC grossly erred in its
initial ruling that a quashal of the
Information was in order. Pedro, on
the
other
hand,
also
misappreciated the true nature,
function, and utility of a motion to
quash. As a consequence, a valid
Information still stands, on the
basis of which Pedro should now
be arraigned and stand trial.
ANTONNE v. BERONILLA G.R.
183824, DECEMBER 8, 2010
Topic: MOTION TO QUASH
Doctrine:
For jeopardy to attach, the
following requisites must concur:
(1) there is a complaint or
information or other formal charge
sufficient in form and substance to
sustain a conviction; (2) the same
is filed before a court of competent
jurisdiction; (3) there is a valid
arraignment or plea to the
charges; and (4) the accused is
convicted or acquitted or the case

is
otherwise
dismissed
or
terminated without his express
consent.
Jeopardy does not attach in
favor of the accused on account of
an order sustaining a motion to
quash, anchored on the ground
that the facts charged do not
constitute an offense is not a bar
to another prosecution for the
same offense.
Motion to quash is a mode by
which an accused assails the
validity of a criminal complaint or
Information filed against him for
insufficiency on its face in point of
law, or for defects which are
apparent in the face of the
Information.
Facts:
Petitionercharges
Bigamy
against Leo R. Beronilla before the
Office of the City Prosecutor of
Pasay City. She alleged that her
marriage with respondent in 1978
had not yet been legally dissolved
when the latter contracted a
second marriage with one Cecile
Maguillo in 1991.
Pending the setting of the case for
arraignment, herein respondent
moved to quash the Information on
the ground that the facts charged
do not constitute an offense. He
informed the court that his
marriage with petitioner was
declared null and void by the RTC;
that the decision became final and
executory; and that
The delimitation of the grounds
available in a such decree has
already been registered with the
Municipal Civil Registrar. He

argued that since the marriage had


been declared null and void from
the beginning, there was actually
no
first
marriage
to
speak
of.Absent a first valid marriage,
the facts alleged in the Information
do not constitute the crime of
bigamy.
The prosecution invoked that a
motion to quash is a hypothetical
admission of the facts alleged in
the information, and that facts
contrary thereto are matters of
defense which may be raised only
during
the
presentation
of
evidence.
After a hearing on the motion, the
court quashed the Information.
Applying Morigo v. People, the trial
court ruled that the accused was
actually never legally married to
petitioner. On this score alone, the
first element appears to be
missing.
Furthermore,
the
statement in the definition of1.
Bigamy which reads before the
first marriage has been legally
dissolved clearly contemplates that
the first marriage must at least be
annullable
or
voidable
but
definitely not void, as in this case.2.
Under the principle of retroactivity
of a marriage being declared void
ab initio, the two were never
married from the beginning.
Meanwhile, in a petition for
certiorari under Rule 65 of the
Rules of Court filed before the
Court of Appeals, herein petitioner
alleged that the Pasay City trial
court acted without or in excess of
jurisdiction or with grave abuse of
discretion amounting to lack or
excess of jurisdiction when it
dismissed the case of bigamy and
denied
her
motion
for

reconsideration.
The Court of Appeals dismissed the
petition mainly arguing that there
is a violation of the rule on double
jeopardy as the dismissal of the
subject
criminal
case
is
tantamount to an acquittal based
on the trial courts finding that the
first essential element of bigamy,
which is a first valid marriage
contracted by private respondent
is wanting.Petitioner filed her
comment/opposition
to
private
respondents motion to quash
before the trial court issued its
Order dated September 20, 2007
dismissing the information. Hence,
if there is no denial of due process,
there can be no grave abuse of
discretion that would merit the
application of the exception to the
double jeopardy rule.
Issues:
Is the filing of this petition is in
violation of the respondents right
against double jeopardy on the
theory that he has already been
practically acquitted when the trial
court quashed the Information?
Did the trial court act without or in
excess of jurisdiction or grave
abuse of discretion when it
sustained respondents motion to
quash on the basis of a fact
contrary to those alleged in the
information?
Held:
The Orders of the RTC as
well as the Resolutions of the
Court of Appeals were set aside.
The criminal case isremandedto
the
trial
court
for
further
proceedings.
1. Well settled is the rule that for

jeopardy to attach, the following


requisites must concur: (1) there is
a complaint or information or other
formal charge sufficient in form
and substance to sustain a
conviction; (2) the same is filed
before a court of competent
jurisdiction; (3) there is a valid
arraignment or plea to the
charges; and (4) the accused is
convicted or acquitted or the case
is
otherwise
dismissed
or
terminated without his express
consent.
The third and fourth requisites are
clearly wanting in the instant case
as (a) respondent has not yet
entered his plea to the charge
when he filed the Motion to Quash
the Information, and (2) the case
was dismissed not merely with his
consent but, in fact, at his
instance.
We reiterate, time and again, that
jeopardy does not attach in favor
of the accused on account of an
order sustaining a motion to
quash. More
specifically, the
granting of a motion to quash
anchored on the ground that the
facts charged do not constitute an
offense is not a bar to another
prosecution for the same offense.
2. It is axiomatic that a complaint
or information must state every
single fact necessary to constitute
the offense charged; otherwise, a
motion to dismiss/quash on the
ground that it charges no offense
may be properly sustained. The
fundamental test in considering a
motion to quash on this ground is
whether the facts alleged, if
hypothetically
admitted,
will
establish the essential elements of
the offense as defined in the law.

Contrary
to
the
petitioners
contention, a reading of the
information will disclose that the
essential elements of the offense
charged are sufficiently alleged.It
is not proper therefore to resolve
the charges at the very outset, in a
preliminary hearing only and
without the benefit of a full-blown
trial. The issues require a fuller
examination.
GIAN PAULO VILLAFLOR v.
DINDO VIVAR Y GOZONG.R.
NO. 134744. JANUARY 16, 2001
TOPIC/DOCTRINE: MOTION TO
QUASH/LACK
OF
PRELIMINARY
INVESTIGATION
NOT
A
GROUND FOR MOTION TO
QUASH
Doctrine:
Preliminary investigation is an
inquiry or proceeding to determine
whether there is sufficient ground
to engender a well-founded belief
that a crime has been committed
and the respondent is probably
guilty thereof, and should be held
for trial. It is a a statutory and
substantive right accorded to the
accused before trial. To deny their
claim
to
a
preliminary
investigation would be to deprive
them of the full measure of their
right to due process.
The absence of a preliminary
investigation does not impair the
validity of the information or
otherwise render it defective, and
the
trial
court,
instead
of
dismissing the information, should
hold in abeyance the proceedings

and order the public prosecutor to


conduct
a
preliminary
investigation.
If the amendment of the
information is only a formal one
(from slight to serious physical
injuries), the accused cannot
demand
a
new
preliminary
investigation. The following have
been held to be merely formal
amendments, viz.: (1)
new
allegations that relate only to the
range of the penalty that the court
might impose in the event of
conviction; (2) an amendment that
does not charge another offense
different or distinct from that
charged in the original one; (3)
additional allegations that do not
alter the prosecutions theory of
the case so as to cause surprise to
the accused and affect the form of
defense to be assumed; and (4) an
amendment
that
does
not
adversely affect any substantial
right of the accused, such as the
right to invoke prescription.
Facts:
An Information forinjuries,
was filed against Respondent
DindoVivar The case stemmed
from the alleged mauling of
Petitioner Gian Paulo Villaflor by
respondent around 1:00 a.m. on
January 27, 1997 outside the Fat
Tuesday Bar at the Ayala Alabang
Town Center, Muntinlupa City.
After the severe beating he took
from
respondent,
petitioner
decided to leave the premises
together with a friend who was in
the restroom when the mauling
incident took place. On his way
out,
petitioner
again
met

respondent who told him, Sa


susunod gagamitin ko na itong
baril ko.
When the injuries sustained by
petitioner turned out to be more
serious than they had appeared at
first, an Information for serious
physical injuries, was filed against
respondent.The earlier charge of
slight
physical
injuries
was
withdrawn. Respondent posted
cash bonds and instead of filing
acounter-affidavit as required by
the trial court he filed on April 21,
1997, a Motion to Quash the
Information for the offense of
grave threats. He contended that
the threat, having been made in
connection with the charge of
serious physical injuries, should
have been absorbed by the latter.
MTC denied the motion to quash
as well as the Motion for
Reconsideration, he was then duly
arraigned and pleaded not guilty.
Respondent filed a Petition for
Certiorari with the RTC of
Muntinlupa City, which held that,
The Judicial Officer appears to
have acted with grave abuse of
discretion amounting to lack of
jurisdiction
in
declaring
and
denying the MOTION TO QUASH
as a prohibitive motion. The same
should have been treated and
[should have] proceeded under the
regular rules of procedure. The
MOTION
TO
QUASH
THE
INFORMATIONS
filed
without
preliminary
investigation
is
therefore granted and these cases
should have been dismissed.
Petitioners filed for Motion for
Reconsideration but such was
denied.

Issues:
A. Whether or not the court
motupropio
order
the
dismissal of the two (2)
criminal cases for serious
physical injuries and grave
threats on the ground that
the public prosecutor failed
to conduct a preliminary
investigation?
B. Whether or not the failure of the
public prosecutor to conduct a
preliminary
investigation
be
considered a ground to quash the
criminal informations for serious
physical injuries and grave threats
filed
against
the
accusedrespondent?
Held:
A. No, the absence of a
preliminary
investigation
does not impair the validity
of
the
information
or
otherwise
render
it
defective. Neither does it
affect the jurisdiction of the
court or constitute a ground
for
quashing
the
information. The trial court,
instead of dismissing the
information, should hold in
abeyance the proceedings
and
order
the
public
prosecutor to conduct a
preliminary
investigation.
Hence, the RTC in this case
erred when it dismissed the
two criminal cases for
serious physical and grave
threats on the ground that
the public prosecutor had
failed
to
conduct
a
preliminary
investigation.
Furthermore, the Supreme

Court do not agree that a


preliminary
investigation
was not conducted. In fact, a
preliminary investigation for
slight physical injuries was
made by the assistant city
prosecutor of Muntinlupa
City. The said Information
was,
however,
amended
when petitioners injuries
turned out to be more
serious and did not heal
within the period specified
in the Revised Penal Code.
The filing of the Amended
Information, without a new
preliminary
investigation,
did not violate the right of
respondent to be protected
from a hasty, malicious and
oppressive prosecution; an
open and public accusation
of a crime; or from the
trouble, the expenses and
the anxiety of a public trial.
B. No, the absence of a preliminary
investigation does not impair the
validity of the information or
otherwise render it defective.
Neither
does
it
affect
the
jurisdiction of the court over the
case or constitute a ground for
quashing the information. Section
3, Rule 117 of the Revised Rules of
Criminal Procedure, provides the
grounds on which an accused can
move to quash the complaint or
information. These are: (a) the
facts charged do not constitute an
offense; (b) the court trying the
case has no jurisdiction over the
offense charged (c) the court
trying the case has no
jurisdiction over the person of the
accused; (d) the officer who filed

the information had no authority to


do so; (e) the information does not
conform
substantially
to
the
prescribed form; (f) more than one
offense is charged, except in those
cases in which existing laws
prescribe a single punishment for
various offenses; (g) the criminal
action or liability has been
extinguished; (h) the information
contains averments which, if true,
would constitute a legal excuse or
justification; and (i) the accused
has been previously convicted or is
in jeopardy of being convicted or
acquitted of the offense charged.
Nowhere in the above-mentioned
section is there any mention of a
lack of a preliminary investigation
as a ground for a motion to quash.
Moreover, such motion is a
prohibited pleading under Section
19 of the Revised Rules on
Summary
Procedure.
In
the
present case, the RTC therefore
erred
in
granting
herein
respondents Motion to Quash.
WILLIAM
MADARANG
v.
COURT OF APPEALSG.R. NO.
143044. JULY 14, 2005
TOPIC/DOCTRINE: MOTION TO
QUASH/RES JUDICATA NOT A
GROUND
TO
QUASH
INFORMATION
Doctrine:
As provided in Section 5, Rule
110 of the Rules of Criminal
Procedure, all criminal actions are
prosecuted under the direction and
control of the public prosecutor.
The prosecution of offenses is thus
the concern of the government
prosecutors. Petitioners should

implead
the
People
of
the
Philippines as respondent in the
RTC and in the CA to enable the
public prosecutor or Solicitor
General, as the case may be, to
comment on the petitions. The
failure to implead is fatal to
petitioners cause.
Facts:
Private respondent Janice
Young-Chua and her husband,
Eduardo
Chan-Chua,
filed
a
complaint
for
replevin
and
damages
against
petitioners
William Madarang and Evans Kho
in the Regional Trial Court of
Quezon City, and raffled to Branch
84.The complaint alleged that
private respondent is the owner of
a 1990 dark gray Kia Pride car,
evidenced
by
Certificate
of
Registration and that petitioners,
through force and intimidation,
took possession of the subject car
by virtue of a falsified Deed of Sale
dated December 3, 1993 allegedly
executed by private respondent in
favor of petitioner Madarang.
Upon
complaint
of
private
respondent, petitioner Madarang
was charged with Falsification of
Public
Document
in
the
Metropolitan Trial Court of Quezon
City (MeTC) and on the same day,
petitioners were charged with
Grave Coercion in the same MeTC
which was then raffled to the same
branch
.The
cases
were
consolidated and jointly tried.
Branch 84 then dismissed the
complaint for replevin upon finding
that the deed of sale is genuine
and
that
private
respondent
voluntarily surrendered possession
of the car to the petitioners, but

respondents were able to file an


appeal. Madarang then filed a
Motion to Dismiss the falsification
case on the ground that the
decision dismissing the replevin
suit in RTC, Branch 84 involving
the same parties absolved him of
criminal liability in the case of
falsification, the case was then
dismissed but upon Motion for
Reconsideration of the respondent
the case dismissal for the case of
falsification was recalled.
Petitioners filed a Second Omnibus
Motion to Quash Criminal on the
ground that the findings of RTC,
Branch 84 that the signature of
private respondent in the deed of
sale is not falsified and that private
respondent
voluntarily
surrendered possession of the car
to
the
petitioners
bar
the
prosecution for falsification and
grave coercion. The MeTC denied
petitioners motion to quash, ruling
that the decision rendered by the
RTC, Branch 84 in the replevin
case cannot absolve petitioners of
the charges in the criminal cases
as said decision has not attained
finality since it is pending appeal
before the CA. Petitioners then
filed a petition forcertioraribefore
the RTC, they assailed the MeTCs
denial of their motion to quash the
informations for falsification of
public
document
and
grave
coercion and alleged that the
MeTC should have adopted the
factual findings of RTC, Branch 84
in the replevin case as res judicata.
They maintain that such factual
findings of RTC, Branch 84 in its
Decision, bar their prosecution in
the criminal cases for falsification
of public document and grave

coercion. They submit that once a


court of competent jurisdiction
puts to finish an issue of fact, it
cannot be disturbed by the lower
court and, accordingly, the factual
findings of RTC, Branch 84 cannot
be overturned by the MeTC. Such
petition was also denied, and
unable to secure a favorable
judgment they now file another
petition for certiorari anchored on
the same issues to the Court of
Appeals but was once again
dismissed. Thus, they now come to
the Supreme Court for review.
Issue:
A. Whether or not a special civil
action for certiorari and
prohibition is the proper
remedy to assail the denial of
a motion to quash an
information.
B. Whether or not res judicata
is as a ground to quash an
information.
Held:
A. No, settled that a special civil
action
forcertiorariand prohibition is not
the proper remedy to assail the
denial of a motion to quash an
information. The established rule
is that when such an adverse
interlocutory order is rendered,
the remedy is not to resort
forthwith
to
certiorari
or
prohibition, but to continue with
the case in due course and, when
an unfavorable verdict is handed
down to take an appeal in the
manner authorized by law. Only
when the court issued such order

without or in excess of jurisdiction


or with grave abuse of discretion
and
when
the
assailed
interlocutory order is patently
erroneous and the remedy of
appeal would not afford adequate
and
expeditious
relief
will
certiorari
be
considered
an
appropriate remedy to assail an
interlocutory
order.
The
declaration of RTC, Branch 84 in
its Decision that the signature of
private respondent in the Deed of
Sale is genuine and she voluntarily
surrendered the car to petitioners
is not res judicata in the criminal
cases for falsification and grave
coercion because there is no
identity of parties as the People of
the Philippines is not a party in the
replevin suit and cannot be bound
by the factual findings therein.
Besides, the decision of RTC,
Branch 84 is still pending appeal
with the CA. Hence, at the time the
MeTC, the RTC and the CA
rendered their assailed order,
decision
and
resolution,
respectively, there existed no
special circumstance to warrant a
dismissal of the cases pending in
the MeTC.
B. No, Section 3, Rule 117 of the
1985 Rules of Criminal Procedure,
the governing law at the time of
the filing of the indictments,
provides the grounds on which an
accused can move to quash the
complaint or information. These
are: (a) the facts charged do not
constitute an offense; (b) the court
trying the case has no jurisdiction
over the offense charged; (c) the
court trying the case has no
jurisdiction over the person of the

accused; (d) the officer who filed


the information had no authority to
do so; (e) the information does not
conform
substantially
to
the
prescribed form; (f) more than one
offense is charged, except in those
cases in which existing laws
prescribe a single punishment for
various offenses; (g) the criminal
action or liability has been
extinguished; (h) the information
contains averments which, if true,
would constitute a legal excuse or
justification; and (i) the accused
has been previously convicted or is
in jeopardy of being convicted or
acquitted of the offense charged.
Nowhere
in
the
enumerated
excepted grounds is there any
mention of res judicata as a
ground to quash an information.

RULE 118 PRE-TRIAL NO


CASES
RULE 119 TRIAL NO CASES
SURLA KRISTINE
RULE

120

Chua vs. Court


Appeals and Chiok

of

JUDGMENT

G.R. No. 140842


April 12, 2007
Topic: Judgment

Doctrine: The rule authorizing


promulgation of judgment in
absentia is intended to obviate the
situation in the past where the
judicial process could be subverted
by the accused jumping bail to
frustrate the promulgation of
judgment.

Facts: Respondent Wilfred Chiok


represented himself as a licensed
stockbroker and an expert in the
stock market when he met
petitioner Rufina Chua. Petitioner
designated respondent as her
stockbroker after he encouraged
her to invest her money in stocks.
Respondent encouraged her to
purchase shares in bulk, she
entrusted him the amount of
P9,563,900.00 to buy shares of
stocks in bulk. However, weeks
after
petitioner
entrusted
respondent with the money, he
admitted that he spent the money.
Respondent issued two checks as
payment but the checks were
dishonored for insufficient funds.
Petitioner then came to now that
respondent was not a licensed
stockbroker but only a telephone
clerk at Bernard Securities, Inc.
Petitioner caused the filing of
an information for estafa against
the
respondent.
Respondent
pleaded not guilty. Respondent
denied the charge against him. He
testified that he buys and sells US
dollars and what actually existed
between him and petitioner was an
unregistered partnership.

After the prosecution and the


defense
had
presented
their
respective evidence, RTC set the
promulgation of judgment on
January
26,
1999.
However,
respondent and his counsel failed
to appear on said date despite
notice.
The
RTC
reset
the
promulgation of judgment on
February 1, 1999, with notice to
respondent. Again, respondent
failed to appear. The trial court
then promulgated its Decision
convicting respondent of estafa.
On February 15, 1999, respondent
filed a motion for reconsideration
of the judgment. RTC issued an
Omnibus
Order
denying
respondents motion. Respondent
elevated the case to the Court of
Appeals.
CA
issued
a
TRO
enjoining
the
RTC
from
implementing the Omnibus Order.

Issue: Whether the RTC should


have promulgated its decision
immediately
when
respondent
failed to appear before the court
despite notice

Ruling: The Court stressed that


when respondent did not appear
during
the
promulgation
of
judgment on January 26, 1999
despite
notice,
and
without
offering any justification therefor,
the trial court should have
immediately
promulgated
its
Decision. The promulgation of
judgment in absentia is mandatory
pursuant to Section 6, Rule 120 of
the Rules of Court.

It bears stressing that the


rule authorizing the promulgation
of judgment in absentia is intended
to obviate the situation in the past
where the judicial process could be
subverted by the accused jumping
bail to frustrate the promulgation
of judgment. The RTC should have
promulgated the judgment in
absentia on January 26, 1999. The
resetting the promulgation on
February 1, 1999 is tantamount to
condoning respondents act of
making a mockery of our judicial
process, thereby defeating the
avowed purpose of the Rule.

Sangguniang Bayan of Taguig


vs. Estrella
A.M. No. 01-1608-RTJ
January 16, 2001
Topic: Judgment
Doctrine:
A
decision
which
correctly applies the law and
jurisprudence will nevertheless be
subject to questions of impropriety
when rendered by a magistrate or
tribunal believed to be less than
impartial and honest.

Facts: Then mayoral candidate


Richard Papa, Jr. filed an election
protest against Isidro Garcia, the
candidate proclaimed mayor of
Taguig in the May 8, 1995
elections before the RTC. Papa
objected to a total of 11,290
ballots or Garcia allegedly having
been written by one or two

persons. Papas motion to recount


the votes for Garcia was granted.
Thus, a final revision report of the
ballots
was
submitted
to
respondent
judge.
Respondent
judge ordered the NBI to examine
the contested ballots and have a
handwriting expert to examine the
contested ballots. After the NBI
finished
its
examination,
respondent
issued
an
order
directing the removal of the ballot
boxes and election paraphernalia
form his courtroom.
On July 22, 1997, Garcia filed
a motion praying that he be
furnished with a copy of the NBI
Reports and/or allow him to copy
or review or at least to read the
said reports. Respondent denied
the motion on the same day as well
as set the date of promulgation of
judgment on July 31, 1997. This
prompted
Garcia
to
file
a
Manifestation and Most Urgent
Motion to Defer and/or Cancel
Scheduled
Promulgation
of
Judgment. The motion was again
denied
by
the
respondent
explaining that it would be
opening the floodgates for undue
delay. Thereafter, Garcia filed a
petition for certiorari, prohibition
and mandamus with a prayer for
restraining order and preliminary
injunction with the COMELEC on
July 30, 1997. COMELEC granted
the motion.
On August 21, 1997, after
the expiration of the TRO, Papa
filed a Motion for Immediate
Promulgation
of
Judgment,
requesting that the same be heard
on August 25, 1997. Respondent

granted the motion the very next


day, setting August 27, 1997 as the
promulgation date. On August 26,
1997, COMELEC issued an order
directing respondent to allow both
parties or their counsel to have
access to the NBI reports and to
give the parties copies thereof
before the promulgation of the
decision. On the day of the
promulgation, respondent gave
Garcias counsel 5 minutes to go
over the NBI report. Thereafter,
judgment
was
promulgated
declaring Papa the duly elected
mayor. Garcia filed a CounterProtest but was ordered dismissed.
Complainants
alleged
that
respondent judge, together with
Papa and the NBI officials violated
Section 3(e) of the Anti-Graft and
Corrupt Practices Act.

Agcaoili, that, A judge should, in


pending or prospective litigation
before him, be scrupulously careful
to avoid such action as may
reasonably tend to waken the
suspicion that his social or
business relations or friendships
constitute
an
element
in
determining his judicial course. He
must not only render a just,
correct and impartial decision but
should do so in such a manner as
to be free from any suspicion as to
his fairness, impartiality and
integrity.

Moll vs. Court of Appeals


G.R. No. 145425
December 9, 2002
Topic: Judgment

Issue: Whether or not the decision


of the respondent may still be
subject to questions of impropriety

Ruling: The Court ruled that a


decision which correctly applies
the law and jurisprudence will
nevertheless
be
subject
to
questions of impropriety when
rendered by a magistrate or
tribunal believed to be less than
impartial and honest.
Respondent was found guilty
of serious. misconduct, partiality,
and inexcusable negligence.
The Court reiterated the
ruling in the case of Evelyn
Agpalasin vs. Judge Ernesto M.

Doctrine: a.The Court will not


entertain
reiterating
identical
petitions after a judgment has
become final.

Facts: There are two criminal


actions filed before the RTC and
the MeTC against the petitioners:
(1) a violation of Section 3(e) of RA
No. 3019 and (2) Criminal Case
No. 4088 for usurpation of
authority as penalized under
Article
177
of
the
RPC,
respectively.
The
promulgation
of
judgment for the Criminal Case
No. 4088 was postponed and was
reset twice due to the absence of

the petitioner, petitioners counsel


and the pendency of his motion to
quash the judgment on the ground
of double jeopardy. The motion to
quash of the petitioner was denied
and a judgment was promulgated.
The defense counsel contended
that the promulgation is unlawful
and unjust, thus, the petitioner and
his counsel left the courtroom.
Petitioner filed a motio n for
reconsideration of the denial of the
motion to quash but was denied.
The petitioner for certiorari before
the
RTC
was
also
denied.
Petitioner appealed the decision to
the Court of Appeals but was also
denied.
Petitioner
field
two
motions for reconsideration before
the CA. Both were denied.
Petitioner
filed
the
following
motions before the Supreme Court:
(1) review on certiorari (2) two
motions for reconsideration before
the Court (3) urgent motion to
elevate the case to en banc and (4)
two motions for reconsideration
before the Court En Banc.

Issue:
Whether
or
not
the
petitioners actions are tenable

Ruling: The Court said that the


petitioner is making a mockery of
justice and trifling with the judicial
process
to
evade
the
final
judgment against him.
The Court takes note of
petitioners fierce determination to
evade the execution of a judgment
which has long become final.. A
perusal of the records indicates

that no new matters or arguments


were raised by petitioner in his
urgent motion to elevate the case
to the en banc. Rather the issues
and grounds cited were a mere
rehash of the issues already more
than sufficiently passed upon in his
petition for review and petition to
refer the case to the en banc.

So vs. Court of Appeals


G.R. No. 138869
August 21, 2001
Topic: Judgment
Doctrine:
An
annulment
of
judgment may be availed of only in
case of extrinsic fraud and lack of
jurisdiction.

Facts: Petitioner was accused for


violating BP No. 22. RTC rendered
a decision convicting petitioner
guilty. Petitioner appealed his
conviction before the Court of
Appeals.
CA
affirmed
RTCs
decision. He then elevated the
case to the Supreme Court. After
the decision had become final and
executory, the RTC issued a
warrant of arrest against the
petitioner. Petitioner before the
RTC an Urgent Motion for
Declaration of Nullity of Judgment
invoking the doctrine in Co vs. CA
which declared that a check issued
merely to guarantee an obligation
is not covered by BP No. 22
pursuant to Circular No. 4. RTC

denied the petition. On appeal, CA


affirmed the decision of the RTC.

Issue:
Whether
or
not
the
petitioner may still file a motion
after a final judgment has been
rendered by the Court

Ruling: The Court dismissed the


petitioner for lack of merit.
Litigation must end and
terminate
sometime
and
somewhere, and it is essential to
an effective administration of
justice that once a judgment has
become final, the issue or cause
therein should be laid to rest. Even
an alleged erroneous application of
a legal principle cannot bring a
judgment that has already attained
the status of finality to an absolute
nullity under the well-entrenched
rule of finality of judgment. This
rule
is
grounded
on
the
fundamental principle of public
policy and sound practice that at
the risk of occasional error, the
judgment of the court must
become final at some definite date
fixed by law.
The Court added that an
annulment of judgment may be
availed of only in case of extrinsic
fraud and lack of jurisdiction.

Teves vs. Sandiganbayan


G.R. No. 154182

December 17, 2004


Topic: Judgment
Doctrine: Pursuant to the variance
doctrine,
a
person
may
be
convicted of an offense proved
even if not charged in the
Information provided it is included
in what is charged.

Facts: Edgar Y. Teves, former


Mayor
of
Valencia,
Negros
Oriental, and his wife Teresita Z.
Teves seeks to annul and set aside
the decision of the Sandiganbayan
convicting them of violation of
Section 3(h) of the Anti-Graft Law
for possessing direct pecuniary
interest in the Valencia Cockpit
and Recreation Center in Valencia.
The
Sandiganbayan,
however, absolved the petitioners
of the charge of causing the
issuance of a business permit or
license to operate the Valencia
Cockpit and Recreation Center for
not being well-founded.
The petitioners assert that
the
Sandiganbayan
committed
serious and palpable errors in
convicting them. In the first place,
the charge was for alleged
unlawful intervention of Mayor
Teves in his official capacity in the
issuance of a cockpit license in
violation of Section 3(h) of the
Anti-Graft Law. But they were
convicted of having a direct
financial or pecuniary interest in
the
Valencia
Cockpit
and
Recreation
Center
prohibited
under Section 89(2) of the LGC of

1991, which is essentially different


from the offense with which they
were
charged.
Thus,
the
petitioners
insist
that
their
constitutional right to be informed
of the nature and cause of the
accusation against them was
transgressed because they were
never apprised at any stage of the
proceedings in the Sandiganbayan
that they were being charged with,
and arraigned and tried for,
violation of the LGC of 1991.
Second, according to the
petitioners,
their
alleged
prohibited pecuniary interest in
the Valencia Cockpit in 1992 was
not proved.
Also assigned as glaring error is
the conviction of Teresita Teves,
who is not a public officer. In the
information, only Mayor Teves was
accused of having a direct financial
or pecuniary interest in the
operation of the Valencia Cockpit
and Recreation Center in Negros
Oriental.

Issue:
Whether
or
not
the
petitioners may be convicted of a
different crime charged in the
information?

Ruling: Edgar Teves was convicted


of violation of Section 3(h) of RA
3019 while Teresita Teves was
acquitted
for there was so
sufficient ground to convict her of
conspiring with her husband to
commit the crime.

There are two modes by which


a public officer who has a direct or
indirect financial or pecuniary
interest in any business, contract,
or transaction may violate Section
3(h) of the Anti-Graft Law:
1. in
connection
with
his
pecuniary interest in any
business,
contract
or
transaction,
the
public
officer intervenes or takes
part in his official capacity;
and
2. when he is prohibited from
having such interest by the
Constitution or any law.
The Sandiganbayan convicted
the petitioners of violation of
Section 3(h) of the Anti-Graft Law
based on the second mode for the
evidence overwhelmingly evinces
that Mayor Teves had a pecuniary
interest in the Valencia Cockpit,
which is prohibited under Section
89(2) of the LGC of 1991.
In view of the variance
doctrine embodied in Section 4, in
relation to Section 5, Rule 120,
Rules of Criminal Procedure, the
petitioners may be convicted of the
crime even if it was not charged in
the information.
Section 4 provides that
when there is a variance between
the offense charged in the
complaint or information and that
proved, and the offense as charged
is included in or necessarily
includes the offense proved, the
accused shall be convicted of the
offense proved which is included in
the offense charged, or of the

offense charged which is included


in the offense proved.
While Section 5 states that
an offense charged necessarily
includes the offense proved when
some of the essential elements or
ingredients of the former, as
alleged in the complaint or
information, constitutes the latter.
And
an
offense
charged
is
necessarily included in the offense
proved
when
the
essential
ingredients
of
the
former
constitute or form part of those
constituting the latter.
It is clear that the essential
ingredients of the offense proved
constitute or form part of those
constituting the offense charged.
The offense proved is necessarily
included in the offense charged, or
the offense charged necessarily
includes the offense proved. The
variance
doctrine
thus
finds
application to this case, thereby
warranting
the
conviction
of
petitioner Edgar Teves for the
offense proved.

People vs. Sumingwa


G.R. No. 183619
October 13, 2009
Topic: Judgment
Doctrine: Pursuant to the variance
doctrine,
a
person
may
be
convicted of an offense proved
even if not charged in the
Information provided it is included
in what is charged.

Facts: In twelve Informations, the


prosecution
charged
appellant
with two counts of Acts of
Lasciviousness, four counts of
Rape, three counts of Unjust
Vexation, one count of Other Light
Threats,
one
count
of
Maltreatment, and one count of
Attempted
Rape
for
acts
committed against his minor
daughter AAA from 1999-2001.
Appellant denied all the allegations
against him and pleaded not guilty.
AAA executed an Affidavit of
Recantation claiming that while
appellant
indeed
committed
lascivious acts against her, she
explained that appellant did not
actually rape her, as there was no
penetration.
The RTC rendered a decision
convicting appellant of six counts
of acts of lasciviousness, one count
of attempted rape and one count of
unjust vexation. In view of the
withdrawal of AAAs earlier claim
of the fact of penetration, the court
sustained
the
innocence
of
appellant on the rape charges and
concluded
that
the
crime
committed was only Acts of
Lasciviousness.
On appeal, the CA affirmed
the conviction of appellant but
convicted him of Qualified Rape
instead of Acts of Lasciviousness.
The appellate court concluded
that,
notwithstanding
AAAs
retraction
of
her
previous
testimonies,
the
prosecution
sufficiently
established
the
commission of the crime of Rape.

It added that the qualifying


circumstances of minority and
relationship
were
adequately
proven.

Issue: Whether or not may be


convicted of Qualified Rape and
Acts of Lasciviousness despite the
retraction of AAAs testimonies.

Ruling: The Supreme Court said


that in rape cases if the testimony
of the complainant is clear,
consistent
and
credible
to
establish
the
crime
beyond
reasonable doubt, a conviction may
be based on it, notwithstanding its
subsequent retraction. Hence, CA
correctly held that AAAs testimony
is credible notwithstanding her
subsequent
retraction.
The
Supreme Court also affirmed that
CA correctly convicted appellant of
Qualified Rape and of Acts of
Lasciviousness.
In her direct testimony, AAA
stated that appellant removed her
short pants and panty, went on top
of her and rubbed his penis against
her vaginal orifice. She resisted by
crossing her legs but her effort
was not enough to prevent
appellant from pulling her leg and
eventually inserting his penis into
her vagina. Clearly, there was
penetration. It is noteworthy that
appellant pulled victims leg, so
that he could insert his penis into
her vagina. This adequately shows
that appellant employed force in
order to accomplish his purpose.
Moreover, in rape committed by a

father against his own daughter,


the formers moral ascendancy and
influence over the latter may
substitute for actual physical
violence and intimidation. The
moral and physical dominion of the
father is sufficient to cow the
victim into submission to his
beastly desires, and no further
proof need be shown to prove lack
of the victims consent to her own
defilement.
While
appellants
conviction was primarily based on
the
prosecutions
testimonial
evidence,
the
same
was
corroborated by physical evidence
consisting of the medical findings
of the medico-legal officer that
there were hymenal lacerations.
When a rape victims account is
straightforward and candid, and is
corroborated
by
the
medical
findings
of
the
examining
physician, the same is sufficient to
support a conviction for rape.
Aside from the fact of commission
of rape, the prosecution likewise
established that appellant is the
biological father of the victim and
that the latter was then fifteen (15)
42 years old. Thus, the CA aptly
convicted him of qualified rape,
defined and penalized by Article
266-B of the RPC.
In Criminal Case Nos. 1649
and 1654, although appellant was
charged
with
qualified
rape
allegedly committed on the second
week of November 2000 and May
27, 2001, he should be convicted
with
Acts
of
Lasciviousness
committed against a child under
Section 5(b), Article III of R.A.
7610.
AAA testified that in
November 2000, while she and

appellant
were
inside
the
bedroom, he went on top of her
and rubbed his penis against her
vaginal orifice until he ejaculated.
She likewise stated in open court
that on May 27, 2001, while inside
their comfort room, appellant
rubbed his penis against her
vagina while they were in a
standing
position.
In
both
instances,
there
was
no
penetration, or even an attempt to
insert his penis into her vagina.
The aforesaid acts of the appellant
are covered by the definitions of
"sexual abuse" and "lascivious
conduct" under Section 2(g) and
(h) of the Rules and Regulations on
the Reporting and Investigation of
Child Abuse Cases promulgated to
implement the provisions of R.A.
7610. Following the variance
doctrine, appellant can be found
guilty of the lesser crime of Acts of
Lasciviousness committed against
a child.
Appellant is likewise guilty of
two
counts
of
Acts
of
Lasciviousness under Section 5(b),
Article III, R.A. 7610 committed
against AAA on the second week of
August 1999 and on the first week
of September 1999. AAA testified
that in August, appellant, with
lewd design, inserted his hands
inside her shirt then fondled her
breasts; and in September, he
forced her to hold his penis until
he ejaculated.
The
Supreme
Court
is
mindful of the fact that appellant
was specifically charged in an
Information
for
Acts
of
Lasciviousness
defined
and

penalized by Article 336 of the


RPC. However, the failure to
designate the offense by statute, or
to mention the specific provision
penalizing the act, or an erroneous
specification of the law violated,
does not vitiate the information if
the facts alleged clearly recite the
facts
constituting
the
crime
charged. The character of the
crime is not determined by the
caption or preamble of the
information
nor
from
the
specification of the provision of
law alleged to have been violated,
but by the recital of the ultimate
facts and circumstances in the
complaint or information. In the
present case, the body of the
information contains an averment
of the acts alleged to have been
committed by appellant which
unmistakably
refers
to
acts
punishable under Section 5(b),
Article III, R.A. 7610.

RULE 121- NT MR NO CASES

RULE 122- APPEAL

Dinglasan,
Appeals

Jr.

vs.

Court

of

order or resolution denying his


second motion for reconsideration.

G.R. No. 145420


September 19, 2006
Topic: Appeal

Doctrine: After the judgment or


final resolution is entered in the
entries of judgment, the case shall
be laid to rest.

Facts: Petitioner was found guilty


of violating Batas Pambansa Blg.
22 by the Regional Trial Court
which was affirmed by the Court of
Appeals and the Supreme Court.
By virtue of the final and executory
judgment
rendered
by
the
Supreme Court, the prosecution
filed a motion with the RTC for the
issuance of the warrant of arrest
and writ of execution in order to
satisfy the judgment.
Alarmed, Dinglasan filed the
instant Petition for New Trial and,
in
the
alternative,
for
the
Reopening of the Case based on
newly discovered evidence. He
urges the Supreme Court to
uphold
substantial
justice,
emphasizing
that
the
newly
discovered evidence is so material
and of such weight that, if,
admitted would probably change
the judgment, hence, suspension of
procedural rules is warranted. He
also argues that judgment attains
finality only upon the receipt of the

Issue: Whether or not the Supreme


Court should grant the petition for
new trial

Ruling: The Supreme Court the


dismissed
the
petition
of
Dinglasan.
The Supreme Court said that
to rule that finality of judgment
shall be reckoned from the receipt
of the resolution or order denying
the
second
motion
for
reconsideration would result to an
absurd situation whereby courts
will be obliged to issue orders or
resolutions denying what is a
prohibited motion in the first
place, in order that the period for
the finality of judgments shall run,
thereby, prolonging the disposition
of cases. Moreover, such a ruling
would allow a party to forestall the
running of the period of finality of
judgments by virtue of filing a
prohibited
pleading;
such
a
situation is not only illogical but
also unjust to the winning party.
It

bears stressing further


that
the Resolution of the
Supreme Court dated 28 June
1999 became final and executory
as evidenced by the Entry of
Judgment
according
to
the
pertinent provision of the Revised
Rules of Court, which reads, After
the judgment or final resolution is
entered in the entries of judgment,
the case shall be laid to rest. A

decision that acquired finality


becomes
immutable
and
unalterable and it may no longer
be modified in any respect even if
the modification is meant to
correct erroneous conclusions of
fact or law and whether it will be
made by the court that rendered it
or by the highest court of the
land.
Furthermore, it should be
emphasized that the applicant for
new trial has the burden of
showing that the new evidence he
seeks to present has complied with
the requisites to justify the holding
of a new trial. Very clearly, the
filing of the instant Petition for
New Trial and/or Reopening of the
Case on 30 October 2000 was
made way beyond the prescriptive
period for doing so.

Philippine Rabbit
Inc., vs. People

Bus

Lines,

G.R. No. 147703

property and was sentenced to


suffer imprisonment and to pay
damages. The court further ruled
that in the event of the insolvency
of accused, petitioner shall be
liable for the civil liabilities of the
accused. Evidently, the judgment
against accused had become final
and executory.
Admittedly,
accused
had
jumped bail and remained at-large.
The CA ruled that the institution of
a criminal case implied the
institution also of the civil action
arising from the offense. Thus,
once determined in the criminal
case
against
the
accusedemployee,
the
employers
subsidiary civil liability as set forth
in Article 103 of the Revised Penal
Code becomes conclusive and
enforceable.

Issue:
Whether
or
not
the
employer may appeal the judgment
of conviction independently of the
accused.

April 14, 2004


Topic: Appeal
Doctrine: When the accusedemployee absconds or jumps bail,
the judgment meted out becomes
final and executory.

Facts: Napoleon Roman was found


guilty and convicted of the crime
of reckless imprudence resulting
to
triple
homicide,
multiple
physical injuries and damage to

Ruling: It is well-established in our


jurisdiction that the appellate
court may, upon motion or motu
proprio, dismiss an appeal during
its pendency if the accused jumps
bail. This rule is based on the
rationale that appellants lose their
standing in court when they
abscond. Unless they surrender or
submit to the courts jurisdiction,
they are deemed to have waived
their right to seek judicial relief.

The accused cannot be


accorded the right to appeal unless
they voluntarily submit to the
jurisdiction of the court or are
otherwise arrested within 15 days
from notice of the judgment
against them. While at large, they
cannot seek relief from the court,
as they are deemed to have waived
the appeal. It should be stressed
that the right to appeal is neither a
natural right nor a part of due
process. It is merely a procedural
remedy of statutory origin, a
remedy that may be exercised only
in the manner prescribed by the
provisions of law authorizing such
exercise.
Hence,
the
legal
requirements must be strictly
complied with.

Agulto vs. Court of Appeals


G.R. No. L-52728

newly discovered evidence, i.e., a


copy of a marriage contract
between Andrea Suico and one
Romeo
Vergeire
supposedly
contracted on July 19, 1960, or
before Andrea's marriage to the
petitioner.
The court denied the motion
on the ground that it was filed too
late because the accused, with due
diligence, could have discovered
the
so-called
newly-discovered
evidence sooner and could have
presented it during the trial. The
court denied petitioners motion
for reconsideration. The petition
for certiorari in the Court of
Appeals was likewise denied.

Issue: Whether or not the Court of


Appeals and the trial court gravely
abused their discretion in refusing
to reopen the trial.

January 17, 1990


Topic: Appeal
Doctrine: A Motion to Reopen Trial
may be presented only after either
or both parties have formally
offered and closed their evidence,
but before judgment.

Facts: An information for bigamy


was filed against the petitioner,
Avelino C. Agulto. After the trial
was finished and the parties had
rested, but before judgment was
promulgated, the accused filed on
November 12, 1975 a motion to
reopen the trial on the ground of

Ruling: The Court of Appeals did


not commit grave abuse of
discretion in refusing to reopen the
trial.
A Motion to Reopen Trial
may be presented only after either
or both parties have formally
offered and closed their evidence,
but before judgment. There is no
specific provision in the Rules of
Court for motions to reopen trial.
It is albeit a recognized procedural
recourse
or
devise,
deriving
validity and acceptance from long
established usage. The reopening
of a case for the reception of

further evidence before judgment


is not the granting of a new trial.
Petitioner's motion to reopen
the trial on the ground of newly
discovered evidence of a previous
marriage between Andrea Suico
and Romeo Vergeire, assuming the
marriage was valid, was not
supported by evidence that said
marriage was still existing when
Andrea Suico wed the petitioner.
On the other hand, the fact that
the fiscal did not charge her with
bigamy
is
significant.
Unlike
Agulto, she was found by the fiscal
to be under no impediment to
contract a second marriage.

Remulla vs. Manlongat


G.R. No. 148189
November 11, 2004
Topic: Appeal
Doctrine: Rules on the perfection
of appeals, particularly on the
period for filing notices of appeal,
must occasionally yield to the
loftier ends of substantial justice
and equity.

Facts: On August 2, 1996, the City


Prosecutors Office of Makati filed
an information for frustrated
murder
against
Remulla.
On
November
28,
1996,
Judge
Francisco X. Velez ordered the
dismissal
of
the
Information
against Remulla on the ground of
forum-shopping.
The
City

Prosecutors Office of Makati


received a copy of the order of
dismissal on December 3, 1996,
while the private prosecutors
received their copy on December
5, 1996. On December 19, 1996,
Prosecutor Andres N. Marcos filed
a Notice of Appeal, impugning the
aforesaid order of dismissal. On
the same day, Judge Nemesio Felix,
acting judge, in place of Judge
Velez who retired, resolved to deny
the Notice of Appeal on the ground
that it was filed out of time. Both
public and private prosecutors
filed a motion to reconsider the
December 19, 1996 order. On
January 8, 1998, the assailed order
was issued by the trial court judge,
reversing his prior order dated
April 23, 1997 and accordingly
denied prosecutions notice of
appeal.
The appellate court held that
the public prosecutors failure to
file the Notice of Appeal on time
was excusable. In their desire to
have
their
appeal
taken,
respondent and his private counsel
consulted the OSG over the
apprehensions
of
the
public
prosecutor about bringing the
appeal by himself.
Petitioner argues that no
exceptional instances merited the
relaxation of the rules.

Issue: Whether or not rules with


regard to the perfecting an appeal
may be relaxed

Ruling: The Supreme Court said


that these rules are not iron-clad,
especially when rules of procedure
must yield to the loftier demands
of substantial justice and equity.
Procedural rules are mere tools
designed
to
facilitate
the
attainment
of
justice;
their
application must be liberalized to
promote public interest.
In a number of cases, the
Supreme Court has in fact relaxed
the period for perfecting an
appeal, especially on grounds of
substantial justice or when there
are other special and meritorious
circumstances and issues. Verily,
the Supreme Court has the power
to relax or suspend the rules or to
exempt a case from their rigid
operation when warranted by
compelling
reasons
and
the
requirements of justice.
In the present case, the late
filing -- by only one day -- of the
prosecutions Notice of Appeal was
excusable,
considering
respondents diligent efforts.

Magestrado v. People
G.R. No. 148072
July 10, 2007
Topic: Appeal
Doctrine: Certiorari generally lies
only when there is no appeal nor
any
other
plain,
speedy
or
adequate remedy available to
petitioners.

Facts: Private respondent Elena M.


Librojo filed a criminal complaint
for perjury against petitioner with
the Office of the City Prosecutor of
Quezon City. After the filing of
petitioners counter-affidavit and
the appended pleadings, the OCP
recommended
the
filing
of
information for perjury against
petitioner.
Assistant
City
Prosecutor Josephine Z. Fernandez
filed
information
for
perjury
against petitioner with MeTC.
On 30 June 1999, petitioner
filed a motion for suspension of
proceedings based on a prejudicial
question alleged that a civil case
for recovery of a sum of money
pending before the RTC. MeTC
denied the motion on the ground
that the resolution of the issues
raised in the civil actions is not
determinative of the guilt or
innocence of the accused trial shall
proceed. On 17 August 1999,
Motion for Reconsideration was
filed by petitioner but was denied
by the MeTC. Petitioner filed a
Petition for Certiorari under Rule
65 of the Revised Rules of Court,
with a prayer for Issuance of a
Writ of Preliminary Injunction
before the RTC. RTC dismissed the
petition.
petitioner filed with CA a Petition
for Certiorari under Rule 65 of the
Revised Rules of Court. Again,
petitioner filed a motion for
reconsideration
but
this
was
denied.

Aggrieved, petitioner filed a


Petition for Certiorari under the
Rule 65 on the ground that RTC
Judge
Estrella
T.
Estrada
committed
grave
abuse
of
discretion amounting to lack or
excess of jurisdiction. The Court of
Appeals dismissed the Petition on
the
ground
that
petitioners
remedy should have been an
appeal from the dismissal by RTC
of his Petition for Certiorari.

Issue: Whether or not the instant


Petition for Certiorari under Rule
65 is the correct and appropriate
remedy

Ruling:
The
Supreme
Court
affirmed the decision of CA.
Resolution or dismissal in
special civil actions, as in the
instant petition, may be appealed
under Section 10, Rule 44 of the
1997 Rules of Civil Procedure and
not by petition for certiorari under
Rule 65 of the same rules.
The Supreme Court said that
correct procedural recourse for
petitioner was appeal, not only
because RTC did not commit any
grave abuse of discretion in
dismissing petitioners Petition for
Certiorari in but also because RTC
Order of dismissal was a final
order from which petitioners
should
have
appealed
in
accordance with Section 2, Rule 41
of the Revised Rules of Court. An
order or a judgment is deemed
final when it finally disposes of a

pending action, so that nothing


more can be done with it in the
trial court.
Certiorari generally lies only
when there is no appeal nor any
other plain, speedy or adequate
remedy available to petitioners.
Here, appeal was available. It was
adequate to deal with any question
whether of fact or of law, whether
of error of jurisdiction or grave
abuse of discretion or error of
judgment which the trial court
might
have
committed.
But
petitioners instead filed a special
civil action for certiorari. The
remedies of appeal and certiorari
are mutually exclusive and not
alternative or successive. A party
cannot substitute the special civil
action of certiorari under Rule 65
of the Rules of Court for the
remedy of appeal. The existence
and availability of the right of
appeal are antithetical to the
availability of the special civil
action for certiorari.
Accordingly, the MeTC was
directed to proceed with the
hearing and trial on the merits and
to expedite proceedings therein,
without prejudice to the right of
the accused to due process.

People vs. Rocha


G.R. No. 173797
August 31, 2007
Topic: Appeal

Doctrine: The mandatory review by


the Supreme Court is only
required for cases where the
penalty imposed is death. Where
the penalty imposed is reclusion
perpetua or life imprisonment, a
review of the trial court decision is
conducted only when the accused
files a notice of appeal.

Facts: Accused-appellants were


found guilty of the crime of
Robbery
with Homicide, and
imposing upon them the penalty of
reclusion perpetua. The Court of
Appeals affirmed the decision of
RTC. Accused-appellants through
the
Public
Attorneys
Office,
appealed the Decision of the CA to
the Supreme Court.
Accused-appellant Rocha, having
been detained for more than
seventeen years, filed a Motion to
Withdraw Appeal, stating that he
intends to apply for parole.
Plaintiff-appellee People of the
Philippines, through the Solicitor
General, filed a Comment opposing
accused-appellant Rochas Motion
to Withdraw Appeal.

Issue: Whether or not the Motions


to Withdraw Appeal of accusedappellants should be granted

Ruling:
The
Supreme
Court
granted the Motions to Withdraw
Appeal of the accused-appellants.

The mandatory review by the


Supreme Court is only required for
cases where the penalty imposed is
death. Where the penalty imposed
is reclusion perpetua or life
imprisonment, a review of the trial
court decision is conducted only
when the accused files a notice of
appeal. Neither the Decision of the
Supreme Court in the Mateo case
nor the abolition of the death
penalty has changed this. As the
penalty imposed by the trial court
and the Court of Appeals in the
case at bar is reclusion perpetua,
the review by the Supreme Court
is not mandatory and, therefore,
the accused-appellants can validly
withdraw their appeal.
The Supreme Court added
that the granting of a Motion to
Withdraw Appeal is addressed to
the sound discretion of the Court.
In the case at bar, there is no
reason to deny accused-appellants
Motion
to
Withdraw
Appeal.
Plaintiff-appellees allegation that
the Motion was for the purpose of
evading the penalty of reclusion
perpetua and trifling with our
judicial system is unsubstantiated,
as the Court of Appeals imposition
of reclusion perpetua, unlike an
imposition of the death penalty,
may be entered by said appellate
court even without another review
by the Supreme Court. Neither
should the Supreme Court deny
the Motions just because of
accused-appellants intention to
apply for executive clemency, since
the granting of such executive
clemency is within the prerogative
of the Executive Department, and
not of the Supreme Court.

RULE 123- PROCEDURE IN MTC


NO CASES

Yu, Karl Alen G.


RULE 124
Topic: Procedure in the Court
of Appeals
1. Dimarucut vs People
G.R. No. 183975, Sept. 20,
2010
Doctrine:
Rule 124, SEC. 8. Dismissal
of appeal for abandonment or
failure to prosecute The Court of
Appeals may, upon motion of the
appellee or motu proprio and with
notice to the appellant in either
case, dismiss the appeal if the
appellant fails to file his brief
within the time prescribed by this
Rule, except where the appellant is
represented by a counsel de oficio.
It
is
clear
under
the
foregoing provision that a criminal
case may be dismissed by the CA
motu proprio and with notice to
the appellant if the latter fails to
file his brief within the prescribed
time. The phrase "with notice to
the appellant" means that a notice
must first be furnished the
appellant to show cause why his
appeal should not be dismissed.

The right to appeal is not a


natural right and is not part of due
process. It is merely a statutory
privilege, and may be exercised
only in accordance with the law.
The party who seeks to avail of the
same must comply with the
requirements of the Rules. Failing
to do so, the right to appeal is lost.
2. Vitto vs CA and People
G.R. No. 134981, June 18,
2003
Doctrine:
An appellant who applies for
a time extension on when to
submit himself before the court
must comply with his commitment.
Such omission is fatal to his
appeal.
The petitioners claim that he
was not aware that he must
surrender himself to the court is a
flimsy excuse. It bears stressing
that he was represented by
counsel throughout the trial and
while his case was on appeal.
Considering that he was facing a
serious charge, it was incumbent
upon him to inform his lawyer of
his whereabouts and to inquire
about the status of his case from
time to time. In the same manner,
his counsel should have exercised
the required diligence incumbent
upon him to acquaint his client
regarding the proceedings in the
appellate court.
Indeed, the law cannot
protect a party who sleeps on his
rights
or
does
some
acts
inconsistent with its prosecution.

3. Uy de Baron vs CA and
People
G.R. No. 140719, Oct. 26,
2001
Doctrine:
The Court held that there
was no grave abuse of discretion
on the part of the Court of Appeals
when it reinstated the appeal
which it earlier dismissed.
Grave abuse of discretion
implies
such
capricious
and
whimsical exercise of judgment as
is equivalent to lack of jurisdiction,
or in other words, where the
power is exercised in an arbitrary
manner by reason of passion or
personal hostility, and it must be so
patent or gross as to amount to an
evasion of positive duty or to a
virtual refusal to perform the duty
enjoined or to act at all in
contemplation of law. But where
the court has jurisdiction over the
subject matter, the orders or
decision
upon
all
questions
pertaining to the cause are orders
or decisions within its jurisdiction
and however erroneous they may
be, they cannot be corrected by
certiorari.
It cannot be said that
respondent Court of Appeals acted
capriciously,
arbitrarily
and
whimsically considering that "the
rule is always in favor of liberality
in construction so that the real
matter
in
dispute
may
be
submitted to judgment of the
court. Imperfections of form and
technicalities of procedure should
be disregarded, unless substantial

rights
would
prejudiced.

otherwise

be

4. People vs Caruncho Jr. et


al
G.R. No. 57804, Jan. 23,
1994
Doctrine:
It should be noted that the
operative words are grave abuse
of discretion. Accordingly, even if
there be an abuse of discretion, if
it is not grave and does not inflict
substantial harm, the issuance of
the prerogative writ of certiorari
will not be warranted. On this
basis, the petition must fail.
5. People vs Paradeza
G.R. No.144590, Feb. 7,
2003
Doctrine:
A
person
accused
and
convicted of an offense may
withdraw his appeal not only
because he is guilty as charged.
There could be other reasons. It
could be due to his prior erroneous
perception
of
the
applicable
provision of law, or of the decision
itself. He may feel that to seek a
pardon might be the better and
faster remedy. Regardless of his
reasons, in our view, he is within
his rights to seek the withdrawal of
his appeal. This option should not
be closed to herein accusedappellant
except
for
clearly
important substantial reasons of
law and policy.

unreasonable
searches
seizures may not be made.
6. Advincula vs CA
G.R. No. 75310, Jan. 16,
1987
Doctrine:
In securing and transferring
court
stenographic
notes,
it
devolves upon the Government and
not on the defendant to cause to be
kept, and in case of an appeal by
the defendant, to be sent to the
Appellate Court, a complete record
of the proceedings in the Court of
First Instance. When the record so
sent is deficient the appeal will not
be dismissed on motion of the
Government. When the record sent
in a criminal case does not contain
all the testimony given in the Court
below, and the testimony lacking
cannot be obtained, the case must
be remanded to the Court below
for a new trial to the extent of the
missing evidence.
Topic: Search and Seizure
1. People vs. Aruta
G.R. No. 120915, April 3,
1998
Doctrine:
Search warrants to be valid
must particularly describe the
place to be searched and the
persons or things to be seized. The
purpose of this rule is to limit the
things to be seized to those and
only those, particularly described
in the warrant so as to leave the
officers of the law with no
discretion regarding what articles
they shall seize to the end that

and

Unreasonable searches and


seizures are the menace against
which
the
constitutional
guarantees afford full protection.
While the power to search and
seize may at times be necessary to
the public welfare, still it may be
exercised and the law enforced
without
transgressing
the
constitutional
rights
of
the
citizens, for the enforcement of no
statute is of sufficient importance
to justify indifference to the basic
principles of government.
2. MHP Garments vs Court
of Appeals and Villacruz
G.R.
No.
86720,
September 2, 1994
Doctrine:
The constitutional protection
of our people against unreasonable
search and seizure is not merely a
pleasing platitude. It vouchsafes
our right to privacy and dignity
against
undesirable
intrusions
committed by any public officer or
private individual. An infringement
of this right justifies an award for
damages. The wantonness of the
wrongful seizure justifies the
award of exemplary damages.
3. People vs. Tudtud
G.R.
No.
144037,
September 26, 2003
Doctrine:
The constitutional guaranty
against unreasonable search and
seizure is not dependent upon any
affirmative act of the citizen, the

courts do not place the citizen in


the position of either contesting an
officers authority by force, or
waiving his constitutional rights;
but instead they hold that a
peaceful submission to a search or
seizure is not a consent or an
invitation thereto, but is merely a
demonstration of regard for the
supremacy of the law.
The
right
against
unreasonable search and seizure
in turn is at the top of the
hierarchy of rights, next only to, if
not on the same plane as, the right
to life, liberty and property, which
is protected by the due process
clause. This is as it should be for,
as stressed by a couple of noted
freedom advocates, the right to
personal security which, along
with the right to privacy, is the
foundation of the right against
unreasonable search and seizure
includes the right to exist, and the
right to enjoyment of life while
existing.
4. People vs. Binad Sy Chua
G.R.
No.
136066-067,
February 4, 2003
Doctrine:
A search incidental to a
lawful arrest is distinct from a stop
and frisk search. In a search
incidental to a lawful arrest, as the
precedent arrest determines the
validity of the incidental search,
the legality of the arrest is
questioned in large majority of
these cases e.g. whether an arrest
was merely used as a pretext for
conducting a search. In this
instance, the law requires that

there first be arrest before a


search can be made and the
process cannot be reversed.
A stop and frisk search
serves a two-fold interest: 1. The
general interest of effective crime
prevention and detection, which
underlies the recognition that a
police
officer
may,
under
appropriate circumstances and in
an appropriate manner, approach a
person
for
purposes
of
investigating
possible criminal
behavior even without probable
cause. 2. The more pressing
interest
of
safety
and
selfpreservation which permit the
police officer to take steps to
assure himself that the person with
whom he deals is not armed with a
deadly
weapon
that
could
unexpectedly, and fatally be used
against the police officer.
5. Malaloan vs. Court of
Appeals
G.R. No. 104879, May 6,
1994
Doctrine:
The court laid down the
following guidelines in issuing
search warrants inside or outside
the courts territorial jurisdiction,
to wit:
1.
The
court
wherein
the
criminal case is pending shall have
primary jurisdiction to issue search
warrants necessitated by and for
purposes
of
said
case.
An
application for a search warrant
may be filed with another court
only
under
extreme
and
compelling circumstances that the

applicant must prove to the


satisfaction of the latter court
which may or may not give due
course
to
the
application
depending on the validity of the
justification offered for not filing
the same in the court with primary
jurisdiction thereover.
2.
When the latter court issues
the search warrant, a motion to
quash the same may be filed in and
shall be resolved by said court,
without prejudice to any proper
recourse to the appropriate higher
court by the party aggrieved by
the resolution of the issuing court.
All grounds and objections then
available, existent or known shall
be raised in the original or
subsequent proceedings for the
quashal of the warrant; otherwise
they shall be deemed waived.
3.
Where no motion to quash
the search warrant was filed in or
resolved by the issuing court, the
interested party may move in the
court where the criminal case is
pending for the suppression as
evidence of the personal property
seized under the warrant if the
same is offered therein for said
purpose. Since two separate courts
with different participations are
involved in this situation, a motion
to quash a search warrant and a
motion to suppress evidence are
alternative and not cumulative
remedies. In order to prevent
forum shopping, a motion to quash

shall consequently be governed by


the omnibus motion rule, provided,
however,
that
objections
not
available, existent or known during
the proceedings for the quashal of
the warrant may be raised in the
hearing of the motion to suppress.
The resolution of the court on the
motion to suppress shall likewise
be subject to any proper remedy in
the appropriate higher court.
4.
Where the court which
issued the search warrant denies
the motion to quash the same and
is not otherwise prevented from
further proceeding thereon, all
personal property seized under the
warrant
shall
forthwith
be
transmitted by it to the court
wherein the criminal case is
pending,
with
the
necessary
safeguards and documentation
therefor.
5.
These
guidelines
shall
likewise be observed where the
same criminal offense is charged
in
different
informations
or
complaints and filed in two or
more courts with concurrent
original jurisdiction over the
criminal action. Where the issue of
which court will try the case shall
have been resolved, such court
shall be considered as vested with
primary jurisdiction to act on
applications for search warrants
incident to the criminal case.

Anda mungkin juga menyukai